Sie sind auf Seite 1von 222

1 BASIC PRINCIPLES, CONCEPTS, AND

DEFINITIONS
Thermodynamics stems from the Greek words therme (heat) and dynamis (force).

Thermodynamics is that branch of the physical sciences that treats various phenomena
of energy and the related properties of matter, especially of the laws of transformation of
heat into other forms of energy and vice versa.

Using thermodynamics
Engineers use principles drawn from thermodynamics and other engineering sciences
such as fluid mechanics and heat and mass transfer, to analyze and design things intended
to meet human needs. The applications of these principles are listed below in which
engineering thermodynamics is important.

Selected areas of applications of Engineering Thermodynamics


Automobile engines
Turbines
Compressor, pumps
Fossil and nuclear fueled power stations
Propulsion systems for aircraft and rockets
Combustion systems
Cryogenic systems, gas separation and liquefaction
Heating, ventilating, and air-conditioning systems
Vapor compression and absorption refrigeration
Heat pumps
Cooling of electronic equipment
Alternative energy systems
Fuel cells
Thermoelectric and thermionic devices
Magneto hydrodynamic (MHD) converters
Solar-activated heating, cooling, and power generation
Geothermal systems
Ocean thermal, wave and tidal power generation
Wind power
Biomedical Applications
Life support systems
Artificial organs

Systems
An important step in any engineering analysis is to describe what is being studied.
In mechanics, normally the first step is to draw the free body diagram and identify all the
forces acting on the given body. In thermodynamics the term system is used to identify

Page | 1
the subject of analysis. Everything external to the system is considered to be part of the
surroundings. The system is distinguished from its surroundings by a specified
boundary. These terms are illustrated in fig1-1. The boundary of a system can be fixed
or movable.

FIGURE 1-1

Type of system

There are two basic kinds of system


a. Closed system is defined when a particular quantity of matter is under study. A
closed system always contains the same matter. There can be no transfer of mass
across its boundary. A special type of closed system that does not interact in any
way with its surroundings is called an isolated system. The term control mass is
sometimes used in placed of closed system.

FIGURE 1-2 CLOSED SYSTEMS

Page | 2
A special case of a closed system is an isolated system. An isolated system is a
system that in no way interacts with its surroundings as shown in figure 1-3.

FIGURE 1-3

b. Open system is defined when a particular quantity of matter may cross the
boundary of a control volume.

m (in)
air
m steam (in)

steam work
Air work (out)
(in)

m (out)
air
m steam (out)
heat

Air in and out of the compressor Steam in and out of the turbine

Imaginary boundary
real boundary
m water

(in)
water mgas (in) m gas (out)

work (in)
Water in and out of the pump gas in and out of the nozzle
FIGURE 1-4 EXAMPLES OF OPEN SYSTEM

Page | 3
PROPERTY, STATE AND PROCESS

A property is a macroscopic characteristic of a system such as mass, volume, energy,


pressure and temperature to which a numerical value can be assigned at a given time
without knowledge of the history of the system.
State refers to the condition of a system as described by its properties. Since there are
relations among the properties of the properties of a system, the state often can be
specified by providing values of a subset of the properties.
Process is a transformation from one state to another. When any of the properties of a
system change, the state changes and the system is said to have undergone a process.
Thermodynamic cycle is a sequence of processes that begins and ends at the same state.

EXTENSIVE AND INTENSIVE PROPERTIES

Extensive property if its value for an overall system is the sum of its values for the parts
into which the system is divided. Mass, volume, energy are extensive properties can
change with time.
Intensive property are not additive; their values are independent of the size o extent of a
size o extent of a system and may vary from pace to place within the system at any
moment. Intensive properties may be functions of both position and time, whereas
extensive properties vary at most with time.

EQUILIBRIUM

Thermodynamic Equilibrium- if a system is isolated from its surrounding and there is no


changes in its observable properties at the moment it was isolated. For a system to be
in equilibrium it must be a single phase that have no tendency to change their
conditions when the overall system is isolated from its surrounding

SYSTEM OF UNITS

Unit is any specified amount of a quantity by comparison with which any other quantity
of the same kind is measured.

Primary dimensions are quantities that are conceive that is suffice to measure other
quantities. In thermodynamics the primary set of dimensions measures mass, length,
time force and temperature.
Secondary dimensions are quantities that can be measured in terms of primary dimension
such as area, velocity, pressure.

Once a set of primary dimension is adopted, a base unit for each primary dimension is
specified.

Page | 4
SYSTEM OF UNITS

Newtons law states that the acceleration of a particular body is directly proportional to
the resultant force acting on it and inversely proportional to its mass. Written as

a F
m

Rewriting the given expression in equation form,

kF ma ma
a= , F= , k=
m k F

Where k is proportionality constant.

System of units where k is unity but not dimensionless:


CGS system: 1 dyne force accelerates 1 gram mass at 1cm/s2
Mks system: 1 Newton force accelerates 1 kg mass at 1m/s2
Fps system; 1 pound force accelerates 1 slug mass at 1ft/s2

1gm 1 dyne 1 kgm 1 newton

1 cm/sec2 1 m/sec2

g m cm kg m m
k =1 k =1
dyne sec 2 newton sec 2

1 slug 1 lbf

1 ft/s2

slug ft
k =1
lb f sec 2

Page | 5
System of units where k is not unity

If the same word is used for both mass and force in a given system, k is neither unity nor
dimensionless.

1 lb force accelerates a 1 lb mass at 32.174 ft/sec2


1 g force accelerates a 1 g mass at 980.66 cm/sec2
1 kg force accelerates a 1 kg mass at 9.8066 m/sec2

1 lbm 1 lbf 1 gm 1 gf

32.174ft/sec2 980.66 cm/sec2

lbm ft g cm
k = 32.174 k = 980.66
lb f sec 2 g f sec 2

1 kgm 1 kgf

9.8066 ft/sec2

kg m m
k = 9.8066
kg f sec 2

Relation between kilogram force (kgf) and Newton (N)

kg m m kg m m
Since, k = 1 and k = 9.8066
newton sec 2 kg f sec 2

kg m m kg m m
Therefore, 1 = 9.8066
N sec 2
kg f sec 2

1kgf =9.8066N

Page | 6
Relation between pound mass (lbm) and slug

slug ft lb ft
Since, k = 1 and k = 32.174 m
lb f sec 2
lb f sec 2

slug ft lb ft
Therefore, 1 = 32.174 m
lb f sec 2
lb f sec 2

1 slug =32.174 lbm

POUNDAL is a unit of force in the English system that produces unit acceleration (1ft/s2)
in a body of unit mass (1 lbm).

1 lbm 1 poundal

1 ft/s2

1 poundal = (1 lbm) (1 ft/s2)

MASS AND WEIGHT


The mass of a body is the absolute quantity of matter in it.
The weight of a body means the force of gravity Fg on the body.

m F Fg
= =
k a g

Where g = acceleration produced by force Fg


a = acceleration produced by another force F

Problems:
1. What is the weight of a 66-kgm man at standard condition?

SOLUTION:

m = 66kg m g = 9.8066m / sec 2

Page | 7
( 66kg m ) 9.8066
m

=
Fg
mg
= sec 2
= 66kg f
k kg m m
9.8066
kg f sec 2

2. The weight of an object is 50 lb. What is its mass at standard condition?

SOLUTION:

Fg = 50lb f g = 32.174 ft / sec 2


( 50lb ) 32.174 lblb secft
f
m
2

=
m
Fg k
= f
= 50lb
m
g ft
32.174 2
sec

3. Five masses in a region where the acceleration due to gravity is 30.5 ft/s2 are as
follows: m1 is 500 g of mass; m2 weighs 800 gf; m3 weighs 15 poundals; m4
weighs 3lbf ; m5 is 0.10 slug of mass. What is the total mass expressed (a) in
grams, (b) in pounds, and (c) in slugs.

SOLUTION:

ft in cm
=
2
g 30.5 2 12 2.54 929.64cm / sec
sec ft in


(800 g ) 980.66 gg sec
f
cm
m
2

(a) =
Fg 2 k
m2 = f
= 843.91g
m
g cm
929.64
sec 2

lb . ft
15 m 2
m3 Fg 3 gm
= = sec=
ft
( 0.49lbm ) 453.6 = 222.26 g m
k g 30.5 2 lbm
sec

lb ft
3 lb f 32.174 m
Fg 4 k
lb f sec 2 gm
=
m4 = 453.6 = 1435.49 g m
g ft lb
30.5 2 m
sec

Page | 8
m5 lb g
(=
0.10 slug ) 32.174 m 453.6 m 1459.41g m
k slug lbm

Total mass= m1 + m2 + m3 + m4 + m5

=
500 + 843.91 + 222.26 + 1435.49 + 1459.41

= 4461.07 g m

4461.07 g m
(b) Total mass= = 9.83lbm
g
453.6 m
gf

9.83lbm
(c) Total mass= = 0.306 slug
lb
32.174 m
slug

4. Note that the gravity acceleration at equatorial sea level is g=32.088 fps2 and that its
variation is -0.003 fps2 per 1000 ft ascent. Find the height in miles above this point
for which (a) the gravity acceleration becomes 30.504 fps2, (b) the weight of a given
man is decreased by 5%. (c) What is the weight of a 180 lbm man atop the 29,131-ft
Mt Everest in Tibet, relative to this point?

SOLUTION:

(a) change in acceleration= 30.504-32.088= -1.584 fps2

1.584 fps 2
=
Height , h = 528, =
000 ft 100miles
0.003 fps 2
1000 ft

(b) F= 0.95 Fg Let Fg = weight of the man at sea level


a
F Fg
=
a g
h 0.95 Fg Fg
=
a g
Fg = =
a 0.95 g ( 0.95)( 32.088
= ) 30.484 fps 2

Page | 9
=h
( 30.484 32.088
=
) fps 2 =
534670 ft 101.3miles
0.003 fps 2
1000 ft
(c) F
a

29.131ft

Fg

g =32.088 fps2
m= 180 lbm

29.131 ft
a =
32.088 fps 2 ( 0.003 fps ) =
2
32.0879 fps 2
1000 ft
ft
(180lbm ) 32.001 2
=
F =
ma sec
= 179.52lb f
k lbm ft
32.174
lb f sec 2

(5). A simple instrument for measuring the acceleration of gravity employs a linear spring
from which a mass is suspended. At a location on earth where the acceleration of gravity
is 32.174 ft/s2, the spring extends 0.291 in. If the spring extends 0.116 in when the
instrument is on Mars, what is the Martian acceleration of gravity? How much would the
spring extend on the moon, where g=5.471 ft/s2?

SOLUTION:

Fg mg
Spring constant,=
c =
k
Where; = spring deflection
Since same spring scale and the same mass, were used

ft
32.174 2 ( 0.116in )
g mars =
sec
= 12.825 fps 2
0.291in

Page | 10
Reading of scale at moon:

(80kg m ) 9.675
m

=
Fgm
mm g
= sec 2
= 78.93kg f
k kg m m
9.8066
kg f sec 2

Specific Volume, Density and Specific Weight


The density of any substance is its mass (not weight) per unit volume.

m
=
V
The specific volume v is the volume of a unit mass.

V 1
=
v =
m
The specific weight of any substance is the force of gravity on unit volume.

Fg
=
V
Since the specific weight is to local acceleration of gravity as the density is to the
standard acceleration, /g = /k, conversion is easily made;

k g
= or =
g k
At or near the surface of the earth, k and g are numerically equal, so are and .

The specific gravity or relative density SG is the ratio of the density of a substance to
the density of some standard substance at a specified temperature (usually water at 4oC,
for which H2O = 1000 kg/m3.


SG =
H O
2

Problems:

1. What is the specific weight of water at standard condition?

Page | 11
Solution:

kg m
g = 9.8066m / sec 2 = 1000
m3

kg m
1000 3m 9.8066 2
g m sec
=
kg
= = 1000 3f
k kg m m m
9.8066
kg f sec 2

2. Two liquids of different densities(1=1500 kg/m3,2=500kg/m3) are poured


together into a 100-L tank, filling it. If the resulting density of the mixture is
800 kg/m3, find the respective quantities of liquids used. Also, find the weight
of the mixture; local g= 9.675 mps2.

Solution:

Mass of mixture, mm

kg
mm =
= m vm 800 3 ( 0.100=
m3 ) 80kg
m
m1 + m2 = mm

1V1 + 2V2 =
mm

1500V1 + 500V2 =
80 (1)

V1 + V2 =
0.100 (2)

Solving equations (1) and (2) simultaneously

V1 = 0.03 m3

V2 = 0.07 m3

kg
m1 =
= 1500 3 ( 0.03m
= ) 45kg
3
1V1
m

kg
m2 =
= 2V2 500 3 ( 0.07=
m3 ) 35kg
m

Page | 12
Weight of mixture,

(80kg m ) 9.675
m

=
Fgm
mm g
= =
s2
78.93kg f
k kg m .m
9.8066
kg f .s 2

PRESSURE

The standard atmospheric pressure is 760 mm Hg or 29.92 in. Hg at 320F, 14.696


psia, or 1 atm, or 101 kPa.

Measuring Pressure
1. By using manometers
(a) absolute pressure is greater than atmospheric pressure

p
p
P= absolute pressure
pg Po= atmospheric pressure
hg
Pg = gage pressure, the pressure due
to the liquid column hg
= h
P= Po + Pg

(b) Absolute pressure is less than atmospheric pressure

p= po - pg

The gage reading is called vacuum pressure or the vacuum

2. By using pressure gages

A pressure gage is a device for measuring gage pressure.

Page | 13
This picture shows the movement in one type of pressure gage, known as the
single tube gage. The fluid enters the tube through the threaded connection. As
the pressure increases, the
tube with an elliptical
section tends to straighten,
the end that is nearest the
linkage toward the right.
The linkage causes the
sector t rotate. The sector
engages a small pinion
gear. The index hand
moves with the pinion
gear. The whole
mechanism is of course
enclosed in a case, and a
graduated dial, from which
the pressure is read, and is
placed under the index
hand.

Pressure Gage
Figure 1-5

Relation bet Atmospheric pressure and the absolute pressures

Absolute pressure
(p = po + p)
g

+Pg
Atmospheric Pressure
(pg = O, p = p)
o

P -Pg vacuum
Absolute pressure (p = po - pg )
Po

P
Zero absolute or total vacuum (p = 0, pg= p )
o

Page | 14
Figure 1-6
GAGE PRESSURE

=
p po + pg
Fg V Ahg
p=
g = =
A A A
ghg ghg
pg =
= hg =
k kv

Problem

1. A 30 m vertical column of fluid (density 1878 kg/m3) is located where g= 9.65 mps2.
Find the pressure at the base of the column.

Solution

m kg m 3
sec 2 m3
9.65 1878
ghg
pg = =
kg m m
( 30 m )
k 1
N sec 2

= 543,680 N/m2 or 543.68 kPa (gage)

Atmospheric pressure
A barometer is used to measure atmospheric pressure.

Barometer
FIGURE 1-7
po = ho
Where ho= the height of column of liquid supported by atmospheric pressure Po.

Page | 15
Problems

1. A vertical column of water will be supported to what height by standard


atmospheric pressure.
:
Solution

At standard condition

w = 62.4 lb/ft 3 po = 14.7 psi

lb in 2
14.7 2 144 2
po in ft
=
ho = = 33.9 ft
w 62.4 3
lb
ft

2. The pressure of a boiler is 9.5 kg/cm2. The barometric pressure of the atmosphere
is 768 mm of Hg. Find the absolute pressure in the boiler. (ME Board Problem-
Oct.1987)

Solution:

Pg= 9.5 kg/cm2 ho= 768 mm Hg

At standard condition:

w= 1000 kg/m3

=po (=
) ( h ) ( sp gr ) ( )( h )
Hg o Hg w o

kg
13.6 1000 3 ( 0.768 m )
=po = m
1.04
kg
cm 2 cm 2
10, 000 2
m

kg
p = po + pg = 1.04 + 9.5 =10.54
cm 2

Page | 16
Absolute Pressure
p =h

Where h= ho hg, the height of column of liquid supported by absolute pressure


p.

If the liquid used in the barometer is mercury, the atmospheric pressure becomes,

po =
= Hg hg ( sp gr ) Hg ( w )( ho )


lb
(13.6 ) 62.4( ho in )
ft 3 lb
=po = 3
0.491 h o 2
in in
1728 3
ft

Where ho= column of mercury in inches

lb
Then, pg = 0.491 h g
in 2

lb
And, p = 0.491 h
in 2

Problems:
1. A pressure gage registers 40 psig in a region where the barometer is 14.5 psia.
Find the absolute pressure in psia, and in kPa.

Solution:

Page | 17

lbm
[1 kg m ] 2.205
kg m
=1 kg m = 0.06853 slug
lb
32.174 m
slug

m m ft ft
=1 2 =
1 2 3.28 3.28
s sec m sec 2

a=3.28 ft/sec2

a= 3.28 ft/sec2

( 0.06863 slug ) 3.28


ma ft
=
F = = 0.2248 lb f
k sec 2

1 newton = 0.22484 lbf

1 lbf = 4.4484 newtons

(1 lb ) 4.4484
N in
39.37
1
lb
= lb m
in 2 in 2

lb N
1 2
= 6895 2
in m

N
lb 2
=p =54.5 2 6895 m 375, 780 Pa or 375.38 kPa \
in lb

in 2

2. Given the barometric pressure of 14.7 psia (29.92 in. Hg abs), make these conversion:
(a) 80 psig to psia and to atmosphere
(b) 20 in. Hg vacuum to in. Hg abs and to psia
(c) 10 psia to psi vacuum and to Pa
(d) 15 in. Hg gage to psia, to torrs, and to Pa. (1 atm = 760 torrs)

Page | 18
Solution

(a) p = po + pg = 14.7 + 80 = 94.17 psia

80 psig
=pg = 5.44 atm
psig
14.7
atm

(b)

hg =20 in
ho=29.92 in

h = 9.92 in. Hg abs

p= 0.491 h

p= (0.491) (9.92) =4.87psia

c)

pg
po=14.7 psia

p= 10psia

pg=4.7 psi vacuum

Pa
pg = ( 4.7 psi ) 6895
psi

= 32,407 Pa (gage)

Page | 19
(d)

hg =15 in
h

ho=29.92 in

h= 29.92 + 15= 44.92 in. Hg abs

p= 0.491 h= (0.491)(44.92) =22.06 psia

=pg
(=
15 )( 760 )
381 torrs
29.92

pg= 0.491 hg

psi Pa
= 0.491 (15 in.) 6895
in. psi

= 50781.68Pa (gage)

3. The piston diameters in the fig are D1= 10cm and D2= 4 cm. If p1= 1000 kpa and
p3= 500 kpa, what is the pressure in chamber 2, in kpa?

Page | 20
Solution:
F3

F2

FV = 0 F1
F1 F2 F3 =
0
FBD CYLINDER
But F = pA

Therefore:

p1 A1 p2 A2 p 3 A3 =
0

2 2
1000 ( 0.1) p2 ( 0.04 ) 500 ( 0.12 0.042 ) =
0
4 4 4

p2 = 3625kpa

3. A pressure cooker cooks a lot faster than an ordinary pan by maintaining a


higher pressure and temperature inside. The lid of a pressure cooker is well
sealed , and steam can only escape through an opening in the middle of the
lid. A separate metal piece, the petcock, sits on top of this opening and
prevents steam from escaping until the pressure force overcomes the weight of
the petcock. The periodic escape of the steam in this manner prevents any
potentially dangerous pressure buildup and keeps the pressure inside at
constant value. Determine the mass of the petcock of apressure cooker whose
operating pressure is 100 KPa gage and has an opening cross sectional area of
4 mm2. Assume an atmospheric pressure of 101 KPa.

Page | 21
Solution:

FBD OF PETCOCK

Fv =
0

F Fg = 0

F = pA
2
kn 1m 1000 N
=F =1000 2 ( 4mm 2 ) 0.4 N
m 1000mm 1kN

Fg= 0.4 N

But
mg
Fg =
k

Page | 22
Mass of petcock

Fg k
m=
g

( 0.4 N ) 1
kg .m

=m =N .s 2
=
0.0408 kg 40.8kg
m
9.8066 2
s

4. Intravenous infusions are usually driven by gravity by hanging the fluid bottle at
sufficient height to counteract the blood pressure in the vein and to force the fluid into
the body. The higher the bottle is raised, the higher the flow rate of the fluid will be.
(a) If it is observed that the fluid and the blood pressure balance each other when the
bottle is 1.2 meters above the arm level, Determine the gage pressure of the blood.
(b) If the gage pressure of the fluid at the arm level needs to be 20 KPa for sufficient
flow rate, determine how high the bottle must be placed. Take the density f the fluid
to be 1020 kg/m3.

Solution:

g
a) =
k
For the fluid,
kg m
1020 3 9.8066 2
m s kn
= = 10.006 3
kg .m 1000 N m
2
1
N .s 1kN

Blood pressure= pressure of the fluid

p fluid = h
kn
= 10.006
= (1.2m ) 12.0074
= kn / m 2 12.00744kPaa
m3

Page | 23
b) For sufficient flow rate of fluid, height of IV bottle

kn
20
p m 2 = 2m
h= =
10.006m3

5. The water in a tank is pressurized by air, and the pressure is measured by a multifluid
manometer as shown. Determine the gage pressure of air in the tank if h1= 0.2 m, h2=
0.3 m, and h3= 0.46 m. Take the densities of water , oil, and mercury to be 1000
kg/m3, 850 kg/m3, and 13,600 kg/m3, respectively.

Solution:
g
=
k
kg m
1000 3 9.8066 2
m s
= water = 9.8066kn / m3
kg .m 1000n
1 2
n.s kn

kg m
850 3 9.8066 2
m s
= oil = 8.3356kn / m3
kg .m 1000n
1 2
n.s kn

kg m
13600 3 9.8066 2
m s
= mercury = 133.37 kn / m3
kg .m 1000n
1 2
n.s kn
For the manometer assembly:

Page | 24
pair water h1 + oil h2 mercury h3 =
0

pair = water h1 oil h2 + mercury h3

pair = 9.8066 ( 0.2 ) 8.3356 ( 0.3) + 133.37 ( 0.46 )


pair = 60.81kpa

Temperature

Absolute temperature is the temperature measured from absolute zero.


Absolute zero temperature is the temperature at which all molecular motion ceases.
Absolute temperature will be denoted by T, thus

T=0
R t 0 F + 460 , degrees rankine
= t 0C + 273 , Kelvin
TK

Degrees Fahrenheit (0F) and degrees Centigrade (0C) indicate the temperature reading
(t).Fahrenheit degrees (F0) and centigrade degrees (C0) indicate temperature change or
difference ( t).

180 F0=100 C0
5
1 F0= C 0
9
9
1 C0= F 0
5
It follows that,

1 F0= 1 R0
And

1 C0= 1 K0

1. Derive the relation between degrees Fahrenheit and degrees Centigrade. (EE Board
Question)

Page | 25
0
F 32 0 C 0
=
212 32 100 0

0
9
=
0
F C + 32
5

5 0
=
0
C ( F 32)
9

2. Show that the specific heat of a substance in Btu/(lb) (F0) is numerically equal to cal/
(g) (C0).

Solution

[ Btu ] 252
cal
Btu Btu
1 =
5 C0
[lb] 454 F 0 0
lb.F 0 g
lb 9 F

Btu cal
1 =1
lb.F 0
( g ) (C 0 )

4. Fred Suave, an old- fashioned engineering student believes that the boiling point
of water is best suited for use as the reference point on temperature scales.
Unhappy that the boiling point corresponds to some odd number in the current
absolute temperature scales, he has proposed a new absolute temperature scale
that he calls the Suave scale. The temperature unit on this scale is suave, denoted
by S, and the boiling point of water on this scale is assigned to be 1000 0S.
Determine the ice point of water on the Suave scale and obtain a relation between
the Suave and Celsius scale.

373K 1000S

273K t

0K 0S

Ice point at Suaves scale

373 273 1000 t


=
373 1000
Page | 26
100
=t 1000 1000
373

= 731.9o S

b) relation bet oC and oS

100 o C 1000 o S
=
100 1000 731.9

0
S = 1000 (100 o C ) ( 268.1) /100
0
S =1000 268.1 + ( 26.81) ( 0 C )

= 731.9 + 26.81 (0C)

Conservation of Mass Principle

Mass and volume flow rates

Mass Flow Rate (m ) , is the amount of mass flowing through a cross section per unit
time.
Volume Flow rate ( V ), is the volume fo the fluid flowing through a cross section per unit
time.
.
The quantity of fluid passing through a given section is given by the formula

V = A

FIGURE 1-8

Where: = mean fluid velocity normal to A


A= cross-sectional area of the stream normal to flow direction

The mass flow rate and volume flow rate is related by,

Page | 27
V A
m=
= = A
v v

Where: V = volume flow rate


A= cross sectional area of the stream
= average speed
m = mass flow rate
= fluid density
v= fluid specific volume

Conservation of Mass

The conservation of mass can be expressed as the net mass transfer to or from a
system during a process is equal to the net change (increase or decrease) in the total mass
of the system during that process. That is,

Total mass - Total mass = Net Change in mass (stored mass)


Entering the system leaving the system within the system

m inlet mexit =
mstored where mstored = m final minitial

In rate form:

mstored
m inlet m exit =
m stored where: m stored =
time

Mass Balance for a control volume;

Single stream (one-inlet ;one- exit) system mass flow

Page | 28
Multi stream (with two o more inlet and exit)
FIGURE 1-9


Mass balance for a control volume If m stored = 0

m inlet = m exit


m1 + m3 = m 2 + m 4

Illustrations:

1. Two inlet ; one exit

Mass balance; m 1 + m 2 =
m 3

2. single stream

Page | 29
Mass balance: m 1 = m 2

1 A11 = 2 A22 , 1 2

For incompressible flow( t1= t2 and is constant)

m 1 = m 2
A11 = A22

Problems

1. Two gaseous streams enter a combining tube and leave as a single mixture. These
data apply at the entrance section:
For one gas, A1= 75 in.2, 1 = 500 fps , v1=10 ft3/lb
For the other gas, A2= 50 in2, m 2 = 16.67 lb/s, 2 = 0.12 lb/ft3
=
At exit, 3 350
= fps, v3 7 ft 3 / lb
Find (a) the speed 2 at section 2, and (b) the flow and area at the exit section

Solution

lb
16.67
m2 s = 400 fps
=
(a) 2 =
A2 2 50 2 lb
144 ft 0.12 ft 3

lb
(b) m 3 = m 1 + m 2 = 26.04 + 16.67 = 42.71
sec

ft 75 2

A11
500 ft
sec 144 26.04 lb
=
m 1 = =
v1 ft 3
sec
10 lb

lb
m 3 = m 1 + m 2 = 26.04 + 16.67 = 42.71
sec

Page | 30
lb ft 3
42.71 7
m 3v3 sec lb
=
A3 = = 0.8542 ft 2
3 ft
350 sec

2. A 10 ft diameter by 15-ft height vertical tank is receiving water ( =62.1 lb/cu ft) at
the rate of 300 gpm and is discharging through a 6 in ID line with a constant speed of 5
fps. At a given instant, the tank is half full. Find the water level and the mass change in
the tank 15 min. later.

Solution:

gal
300 min lb lb
Mass flow rate entering= 62.1 3 = 2490.6
gal ft min
7.48 ft 3

6 ft lb
2

Mass flow rate leaving = A = ft 3
ft 5 x 60 62.1
4 12 min

lb
= 3658
min

Mass change = (3658 2490.6) (15) = 17511 lb (decreased)

Page | 31
17,511 lb
Volume change= = 282 ft 3
lb
62.1 3
ft

282 ft 3
Decreased in height = = 3.59 ft
78.54 ft 2

Water level after 15 min = 7.5 3.59 = 3.91 ft

5. A cyclone separator like that jn the fig is used to remove fine solid particles, such
as fly ash, that are suspended in a gas stream. In the flue gas system of a coal
fired power pant, the mass fraction of fly ash in the exhaust gases is
approximately 0.001. Determine the mass flow rates of exhaust gases and the flue
gas when 10 kg/hr of fly ash is collected at the ash pit.

SOLUTION:

mash
= .001
mexhaust

m ash = 10kg / hr

= m ash + m flue
m exhaust
mash
= m ash + m flue
.001

Page | 32
kg
10
=hr 10 kg + m
flue
.001 hr

10
m flue= 10= 9990kg / hr
.001
kg
10
m= = hr 10, 000kg / hr
exhaust
.001

Page | 33
Review Problems

1. What is the mass in grams and the weight in dynes and in gram force of 12 oz of
salt? Local g is 9.65 m/s2 1 lbm=16 oz
Ans. 340.2 gm;328,300 dynes; 334.8 gf

2. A mass of 0.10 slug in space is subjected to an external vertical force of 4 lb. If the
local gravity acceleration is g= 30.5 fps2 and if friction effects are neglected, determine
the acceleration of the mass if the external vertical force is acting (a) upward and (b)
downward.
Ans. (a) 9.5 fps2; (b) 70.5 fps2
3. The mass of a given airplane at sea level (g= 32.1 fps2) is 10 tons. Find its mass in lb,
slugs, and kg and its (gravitational) weight in lb when it is travelling at a 50,000 ft
elevation. The acceleration of gravity g decreases by 3.33 x 10-6 fps2 for each foot of
elevation
Ans. 20,000 lbm; 621.62 slugs; 19,850 lbf

4. A lunar excursion module (LEM) weights 1500 kg, on earth where g= 9.75 mps2. What
will be its weight on the surface of the moon where gm =1.70 mps2. On the surface of the
moon, what will be the force in kgf and in newtons required to accelerate the module at
10 mps2 ?

Ans. 261.5 kgf; 1538.5 kgf; 15,087 N

5. The mass of a fluid system is 0.311 slug, its density is 30 lb/ft3 and g is 31.90 fps2.
Find (a) the specific volume, (b) the specific weight, and (c) the total volume.
Ans. (a) 0.0333 ft3/lb; (b) 29.75 lb/ft3; (c) 0.3335 ft3

6. A cylindrical drum (2 ft diameter, 3- ft height) is filled with a fluid whose density is


40 lb/ft3. Determine (a) the total volume of a fluid, (b) its total mass in pounds and slugs,
(c) its specific volume, and (d) its specific weight where g= 31.90 fps2.
Ans. (a) 9.43 ft 3; (b) 377.2 lb;11.72 slugs; (c) 0.025 ft3/lb; (d) 39.66 lb/ft3.

7. A weatherman carried and aneroid barometer from the ground floor to his office atop
the sears tower in Chicago. On the ground level, the barometer read 30.150 in Hg
absolute; topside it read 28,607 in. Hg absolute. Assume that the average atmospheric air
density was 0.075 lb/ft3 and estimate the height of the building.
Ans. 1455 ft

8. a vacuum gauge mounted on a condenser reads 0.66 m Hg. What is the absolute
pressure in the condenser in kPa when the atmospheric pressure is 101.3 kPa?
Ans. 13.28 kPa

Page | 34
9. Convert the following readings of pressure to kPa absolute, Assuming that the
barometer reads 760 mm Hg; (a) 90 cm Hg gage; (b) 40 cm Hg vacuum; (c) 100 psig; (d)
8 in. Hg vacuum, and (e) 76 in. Hg. Gage.
Ans. (a) 221.24 kPa; (b) 48 kPa; (c) 790.83 kPa; (d) 74.213 kPa; (e) 358.591
kPa

10. A fluid moves in a steady flow manner between two sections in a flow line. At
section 1: A1, = 10 ft2, 1 =100 fpm, v1= 4 ft3/lb. At section 2: A2, = 2 ft2, 2 =0.20 lb/ft3.
Calculate (a) the mass flow rate and (b) the speed at section 2.
Ans. (a) 15,000 lb/h; (b) 10.42 fps

11. If a pump discharges 75 gpm of water whose specific weight is 61.5 lb/ft3 (g = 31.95
fps2), find (a) the mass flow rate in lb/min, and (b) and total time required to fill a vertical
cylinder tank 10 ft in diameter and 12 ft high.
Ans. (a) 621.2 lb/min; (b) 93.97 min
12. Fresh water and sea water flowing in parallel horizontal pipelines are connected to
each other by a double U-tube manometer as shown. Determine the pressure difference
between the two pipelines. Take the density of sea water at that location to be
=1035kg/m3.

13. The gage pressure of the air in the tank shown is measured to be 80 kpa. Determine
the differential height of the mercury column.

Page | 35
14. The top part of the water tank is divided into two components as shown. Now a
fluid with an unknown density is poured into one side , and the water level rises a
certain amount on the other side to compensate for this effect. Based on the final
heights on the figure, determine the density of the fluid added. Assume the liquid
does not mix water.

Page | 36
2 Conservation of Energy

Forms of Energy
Gravitational Potential energy (PE)

The gravitational potential energy of a body is its energy due to its position or
elevation.

Figure 2-1
mgz
PE=Fg z =
k
mg
PE2-PE1 = (z2-z1)
k

P E= change in potential energy, in KJ or Btu

The potential energy per unit mass is denoted by

PE
pe = , in KJ/kg
m
g(z z )
pe = 2 1
k
pe = change in potential energy per unit mass, in KJ/kg or Btu/lb

Page | 37
Kinetic energy (KE)

The energy or stored capacity for performing work possessed by a moving body,
by virtue of its momentum is called kinetic energy.

Figure 2-2

m 2
KE=
2k
m 2
E = E2 KE1 = (2 12 )
2k

E = Change in kinetic energy, in KJ or Btu

The change in kinetic energy per unit mass is given by

KE
ke =
m
( 2 2 )
ke = 2 1
2k
ke = change of potential energy per unit mass in KJ/kg or BTU/lb

Internal Energy (U, u)

Internal energy is energy stored within a body or substance associated. to the


molecular structure of a system and the degree of the molecular activity, and they are
independent of outside reference frames and can be viewed as the sum of the kinetic and
potential energies of the molecules.

u= specific internal energy (unit mass) u= u2 u1

U= mu = total internal energy (m mass) U = U 2 U1

Page | 38
Work (W)

Work is the product of the displacement of the body and the component of the
force in the direction of the displacement. Work is energy in transition; that is, it exists
only when a force is moving through a distance.
To cite examples, A rising piston, a rotating shaft, an electric wire crossing the
boundary (as shown in Figure 2-3) are all associated with work interactions
.

Figure 2-3

Where: We = is the electrical work


V2
W=
e VI= I =2
R , V=voltage, I= current , R= resistance
R
Wpiston (or Wn ) = is the moving boundary work or non flow work

Wshaft = is the shaft work= 2 n T, T= torque, n = rotational speed in rpm

Work is also a form of energy transfer like heat, therefore has energy units of kJ or BTU.
The work done per unit mass of a system is denoted by w and is expressed as

W
w= , ( in kJ/kg, ft-lbf / lbm or BTU/lbm)
m

The work done per unit time is called power and is denoted by W and is expressed as

W
W= (in kJ/sec or kw ; ft-lbf / sec; BTU/ sec or hp)
t


The relationship among w, W, and W is shown figure 2-4

Page | 39
W = 30kJ
30kJ
m = 2kg
work
t =5sec


W = 6kw
w = 15kJ / kg

Figure 2-4

Work of a Nonflow System

Figure 2-5

The work done as the piston moves from e to f is

=
dW F=
xdx ( pA=
)dL pdV

Which is the area under the curve e-f on the pV plane. Therefore, the total work done
as the piston moves from 1 to 2 is

2
W = pdV
1

Page | 40
Which is the area under the curve
1-e-f-2.

The area under the curve of the process on the pV plane represents the work done
during a nonflow reversible process.

Work done by the system is positive (outflow of energy)


Work done on the system is negative (inflow of energy)

Flow Work(Wf)

Flow work or flow energy is work done in pushing a fluid across a boundary,
usually into or out of a system.

Figure 2-6

W=
f = pAL
FL

W f = pV

W f = W f 2 W f 1 = p2V2 p1V1

W f = Change in flow work


Specific flow work is given by

wf = pv

Heat (Q)

Heat is defined as the form of energy that is transferred between two systems(or a
system and its surrounding) by virtue of a temperature difference.

As a form of energy, heat denoted by Q, has energy units KJ and BTU being the most
common one.

Page | 41
Heat transfer per unit mass of a system is denoted by q and is determined from

Q
q= (KJ/kg or BTU/lbm)
m

Rate of heat transfer (the amount of heat transfer per unit time) is denoted by Q , where
the over-dot stands for per unit time.

The heat transfer rate has units kJ/sec, which is equivalent to kW. And BTU/min or
BTU/hr.


The relationship among q, Q, and Q is shown figure 2-7

Q = 30 KJ
30kJ
m = 2kg
heat
t =5sec


Q = 6kw
kJ
q = 15
kg

Figure 2-7
Convention:
Q is positive when heat is added to the body or system.
Q is negative when heat is rejected by the body or system.

Energy Change of a system, Esystem

In the absence of electric, magnetic, and surface tension effects(i.e. for simple
compressible systems), the change in the total energy of a system during a process is the
sum of the changes in its internal, kinetic and potential energies . This can be expressed
as:

Energy change = Energy at final state - Energy at initial state


E system = E final - E initial = E 2 - E1
=U + PE + KE

Page | 42
Energy Transport by Mass, Emass

Mass flow in and out of the system serves as an additional mechanism of energy
transfer. When mass enters a system the energy of the system increases because mass
carries energy with it. Likewise, when mass leaves the system the energy contained
within the system decreases because the leaving mass takes out some energy with it.
For example, when some heat hot water is taken out of a water heater and is replaced
by the same amount of cold water, the energy content of the hot water tank decreases
as s result of this mass interaction.

The fluid entering or leaving a control volume possesses four forms of energy,
internal, kinetic, potential, and flow work as compared to control mass. The total
energy of a flowing fluid on a unit-mass basis (denoted by emass) is expressed as

gz 2
emass = pe + ke + u + w f = + + u + pv
k 2k

The total energy of a flowing fluid of mass m is

Amount of energy transport:

gz 2 kJ Btu
= m +
Emass + u + pv or
k 2k kg lb

Rate of Energy Transport:

gz 2 Btu
= m +
E mass + u + pv kW or
k 2k min

Conservation of Energy

The law of conservation of energy states that energy is neither created nor
destroyed.

The first law of thermodynamics states that one form of energy maybe converted
into another and therefore , every bit of energy should be accounted for during a
process.

Page | 43
Energy Accounting or Energy Balance

The conservation of energy principle may be expressed as follows: The net change
(increase or decrease) in the total energy of the system during a process is equal to the
difference between the total energy entering and the total energy leaving the system
during that process. That is during a process,

total energy total energy change in the total


- =
enetering the system leaving the system energy of the system

Or
Ein - E out = E system
Net change in energy change in internal, kinetic
transfer by heat, work, potential, etc. energies
and mass

General Energy Equation

Noting that energy can be transferred in the forms of heat, work and mass and
that the net transfer of a quantity is equal to the difference bet the amounts transferred in
and out, the energy balance can be written more explicitly as

( Qin Qout ) + (Win Wout ) + ( Emass ,in Emass ,out ) =


Esystem
Where the subscripts in and out denote quantities that enter and leave the system,
respectively. All six quantities on the left side of the equation represent amounts and
thus they are positive quantities. The direction of any energy transfer is described by the
subscripts in and out.

Energy Balance for Closed System:

For closed system no mass flow across the boundaries, therefore Emass=0, and for
stationary system KE = PE = 0 .
Illustration 1:

Figure 2-8

Page | 44
Since t Qout =Win = 0

( Qin Qout ) + (Win Wout ) + ( Emass ,in Emass ,out ) =


Esystem

( Qin 0 ) + ( 0 Wout ) =
U

Q - Wn = U
Q = U + Wn

Where:Wout= Wn= non-flow work or moving boundary work

Illustration 2:

Figure 2-9

Since t Qin =Wout = 0

( Qin Qout ) + (Win Wout ) + ( Emass ,in Emass ,out ) =


Esystem

( 0 Qout ) + (Win 0 ) =U

-Q + Wp = U
Where:Win= Wp= paddle work
Wp = U + Q

Page | 45
Steady Flow Energy Equation

Characteristics of steady flow system.

1. There is neither accumulation nor diminution of mass within the system.


2. There is neither accumulation nor diminution of energy within the system.
3. The state of working substance at any point in the system remains constant.

Illustration:

Figure 2-10

( Qin Qout ) + (Win Wout ) + ( Emass ,in Emass ,out ) =


Esystem

During a steady flow process, the total energy content of a control volume remains
constant (Ecv = constant), therefore the change in the total energy of the control volume is
zero (Ecv = 0).

( Qin Qout ) + (Win Wout ) + ( Emass ,in Emass ,out ) =


0

Or the energy balance can be expressed simply as

Ein = Eout
Net energy transfer in net energy transfer out
By heat, work, and mass By heat, work, and mass

Therefore,

Emass (1) + Q = E mass( 2) + W

PE1 + KE1 + U1 + W f 1 + Q = PE2 + KE 2 + U 2 + W f 2 + W

Page | 46
In rate form:

g z
1 2 g z 2
m + 1 + w f 1 + u1 + Q = m 2
+ 2 + w f 2 + u2 + W
k 2k k 2k


Dividing the equation by m gives the energy equation on a unit-mass basis as

g z1 12 g z2 2 2
+ + w f1 + u 1 + q = + + w f 2 + u2 + w
k 2k k 2k

Enthalpy (H, h)

Enthalpy is a composite property applicable to all fluids and is defined by

h= u + pv and H= mh= U + pV

The steady flow energy equation becomes

PE1 + KE1 + H1 + Q= PE2 + KE2 + H 2 + W

In rate form:

pe1 + ke1 + w f 1 + u1 + q = pe2 + ke2 + w f 2 + u2 + w

gz 2 gz 2
m 1 + 1 + h1 + Q = m 2 + 2 + h 2 + W
k 2k k 2k

On a unit mass basis:

gz1 12 gz 2 2 2
+ + h 1 + q = + + h2 + w
k 2k k 2k

Page | 47
Energy Conversion Efficiency
Performance( Efficiency)= desired output/ required output

The efficiencies of various devices are defined as;

Pump efficiency


E mech , fluid
Mechanical energy increase of the fluid W pump ,u
pump
= = =
Mechanical energy input W shaft ,in W pump


Where: W pump ,u = useful pumping power supplied to the fluid

Turbine efficiency:


Mechanical energy output W shaft ,out W turbine
turbine
= = =
Mechanical energy decrease of the fluid
E mech , fluid W turbine ,e

Where: W turbine = mechanical power extracted from the fluid by the turbine

Motor efficiency


Mechanical power output W shaft ,out
=motor =
Electric power input W elect ,in

Generator Efficiency:


Electric power output W elect ,out
= generator =
Mechanical power input W
shaft ,in

Overall Efficiencies:

W pump ,u E mech , fluid
pump
= motor pump
= motor =
W elect ,in W elect ,in

Page | 48
And


W elect ,out W elect ,out
turbine
= gen turbine=
generator =

W turbine ,e E mech , fluid

Problems

1. During a steady flow process, the pressure of the working substance drops from
200 to 20 psia, the speed increases from 200 to 1000 fps, the internal energy of
the open system decreases 25 Btu/lb, and the specific volume increases from 1 to
8 ft3. No heat is transferred. Sketch an energy diagram. Determine the work per
lb. Is it done on or by the substance? Determine the work in hp for 10 lb per min.
(1 hp = 42.4 Btu/min.)

Solution

p1= 200 psia p2=20 psia 1=200 fps 2=1000 fps

v1=1 ft3/lb v2=8 ft3/lb u =25 Btu/lb Q=0

Basis unit-mas

pe1 + ke1 + w f 1 + u1 + q = pe2 + ke2 + w f 2 + u2 + w

gz1 12 gz 2 2 2
+ + u1 + w f 1 + q = + + u2 + w f 2 + w
k 2k k 2k
2
ft
2 200
=
v
ke1= 1 = s
0.80
BTu
2k lb ft ft lb f lbm
(2) 32.174 m 2 778
lb f s BTu

Page | 49
[1000]
2
v22 BTu
=ke = = 19.97
2k (2) [32.174][ 778]
2
lbm

lb in.2 ft 3
in.2 ft 2 lbm
200 144 1
Btu
=
w p=
1v1 = 37.02
f1
ft lb f lbm
778
BTu

lb in.2 ft 3
in.2 ft 2 lbm
20 144 8
=
w p= = 29.61
Btu
2 v2
f2
ft lb f lbm
778
BTu

ke1 + w=
f1 ke2 + w f 2 + u + w

0.8 + 37.02= 19.97 + 29.61 25 + w

Btu
w = 13.24 ( by )
lbm

Btu lb

13.24 10
lbm min
=W = 3.12hp
Btu
42.4
( min )( hp )
2. Steam is supplied to a fully loaded 100-hp turbine at 200 psia with u1= 1163.3
Btu/lb, v1= 2.65 ft3/lb and 1 = 400 fps. Exhaust is at 1 psia with u2 = 925 Btu/lb,
v2= 294 ft3/lb and 2 = 1100 fps. The heat loss from the steam in the turbine is 10
Btu/lb. Neglect potential energy change and determine (a) the work per lb steam
and (b) the steam flow rate in lb/h.

Page | 50
Solution

p1=200 psia u1=1163.3 Btu/lb v1=2.65 ft3/lb

P2= 1 psia u2= 925 Btu/lb v2= 294 ft3/lb

1=400 fps 2= 1100 fps Q= -10 Btu/lb

(a) Basis unit-mass

pe1 + ke1 + w f 1 + u1 + q= pe2 + ke2 + w f 2 + u2 + w

ke1 + w f 1 + u1 + q = ke2 + w f 2 + u2 + w

( 400 )
2
12 Btu
=
ke = = 3.20
1
2k ( 2 )( 32.174 )( 778) lbm

(1100 ) =
2
22 Btu
=
ke = 24.17
2
2k ( 2 )( 32.174 )( 778) lbm

=
w p=
( 200 )(144 )( 2.65
=
) Btu
f1 1v1 98.10
778 lbm

=
w p=
(1)(144 )( 295
=
) Btu
f2 2 v2 54.6
778 lbm

ke1 + w f 1 + u1 + q = ke2 + w f 2 + u2 + w

3.20 + 98.10 + 1163.3 + ( 10


= ) 24.17 + 54.6 + 925 + W
Btu
w = 251
lbm

Page | 51
Btu
(100hp ) 2544

( hr )( hp ) lb
=
(b) steam flow= m = 1014 m
Btu hr
251
lbm

3. An air compressor (an open system) receives 272 kg per min of air at 99.29 kPa
and a specific volume of 0.026 m3/kg. The air flows steady through the
compressor and is discharged at 689.5 kPa and 0.0051 m3/kg. The initial internal
energy of the air is 1594 J/kg; at discharge, the internal energy is 6241 J/kg. The
cooling water circulated around the cylinder carries away 4383 J/kg of air. The
change in kinetic energy is 896 J/kg increase. Sketch an energy diagram. Compute
the work.

Solution

p1=99.29 kPa v1= 0.026 m3/kg u1= 1594 J/kg Q= -4383 J/kg m= 272 kg/min

p2= 689.5 kPa v2= 0.0051 m3/kg u2= 6241 J/kg K =


896 J/kg

On a unit-mass basis:

pe1 + ke1 + w f 1 + u1 + q= pe2 + ke2 + w f 2 + u2 + w

ke1 + w f 1 + u1 + q = ke2 + w f 2 + u2 + w

kN m3 kj
=
w f1 p=
1v1 =
99.29 m 2 0.026 kg 2.583 kg

kN m3 kJ
=
w f2 p=
2 v2 =
689.5 m 2 0.0051 kg 3.516 kg

Page | 52
w f 1 + u1 + q =ke + w f 2 + u2 + w

2.582 + 1.594 4.383 = 0.896 + 3.516 + 6.241 + w

kJ
w = 10.86
kg

kJ kg
W = 10.86 272
kg min

kJ
W = 2954
min

4. A centrifugal pump operating under steady flow conditions delivers 2,270 kg/min
of water from an initial pressure of 82, 740 Pa to a final pressure of 275,800 Pa.
The diameter of the inlet pipe to the pump is 15.24 cm and the diameter of the
discharge pipe is 10.16 cm. What is the work?

Solution

m= 2270 kg/min d1= 0.1524 m p1= 82,740 Pa =1000 kg/m3


d2= 0.1016 m p2= 275,800 Pa


= = ( 0.1524 ) 0.01824m2
2
Area at entrance, A1
4


Area= = ( 0.1016 ) 0.008107m2
2
at exit, A2
4

2270 kg m

m 60 s
1 =
Speed at entrance,= = 2.074m / s
p1 A1 kg m
1000 m3 0.01824m
3

Page | 53
2270 kg m

m 60 s
=
Speed at exit, 2 = = 4.667 m / s
p2 A2 kg m
1000 m3 0.008107 m
3

Basis unit-mass

2
m
2.074
1 s N m
2
=
ke = = 2.151
1
2k kg m m
( 2 )1 N s 2
kg m

2
m
4.667
2
2
N m
=
s
=
ke = 10.89
2
2k kg m m
( 2 )1 N s 2
kg m

N
82, 740
p1 m=2 N m
w= p1=
v1 = 82.74
f1
1 kg
1000 3 kg m
m

N
275,800
p2 m=2 N m
w= p2=
v2 = 275.8
f2
2 kg
1000 3 kg m
m

pe1 + ke1 + u1 + w f 1 + q = pe2 + ke2 + u2 + w f 2 + w

ke1 + w f 1 = ke2 + w f 2 + w

2.151 + 82.74 = 10.89 + 275.8 + w

N m
w = 201.8
kg m

N m kg
W = 201.8 2270
kg m min

Page | 54
kJ
W = 458.1
min

5. A turbine operates under steady flow conditions, receiving steam at the following
state: pressure 1200 kPa, temperature 188oC, enthalpy 2785 kJ/kg, speed 33.3 m/s
and elevation 3 m. The steam leaves the turbine at the following state; pressure 20
kPa, enthalpy 2512 kJ/kg, speed 100 m/s and elevation 0 m. Heat is lost to the
surroundings at the rate of 0.29 kJ/s. If the rate of steam flow through the turbine
is 0.42 kg/s, what is the power output of the turbine in kW?

Solution

z1=3 m z2= 0m
kJ kJ
h1= 2785 h2= 2512
kg kg

m m
1=33.3 2= 100
s s

kJ kg
Q = -0.29 m = 0.42
sec sec

Basis unit-mass

m
9.8066 2 ( 3m )
gz1 s KJ
=
pe = = 0.0294
1
k kg m J kg
2
1 1000
N s kJ

2
m
33.3
1
2
s KJ
=
ke = = 0.5544
1
kg m
2k
( 2 )1 2 kg
N s

Page | 55
2
m
100
2
2
=
s KJ
=
ke = 5.000
2
kg m
2k
( 2 )1 2 kg
N s

kJ
0.29
q= s = 0.6905 kJ
kg kg
0.42
s

pe1 + ke1 + h1 + q= pe2 + ke2 + h2 + w

pe1 + ke1 + h1 + q = ke2 + h2 + w

0.0294 + 0.5544 + 2785 + ( 0.6905


= ) 5.000 + 2512 + w
kJ
w = 267.9
kg

kJ kg
W = 267.9 0.42
kg s


W = 112.52kW

6. The demand for electric power is usually much higher during the day than it is at
night, and utility companies like NAPOCOR often sell power at much lower
prices at night. Suppose NAPOCOR is selling electric power for Php 1.50/kWh at
night and is willing to pay Php 4.00/kWh for power produced during the day. To
take advantage of this opportunity, an entrepreneur is considering building a large
reservoir 40 M above the lake level, pumping water from the lake to the reservoir
at night letting the water flow from the reservoir back to the lake during the day,
producing power as the pump motor operates as a turbine generator during
reverse flow. Preliminary analysis shows that water flow rate of 2m3/s can be
used in either direction. The combined pump motor and turbine generator
efficiencies are expected to be 75% each. Disregarding the losses in piping and
assuming the system operates for 10 hrs each in the pump and turbine modes
during a typical day, determine the potential revenue this pump turbine system
can generate per year.

Page | 56
Solution:

cost to produce electricity = P1.50 kw/hr


selling price of produced electricity = P4.00 kw/hr

z1 = 40m pump motor = 75% turbine generator = 75%

kg m3 kg
=m =
water V 1000 3
=2 2000
m sec sec

When operating as a pump:


Basis unit mass:

pe1 + ke1 + w f 1 + u1 + q = pe2 + ke2 + w f 2 + u2 + w

0 = pe2 + w

m
9.81 2 ( 40m )

gz sec kJ
w=
2 = =
0.3924 (work done on)
k kg m nm kg
2
1 1000
n sec kJ

kg kJ
=
W m=
w 2000 0.3924 = 784.8kW
sec kg

W mech , fluid 784.8kW
=
W elect ,in = = 1046.4kW
pump motor 0.75

Page | 57
Incurred cost to operate pump-motor:


Cincurred = W elec in ( cost to produce electricity )( no of hrs in operation )

10hrs 365day
Cincurred / year (1046.4
= kW ) ( P 1.50/kW-hr ) P 5, 729, 040
day year

When operating as a turbine

pe1 + ke1 + w f 1 + u1 + q = pe2 + ke2 + w f 2 + u2 + w

pe1 = w

m
9.81 2 ( 40m )
=
gz1
w = sec
= 0.3924
kJ
(work done by)
k kg m nm kg
2
1 1000
n sec kJ

kg kJ
=
W m=
w 2000 0.3924 = 784.8kW
sec kg

= =
W elect ,out W mech , fluid turbine gen =
784.8 kW ( 0.75 ) 588.6kW

Cost to sell the energy produced


Cselling = W elec ouy ( selling price of electricity )( no of hrs in operation )

10hrs 365day
Cselling / year 588.6kW ) ( P 4.00/kW-hr )
(= P 8,593,560
day year

Revenue= Cselling / year - Cincurred / year

Re venue = P 8,593,560 - P 5,729,040 = P 3,224,520

7. A classroom that normally contains 40 people is to be air conditioned with


window type air conditioning units of 5 kW cooling capacity. A person at rest
may be assumed to dissipate heat at a rate of about 360 kJ/hr. There are 10 light
bulbs in the room each with a rating of 100 watts. The rate of heat transfer to the
room through the walls and the window is estimated 15.000 kJ/hr. If the room air

Page | 58
is at a constant temp of 210C. Determine the number of window type air
conditioner required.

Solution:

kJ
No of people= 40 Q people at rest / person = 360
hr

No of lights= 10 Q lights / unit = 100watts

kJ
Q walls + window+ = 15,000 Q air con / unit = 5 kw
hr

kJ
360 hr kJ
=Q people (=40 person ) 14, 400
person hr

kJ
1
100 watts sec 3600sec 3600 kJ

=Q lights (10
= lights ) 1000 watts =
lights 1000 watts hr hr


Q in =
Qout


Q aircon / unit ( no. of units )
Q people + Q lights + Q walls + windows =


Q + Q lights + Q walls + windows 14400 + 3600 + 15000
no of units= people =
= 1.83units
kJ 3600sec
Q aircon / unit 5
sec 1hr
Use 2 window type air-conditioner

8. Air enters the diffuser of a jet engine steadily with a velocity of 200 m/s and an
enthalpy of 283.4 kj/kg. The inlet area of the diffuser is 0.4 m2. The specific
volume of entering air is taken as 1.015 m3/kg. The air leaves the diffuser with a
velocity that is very small compared with the inlet velocity. Determine a) mass
flow rate of the air b) the exit enthalpy.
Solution:
m3 m kJ
v1 = 1.015 1 = 200 h1 = 283.4 A1 = 0.4m 2
kg sec kg

Page | 59
m
0.4m 2 200
A11

sec kg
a) =
m = 3
= 78.81
v1 m sec
1.015
kg

b) Basis unit-mass
2
m
12 200
=
ke = sec
= 20
kJ
kg m 1000n m
1
2k kg
2 1 2
n sec 1kJ

pe1 + ke1 + h1 + q = pe2 + ke2 + h2 + w

ke1 + h1 =
h2

kJ kJ kJ
h2 =
20 + 283.4 =303.4
kg kg kg


kg kJ kJ
=
H 2 m= h 2 78.81 303.4= 23,910.95
sec kg sec

9. The power output of an adiabatic steam turbine is 5 MW. The inlet conditions are
as follows: z1= 10 m, 1= 50 m/s, p1= 2 mpa, u1=2945.2 KJ/ kg, v1= 0.1512
m3/kg. The exit conditions are as follows: p2=15 kpa, 2= 180 m/s, z2= 6 m, u2=
2226.44 kj/kg, v2= 9.019 m3/kg. Determine a) The work done per unit mass of
steam flowing the turbine. b) Calculate the mass flow rate of steam.

Solution


W = 5MW z1= 10 m 1= 50 m/s p1= 2 mpa

u1=2945.2 KJ/ kg v1= 0.1512 m3/kg. p2=15 kpa 2= 180 m/s

Page | 60
z2= 6 m u2= 2226.44 kj/kg v2= 9.019 m3/kg

a) Basis unit-mass

m
9.81 2 (10m )
=
pe
gz1
= sec
= 0.0981
kJ
1
k kg m 1000n m kg
2
1
n sec 1kJ

m
9.81 2 ( 6m )
=
pe
gz2
= sec
= 0.05886
kJ
2
k kg m 1000n m kg
2
1
n sec 1kJ

2
m
12 50
=
ke = sec = 1.25
kJ
1
kg m 1000n m
2 ( )
2k kg
1 2
n sec 1kJ

2
m
2 2 180
=
ke = sec
= 16.2
kJ
2
kg m 1000n m
2 ( )
2k kg
1 2
n sec 1kJ

kn m3 kJ
=
w f1 p=
1v1 2000 2 0.1512 = 302.4
m kg kg

kn m3 kJ
=
w f2 p=
2 v2 15 2 9.019 = 135.285
m kg kg

pe1 + ke1 + u1 + w f 1 + q = pe2 + ke2 + u2 + w f 2 + w

0.0981 + 1.25 + 302.4 + 2945.2


= 0.05886 + 16.2 + 135.285 + 2226.44 + w

kJ
w = 870.96 (done by the system)
kg

Page | 61
kJ
5000
W sec= 5.74 kg
= =
b) m
w 870.96 kJ sec
kg

10. Refrigerant-134a is to be cooled by water in a condenser. The refrigerant enters


the condenser with an enthalpy of 303.85 kj/kg and leaves with an enthalpy of
100.87 kj/kg. the cooling water enters at 1.7057 KPa with an internal energy of
62.98 kj/kg and with an density of 999 kg/m and leaves at 3.17 KPa with an
internal energy of 104.83 kj/kg with a density of 1000kg/m. The refrigerant mass
flow rate is 6 kg/min. Determine (a) The mass flow rate of the cooling water
required (b) The heat absorbed by the cooling water

Solution:

For Refrigerant 134-a For cooling water:



kg
=
m1 m=
2 = 6
m refrigerant p3= 1.7057 kPa
min
u3= 62.98 kJ/kg
kJ
h1 = 303.85 3= 999 kg/m3
kg
p4= 3.17 kPa
kJ
h2 = 100.87 u4= 104.83 kj/kg
kg
4= 1000kJ/kg
a)

Mass balance:


min = mout

For each fluid stream since there is no mixing,



=
m1 m=
2 m refrigerant

m=
3 m=
4 m water

Energy Balance:

E in = E out


Since=Q 0,=
W 0, ke=0, pe=0

Page | 62
p p
m1 h1 + m3 3 + u3 = m2 h2 + m4 4 + u4
3 4

Combining and rearranging:

p p
m refrigerant ( h1=
h2 ) m water 4 + u4 3 + u3
4 3

kN kN
3.17 m 2 kJ 1.7057 2
m + 62.98 kJ kg kJ kJ
m water + 104.83 = 6 303.85 100.87
kg
1000 3 kg 999 kg kg min kg kg
m m 3

kg
m water = 29.1
min
b)

E in = E out

kg kJ kJ
Q water m refrigerant ( h=
= 1 h2 ) 6 303.85 100.87
min kg kg

kJ
Q water = 1217.88
min

Page | 63
Review Problems

1. Assuming that there are no heat effects and no frictional effects, find the kinetic
energy and speed of a 3220-lb body after it falls 778 ft from rest. Start with the
steady flow equation, deleting energy terms which are irrelevant.
Ans. 225 fps

2. A reciprocating compressor draws in 500 cubic feet per minute of air whose
density is 0.079 lb/cu ft and discharges it with a density of 0.304 lb/cu ft. At the
suction, p1=15 psia; at discharge, p2=80 psia. The increase in the specific internal
energy is 33.8 Btu/lb and the heat transferred from the air by cooling is 13 Btu/lb .
Determine the work on the air in Btu/min and in hp. Neglect change in kinetic
energy.
Ans. 56.25 hp

3. Steam enters a turbine with an enthalpy of 1292 Btu/lb and leaves with an
enthalpy of 1098 Btu/lb. The transfer heat is 13 Btu/lb. What is the work in
Btu/min and in hp for a flow of 2 lb/sec?
Ans. 512.3 hp

4. A thermodynamic steady flow system receives 4.56 kg per min of a fluid where
p1=137.90 kPa, v1=0.0388 m3/kg, 1=122 m/s, and u1=17.16 kJ/kg. The fluid
leaves the system at a boundary where p2=551.6 kPa, v2=0.193 m3/kg, 2= 183
m/s and u2=52.80 kJ/kg. During passage through the system the fluid receives
3000 J/s of heat. Determine the work.
Ans. -486 kJ/min

5. Air flows steadily at the rate of 0.5 kg/s through an air compressor, entering at 7
m/s speed, 100 kPa pressure and, 0.95 m3/kg specific volume, and leaving at 5
m/s, 700 kPa, and 0.19 m3/kg. The internal energy of the air leaving is 90 kJ/kg
greater than that of the air entering. Cooling water in the compressor jackets
absorbs heat from the air at the rate of 58 kW. Compute the work in kW.
Ans. -122 kW

6. In a steady flow apparatus, 135 kJ of work is done by each kg of fluid. The


specific volume of the fluid, pressure, and speed at the inlet are 0.37 m3/kg, 600
kPa, and 16 m/s. the inlet is 32 m above the floor, and the discharge pipe is at
floor level. The discharge conditions are 0.62 m3/kg , 100 kPa, and 270 m/s. The
total heat loss between the inlet and discharge is 9 kJ/kg of fluid. In flowing
through this apparatus, does the specific internal energy increase or decrease, and
by how much?
Ans. -20.01 kJ/kg

7. Steam enters a turbine stage with an enthalpy of 3628 kJ/kg at 70 m/s and leaves
the same stage with an enthalpy of 2846 kJ/kg and a velocity of 124 m/s.
Calculate the work done by the steam

Page | 64
Ans. 776.8 kJ/kg (ME board problem Oct. 1986)
8. A hot water streams enters a mixing chamber with a mass flow rate of 0.5 kg/s
and an enthalpy of 2482.2 kj/kg where it is mixed with a stream of cooled water
with an enthalpy of 83.96 kj/kg. If it is desired that the mixture leave the chamber
with an enthalpy of 175.91 kj/kg. Determine the mass flow rate of the cool water
stream.
9. A desktop computer is to be cooled by a fan. The electronic components of the
computer consume 60 W of power under full load conditions. The computer is to
operate in environments at temp. up to 45 C and at elevations up to 3400 m
where the average atmospheric pressure is 66.63 KPa, air density at this condition
0.73 kg/m. The exit temp. of air is not to exceed 60 C, air density of 0.7 kg/m,
to meet reliability requirements. Also the average velocity of the air is not to
exceed 110 m/min at the exit of the computer case where the fan is installed to
keep the noise level down. Assume that there is an increase in internal energy of
10.779 kj/kg. Determine the flow rate of the fan that needs to be installed and the
diameter of the casing of the fan.
10. A hair dryer is basically a duct in which a few layers of electric resistors are
placed. A small fan pulls the air and forces it through the resistors where it is
heated. Air enters a 1200 W hair dryer with a density of 1.19 kg/m. The rise in
internal energy is 17.96 kj/kg. The cross sectional area of the hair dryer at the exit
is 60 cm. neglecting the power consumed by the fan and heat losses through the
walls of the hair dryer. Determine (a) volume flow rate air at the inlet (b) velocity
of the air at the exit.

Page | 65
3 The Ideal Gas
An ideal gas is ideal only in the sense that it conforms to the simple perfect gas
laws.

Boyles Law

If the temperature of a given quantity of gas is held constant, the volume of the
gas varies inversely with the absolute pressure during a change of state.

1 C
V or V =
p p

pV=C or p1V1 = p2V2

Charles Law

(1) If the pressure on a particular quantity of gas is held constant, then, with any
change of state, the volume will vary directly as the absolute temperature.

V T or V=CT

V V1 V2
=C or =
T T1 T2

(2) If the volume of a particular quantity of gas is held constant, then, with any
change of state, the pressure will vary directly as the absolute temperature.

PT or p=CT

P P1 P2
=C or =
T T1 T2

Equation of State or Characteristic Equation of a Perfect Gas

Combining Boyles and Charles Laws,

p1V1 p2V2
= = C , a constant
T1 T2

pV
= mR
T

Page | 66
pV = mRT

pv = RT
(unit mass)

Where p= absolute pressure


V=volume
v=specific volume
m= mass
T= absolute temperature
R= specific gas constant or simply gas constant

P V m T R

lb f o ft lb f
English units ft 3 lbm R
ft 2
lbm o R

N N m
SI Units m3 kg K
m2 kg K

Problems

1. A drum 6 in. in diameter and 40 in. long contained acetylene at 250 psia and 90oF.
After some of the acetylene was used, the pressure was 200 psia and the
temperature was 850F, (a) What proportion of the acetylene was used? (b) What
volume would the used acetylene occupy at 14.7 psia and 800F? R for acetylene is
59.35 ft. lb/lb0R.

Solution

(a) Let m1= mass of acetylene initially in the drum


m2= mass of acetylene left in the drum
m3= mass of acetylene used
p1= 250 psia
T1=900F + 460 = 5500R
P2= 200 psia
T2= 850F + 460 = 5450R

( 6 ) ( 40 )
2

volume of the drum= = 0.6545 ft 3


( 4 (1728) )

Page | 67
=
m1
p1V1
=
( 250 )(144 )( 0.6545
=
) 0.7218 lb
RT1 ( 59.35)( 550 )

=
m2
p2V2
=
( 200 )(144 )( 0.6545
=
) 0.5828 lb
RT2 ( 59.35)( 545)

m3 = m1 m2 = 0.7218 0.5828 =1390 lb

m3 0.1390
=
Acetylene used= = 0.1926 or 19.26 O O
m1 0.7218

(b) p3 = 14.7 psia

T3= 800F + 460 = 5400R

=V3
m3 RT3
=
( 0.139 )( 59.35)(=
540 )
2.105 ft 3
p3 (14.7 )(144 )
2. The volume of a 6 x 12-ft tank is 339.3 ft3. It contains air at 200 psig and 850F. How
many 1ft3 drums can be filled to 50 psig and 800F if it is assumed that the air temperature
in the tank remains at 850F? The drums have been sitting around in the atmosphere which
is at 14.7 psia and 800F.

Solution

Let m1= mass of air initially in the tank


m2= mass of air left in the tank
m3= mass of air initially in the drum
m4= mass of air in the drum after filling

p1= 200+ 14.7=214.7 psia p3=14.7 psia


T1= 85 + 460= 5450R T3= 80 + 460 = 5400R
p2=50 + 14.7=64.7 psia p4= 50 + 14.7 = 64.7 psia
T2= 85 + 460 = 5450R T4= 80 + 460 = 540 0R

For the tank

=
m1
p1V1
=
( 214.7 )(144 )( 339.3
=
) 360.9 lb
RT1 ( 53.34 )( 545)

=
m2
p2V2
=
( 64.7 )(144 )( 339.3
=
) 108.7 lb
RT2 ( 53.34 )( 545)

Page | 68
Mass of air that can be used= 360.9 108.7 =252.2 lb

For the drums

=
m3
p3V3
=
(14.7 )(144 =)(1) 0.0735 lb
RT3 ( 53.34 )( 540 )

=
m4
p4V4
=
( 64.7 )(144=)(1) 0.3235 lb
RT4 ( 53.34 )( 540 )
Mass of air filled in each drum = 0.3235 0.0735= 0.25 lb

252.2
Number of drums filled up = = 1009 drums
0.25

3. It is planned to lift and move logs from almost inaccessible forest areas by means of
balloons. Helium at atmospheric pressure (101.325 kPa) and temperature 21.10C is to be
used in the balloons. What minimum balloon diameter (assume spherical shape) will be
required for a gross lifting force of 20 metric tons?

Let ma= mass of air displaced by the balloon

mhe = mass of helium

V = volume of the balloon

For air
J
Ra = 287.08
kg K

Pa= 101.325 Pa

Ta= 21.1 + 273 =294.1 K

Page | 69
For helium

J
Rhe = 2077.67
kg K
Phe = 101,325 Pa

The= 21.1 + 273 = 294.1 K

pheV 101,325 V
=
mhe = = 0.1658 kg
RheThe ( 2077.67 )( 294.1)
=
ma mhe + 20, 000

=
1.2001 V 0.1658 V + 20, 000

V = 19,337 m3

4 3
r = 19,337
3

r= 16.65 m

d= 2(16.65) =33.3 m

4. Two vessels A and B of different sizes are connected by a pipe with a valve. Vessel A
contains 142 L of air at 2767 .92 kPa, 93.330C. Vessel B, of unknown volume, contains
air at 68.95 kPa, 4.44 0C. The valve is opened and, when the properties have been
determined, it is found that pm=1378.96 kPa, Tm= 43.330C. What is the volume of vessel
B?

Solution

For vessel A

Pa=2,767.92 kPa
Va= 142 Liters
Ta= 93.33 + 273 = 366.33 K

For vessel B

Pb= 68.95 kPa

Tb= 4.44 +273= 277.44 K

Page | 70
For the mixture

Pm = 1378.96 kPa
Tm = 43.33 + 273 = 316.33 K

m=
m ma + mb

pmVm paVa pbVb


= +
RTm RTa RTb

(1378.96 ) Vm
=
( 2767.92 )(142 ) + ( 68.95) Vb
316.33 366.33 277.44

=
4.36 Vm 1072.9 + 0.25 Vb (1)

=
Vm 142 + Vb (2)

Solving equations 1 and 2 simultaneously,

Vb=110.4 liters

5. A mass of 2.4 kg of air at 150 kpa and 12oC and is contained in a gas- tight, frictionless
piston-cylinder device. The air is now compressed to a final pressure of 600 kPa. During
the process, heat is transferred from the air such that the temperature inside the cylinder
remains constant. Calculate the final volume of the piston of the piston cylinder
assembly.

Solution:

m = 2.4kg
T1 = 273 + 12 = 2850 k
p1 = 150kpaa
p2 = 600kpaa

kn m
( 2.4kg ) 0.28708
kg K
0 ( 2850 K )
=
V1
mRT
=
p1 kn
150 2
m
V1 = 1.312m3

Page | 71
p1V1 = p2V2

p1V1 150kpa (1.312m )


3

=
V2 = = 0.3255m3
p2 600kpa

6. A cylinder glass tubing with an inside diameter of 30mm and 900 mm long with one
end closed is immersed vertically with the open end down into a tank of cleaning
solvent (sp gr =0.73) until only 50mm of its length remain above the liquid surface.
If the atmospheric pressure is 101 kpa, how high will the fluid rise in the tube?

Solution:

From Boyles Law,

p1V1 = p2V2
p1 = 101kpa

V1 = ( 0.03) ( 0.90 )
2

4
101 + 9.81( 0.73) y =
p2 = 101 + 7.16 y

= ( 0.03) ( y + 0.05)
2
V2
4


(101kpa ) ( 0.03) ( 0.90 ) =
(101 + 7.16 y ) ( 0.03) ( y + 0.05)
2 2

4 4

y = 0.8014m

Rise of fluid in the tube=0.85 - 0.8014

The fluid will rise 48.6 mm inside the tube.

Page | 72
Specific Heat

The specific heat of a substance is defined as the quantity of heat required to


change the temperature of unit mass through one degree.
In dimensional form,

heat (energy units)


c
(mass)(change of temperature)

In differential quantities,

dQ
c= or dQ=mcdT
mdT

and for a particular mass m,

2
Q = m cdT
1
(the specific heat equation)

If the mean or instantaneous value of specific heat is used,

dT mc (T2 T1 )
2
Q=mc=
1
( constant specific heat)

Constant Volume specific Heat (cv)

Qv= U

Qv= mcv(T2-T1)

Page | 73
Constant Pressure specific heat (cp)

=
Q p mc p (T2 T1 )
2
Qp =
U + W = U +
1
pdV
Q p =U + p ( v 2 v1 )

= U 2 U1 + p2V2 p1V1

Qp =
H 2 H1 =
H

Ratio of specific heat

cp
k= 1
cv

Internal Energy of an Ideal Gas

Joules Law states that The change of internal energy of an ideal gas is a
function of only the temperature change. Therefore, U is given by the formula,

U mcv (T2 T1 )
=

Whether the volume remains constant or not.

Enthalpy of an Ideal Gas

The change of enthalpy of an ideal gas is given by the formula,

H=mc p (T2 T1 )

Whether the pressure remains constant or not.

Page | 74
Relation Between Cp and Cv

From h=u+pv and pv=RT

dh = du + RdT

CpdT=CvdT + RdT

C=
p Cv + R
R
Cv =
k 1
kR
Cp =
k 1

Problems

ft lb
1. For a certain ideal gas R = 25.8 and k= 1.09 (a) What are the values of Cp and
lb0 R
Cv? (b) What mass of this gas would occupy a volume of 15 ft3 at 75 psia and 800F?
(c) If 30 Btu are transferred to this gas at constant volume in (b), what are the
resulting temperature and pressure?

Solution

(a)=
Cp=
kR (=
1.09 )( 25.8 )
312.47
ft lb
or 0.4016
Btu
k 1 1.09 1 lb R 0
lb R 0

C 0.4016 Btu
Cv=
= p = 0.3685
k 1.09 lb R 0

(b) V=15 ft3 p= 75 psia T=80 + 460 = 5400R

=
m=
pV (=
75 )(144 )(15 )
11.63 lb
RT ( 25.8)( 540 )

Q=mcv (T2 T1 )
=30 11.63 ( 0.3685 )(T2 540 )
T2 = 547 0 R

547
= (T2 / T1 ) 75
p2 p1= = 76 psia
540

Page | 75
2. For a certain gas R=320 J/kg K and Cv=0.84 kJ/kg. K0 (a) Find Cp and k. (b) If 5 kg
of this gas undergo a reversible nonflow constant pressure process from V1=1.1133
m3 and p1= 690 kPa to a state where T2= 5550C, find U and H.

Solution

(a)
kJ
C p = CV + R = 0.84 + 0.32 = 1.16
kg k 0
R 0.32
k= + 1= + 1= 1.381
Cv 0.84

(b)

=
T1
p1V1
=
( 690, 000 )(1.133 )
= 488.6 k
mR ( 5)( 320 )
=
U mCv (T2 =T1 ) 5 ( 0.84 )( 828 488.6 )

= 1425.5 kJ

H mC p (T2 =
= T1 ) 5 (1.16 )( 828 488.6 )

= 1968.5 kJ

3. Consider a well insulated horizontal rigid cylinder that is divided into two
compartments by a piston that is free to move but does not allow either gas to leak
into other side. Initially, one side of the piston contains 1m3of nitrogen gas at 150
kPa and 80 oC while the other side contains 1 m3 of helium gas at 500 kPa and 25
o
C. Now thermal equilibrium is established in the cylinder as a result of heat
transfer through the piston. Determine a) the final temperature in the cylinder b)
the final volume of each compartment c) if the piston were not free to move, what
would be the final temperature? Cp of nitrogen= 1.0414 kJ/kg.Ko cv= 0.7442
kJ/kg. Ko, for helium cp =5.2028 kJ/kg.Ko , cv= 3.1233 kJ/kg.Ko .

Solution:

a) Since piston is free to move, process is constant pressure

Page | 76
nitrogen helium

For Nitrogen:

V = 1m3
p = 150kpaa
T =80 + 273 =3530 K

cp = 1.0414 kJ/kg.Ko

cv= 0.7442 kJ/kg. Ko

= c p cv
R
kj
R =1.0414 0.7442 = 0.2972
kg K
150 2 (1m3 )
kn
pV m
=m = = 1.43kg
RT knm
0.2972 kg 0 K ( 353 K )
0


For Helium:

V = 1m3
p = 500kpaa
T =25 + 273 =2980 K

cp =5.2028 kJ/kg.Ko

cv= 3.1233 kJ/kg.Ko

= c p cv
R
kj
R = 5.2028 3.1233 = 2.0795
kg K

2 (
1m3 )
kn
500
pV m
=
m = = 0.807 kg
RT knm
2.0795 kg 0 K ( 298 K )
0

Page | 77
Heat absorbed by helium = Heat rejected by Nitrogen

( mc t )
p helium
=( mc p t )
nitrogen

( 0.807kg ) ( 5.2028=
kJ / kg.K o )(T f 2980 K ) (1.43kg ) (1.0414 kJ / kg.K o )( 3530 K T f )

Tf = 312.40K

b)
At thermal Equilibrium,
Final pressure and temperature of Helium and Nitrogen are equal

mR mR
=

V f helium V f nitrogen

0.807 ( 2.0795 ) / V f helium = 1.43 ( 0.2972 ) / V f nitrogen

V f helium =
(.807 )( 2.0795) V
(1.43)( 0.2972 ) f nitrogen

V f helium = 3.94V f nitrogen (1)

V f helium + V f nitrogen =
2 m3 (2)

Substitute (1) in (2)

3.94V f nitrogen + V f nitrogen =


2m3

2
=
V f nitrogen = 0.4048m3
4.94

V f helium = 1.5952m3

c) If the piston does not move, Volume is constant

Heat absorbed by helium = Heat rejected by Nitrogen

( mcv t )helium =( mcv t )nitrogen

Page | 78
( 0.807kg ) ( 3.1233=
kJ / kg.K o )(T f 2980 K ) (1.43kg ) ( 0.7442kJ / kg.K o )( 3530 K T f )

T f = 314.320 K

4. If you ever slapped someone or got slapped yourself, you probably remember the
burning sensation. Imagine you had the unfortunate occasion of being slapped by
an angry person, which caused the affected area of the face to rise by 1.8oC (ouch!).
Assuming the slapping hand has a mass of 1.2 kg and about 0.15 kg of the tissue of
the face and the hand is affected by the incident, estimate the velocity of the hand
just before impact. Take the specific heat of the tissue to be 3.8 kJ/kg.oC. Assume
there is no change in potential energy and there is no heat transferred from the
affected area to the surroundings.

Solution:
Ein = Eout

U tissue =
KEhand
m 2
( v )tissue
mc t =
2k hand
2 ( mcv t )tissue
=
mhand
kg m
2 1 2 (
0.15kg ) ( 3.8kJ / kg .0 K )(1.8o K )
= n sec 1000n

1.2kg 1kn
= 41.4m / s

5. A student living in a 4-m x 6-m x 6-m dormitory room turns on her 150-W fan before
she leaves the room on a summer day, hoping that the room will be cooler when she
comes back in the evening. Assuming all the doors and windows are tightly closed
and disregarding any heat transfer through the walls and the windows, determine the
temperature in the room when she comes back 10 hrs later. Assume the room to be at
100kPa and 15oC in the morning when she leaves.

Solution:

Size of room=4-m x 6-m x 6-m p1=100kPa

J
= =
W 150 watts 150 Ti= 15 + 273= 288 K
sec

Page | 79
Energy balance

W= Q

=Q mcv (T f Ti ) Where Tf =final room temp.


Ti= initial room temp

T f = 331.19 K

100 2 (144m3 )
kN
pV m
=
m = = 174.17 kg
RT1 kn m
0.28708 kg K ( 288 K )

(10hrs )
J 3600sec 1kJ
W =
150 5400kJ
sec 1hr 1000

kJ
=5400kJ 174.17 kg 0.7186 (T f 288 K )
kg K

T f = 331.19 K =58.19 C

Entropy (S,s)

Entropy is the property of a substance which remains constant if no heat enters or


leaves the substance, while it does work or alters its volume, but which increases or
diminishes when a small amount of heat enter or leave.
Entropy can be viewed as a measure of molecular disorder or molecular
randomness. As a system becomes more disordered, the positions of the molecules
becomes less predictable and the entropy increases.
The change of entropy of a substance receiving (or delivering) heat is defined by

dQ 2
dS =
T
or S = 1 dQ
Where: dQ = heat transferred at the temperature T
S= total change of entropy

mcdT 2
S =
T 1

2 dT T
= 1 T mc ln T12
S mc=

Page | 80
Temperature- Entropy Coordinates

dQ = TdS

2
Q = TdS
1

The area under the curve of the


Process on the TS plane represents
The quantity of heat transferred
During the process.

Other Energy Relations

2
Vdp
= Ws +
1

(Reversible steady flow, P=0)

The area behind the curve of the


Process on the pV planes represents
The work of a steady flow process
When K =0, or it represents K
When Ws=0.

Page | 81
Any process that can be made to go in the reverse direction by an infinitesimal
change in the conditions is called a reversible process otherwise, it is irreversible.

Review Problems

1. An automobile tire is inflated to 32 psig pressure at 500F after being driven the
temperature rise to 750F. Determine the final gage pressure assuming the volume
remains constant.
Ans. 34.29 psig (EE Board Problem)

2. If 100 ft3 of atmospheric air at zero Fahrenheit temperature are compressed to a


volume of 1ft3 at a temperature of 2000F, what will be the pressure of the air in
psi?
Ans. 2109 psia (EE Board Problem)

3. A 10 ft3 tank contains gas at a pressure of 500 psia, temperature of 850F and a
weight of 25 pounds. A part of the gas was discharged and the temperature and
pressure changed to 700F and 300 psia, respectively. Heat was applied and the
temperature was back to 850F. Find the final weight, volume, and pressure of the
gas.
Ans. 15.43 lb; 10 ft3; 308.5 psia (EE board problem)

4. Four hundred cubic centimeters of a gas at 740 mm Hg absolute and 180C


undergoes a process until the pressure becomes 760 mm Hg absolute and the
temperature 00C. What is the final volume of the gas?
Ans. 365 cc (EE board problem)

5. A motorist equips his automobile tires with a relief-type valve so that the pressure
inside the tire never will exceed 240 kPa (gage). He starts a trip with a pressure of
200 kPa (gage) and a temperature of 230C in the tires. During the long drive,the
temperature of the air in the tires reaches 830C. Each tire contains 0.11 kg of air.
Determine (a) the mass of air escaping each tire, (b) the pressure of the tire when
the temperature returns to 230C.
Ans. (a) 0.0064 kg; (b)182.48 kPa (gage)

6. A 6 m3 tank contains helium at 400 K and is evacuated from atmospheric pressure


to a pressure of 740 mm Hg vacuum. Determine (a) mass of helium remaining in
the tank, (b) mass of helium pumped out, (c) the temperature of the remaining
helium falls to 100C. What is the pressure in kPa?
Ans. (a) 0.01925 kg; (b) 0.7123 kg; (c) 1.886 kPa

Page | 82
7. An automobile tire contains 3730 cu in. of air at 32 psig and 800F. (a) What mass
of air is in the tire? (b) In operation, the air temperature increases to 1450F. If the
tire is inflexible, what is the resulting percentage increase in gage pressure? (c)
What mass of the 1450F air must be bled off to reduce the pressure back to its
original value?
Ans. (a) 0.5041 lb; (b) 17.53%; (c) 0.0542 lb

8. A spherical balloon is 40 ft in diameter and surrounded by air at 600F and 29.92 in


Hg abs. (a) If the balloon is filled with hydrogen at a temperature of 700F and
atmospheric pressure, what total load can it lift? (b) If it contains helium instead
of hydrogen, other conditions remaining the same, what load can it lift? (c)
Helium is nearly twice as heavy as hydrogen. Does it have half the lifting force?
R for hydrogen is 766.54 and for helium is 386.04 ft. lb/lb.0R.
Ans. (a) 2381 lb; (b) 2209 lb

9. A reservoir contains 2.83 cu m of carbon monoxide at 6895 kPa and 23.60C. An


evacuated tank is filled from the reservoir to a pressure of 3497 kPa and a
temperature of 12.40C, while the pressure in the reservoir decreases to 6205 kPa
and the temperature to 18.30C. What is the volume of the tank? R for CO is
296.92 J/kg. K0
Ans. 0.451 m3

10. A gas initially at 15 psia and 2 cu ft undergoes a process to 90 psia and 0.60 cu ft,
during which the enthalpy increases by 15.5 Btu; Cv= 2.44 Btu/lb. R0. Determine
(a) U, (b) Cp, and (c) R
Ans. (a) 11.06 Btu; (b) 3.42 Btu/lb.R0; (c)762.4 ft lb/lb0 R

11. For a certain gas, R= 0.277 kJ/kg.K and k=1.384, (a) What are the value of Cp and
Cv? (b) what mass of this gas would occupy a volume of 0.425 cu m at 517.11
kPa and 26.70C? (c) if 31.65 kJ are transferred to this gas at constant volume in
(b), what are the resulting temperature and pressure?
Ans. (a) 0.7214 and 0.994 kJ/kg.R0; (b) 2.647 kg; (c) 43.27 0C, 545.75 kPa

12. A mass of 0.1 kg of helium fills a 0.2 m3 rigid vessel at 350 kpa. The vessel is
heated until the pressure is 700 kpa. Calculate the final temperature, for helium
R= 2077.67 kJ/kg.oK, cp=5.2028 kJ/kg . oK, cv= 3.1233 kJ/kg. oK.

13. A 1 m3 tank containing air at 25oC and 500 kPa is connected through a valve to
another tank containing 5 kg of air at 35oC and 200 kPa. The valve is opened, and
the entire system was allowed to reach thermal equilibrium with the surroundings,
at 20oC. Determine the volume of the second tank and the final equilibrium
pressure of air. Ans. 2.21 m3, 284.1 kPa

Page | 83
14. Nitrogen in the amount of 0.2 kg fills a 0.05 m3 piston cylinder device at 400
kPa. The piston is now moved by changing the weights until the volume is twice
its original size. During this process, Nitrogens temperature is maintained
constant. Determine the final pressure in the device.

15. A rigid tank contains 20 lbm of air at 50 psia and 80oF. The air is now heated
until its pressure doubles. Determine (a) the volume of the tank (b) the amount of
heat transfer.

16. The increase of enthalpy of an ideal gas is 141.7 kJ when the pressure increases
from 103.4 kPaa to 1034 kPaa and the volume decreases from 477liters to 74.5
liters. Determine (a) the change of internal energy (b) the final temperature of the
gas if the initial temperature is 28.30C. Ans (a) 11.4 kJ (b) 1980C

Page | 84
4 Processes of Ideal Gases
Constant Volume Process

An isometric process is a reversible constant volume process. A constant volume


process may be reversible or irreversible.

ISOMETRIC PROCESS

(a) Relation between p and T

T2 p2
=
T1 p1

(b) nonflow work

2
=
Wn =
pdV 0
1

(c) The change of internal energy.


=U mcv (T2 T1 )

(d) The heat transferred.

=
Q Wn + U

Since Wn=0

Q=U =mcv (T2 T1 )

Page | 85
(e) The change of enthalpy.

H mc p (T2 T1 )
=

(f) The change of entropy.

2
dQ 2 dT
= mcv = mcv [ ln T ]
2
=
S 1 T 1 T 1

T2 p
=
S mc=
v ln mcv ln 2
T1 p1

(g) Reversible steady flow constant volume

(1)
Q = U + KE + W f + Ws + PE

Ws = ( W f + KE + PE )

Ws = W f = V ( p1 p2 ) , if PE =0, KE =0

mR (T2 T1 )
Ws =

(2)
2
Vdp= Ws + KE
1

V ( p2 p1=
) Ws + KE

V ( p1 p2=
) Ws + KE

mR(T1 T2=
) Ws + KE

(1 k ) U= Ws + KE

(h) Irreversible nonflow constant volume process.

Q=U + Wn

For reversible nonflow, Wn=0


For irreversible nonflow, Wn 0

Page | 86
WHERE:

Wn=nonflow wok

Ws=steady flow work

Problems

1. Ten cu ft of air at 300 psia and 4000F is cooled to 1400F at constant volume. What
are (a) the final pressure, (b) the work, (c) the change of internal energy, (d) the
transferred heat, (e) the change of enthalpy, and (f) the change of entropy?

Solution

m= 10ft3 V= 10 cu ft p1=300 psia T1= 400+460=8600R


0
T2=140+460=600 R

(a)=
p2
p1T2
=
( 300 )( 600
=
) 209 psia
T1 860

(b) W=0

lb in 2
2 (
300 2
144 10 ft 3 )
p1V1 in ft
(c)=
m = = 9.417 lb
ft lb
( 860 R )
RT1 0
53.34
lb R


=U mcv (T2 T1 )

= ( 9.417 )( 0.1714 )( 600 860 ) =


420 Btu

(d) Q =
mcv (T2 T1 ) =
420 Btu

Page | 87
H mc p (T2 T1 ) = ( 9.417 )( 0.24 )( 600 860 ) =
(e) = 588 Btu

T2
(f) S =mcv ln
T1
600 Btu
= ( 9.417 ) (0.1714) ln = 0.581 0
860 R

2. There are 1.36 kg of gas, for which R= 377 J/kg. K and k= 1.25, that undergo a
nonflow constant volume process from p1=551.6 kPa and T1=60 C to p2=1655
kPa. During the process, the gas is internally stirred and there are also added
105.5 kJ of heat. Determine (a) T2, (b) The work input and (c) the change of
entropy.

Solution

k= 1.25 R= 377 J/kg-k m=1.36 kg


Q=105.5 kJ p1=551.6 kPa
p2=1655 kPa T1=60+273 =3330 K

(a) =
T2
T1 p2
=
( 333)(1655
=
) 9990 K
p1 551.6

R 377 J
(b) =
cv = = 1508
k 1 1.25 1 kg K


=U mcv (T2 T1 )

= (1.36 )(1.508)(=
999 333) 1366 kJ

Wn = Q U = 105.5 1366 = 1260.5 kJ

T2 999 kJ
=
(c) S mc=
v ln (1.36 )(1.508) ln = 2.253
T1 333 K

Page | 88
3. A group of 50 persons attended a secret meeting in a room which is 12 meters
wide by 10 meters long and a ceiling of 3 meters. The room is completely sealed
off and insulated. Each person gives off 150 kcal per hour of heat and occupies a
volume of 0.2 cubic meters. The room has an initial pressure of 101.3 kPa and
temperature of 160C. Calculate the room temperature after 10 minutes. (ME board
problem April 1984)

Solution

p1=101.3 kPa T1=16 +273 =289 K

Btu cal kcal


Cv=0.1714 =0.1714 0
=0.1714
lb.F 0
g .C kg .K 0

Q (=
50 persons )(150kcal / person hour ) 7500kcal / hr

Volume of room = (12) (10) (3) =360 m3

Volume of air, V = 360-(0.2) (50) =350 m3

Mass of air,=
m
p1V
=
(101.3)( 350=
) 427.34 kg
RT1 ( 0.28708)( 289 )

kcal
[10 min ]
1hr
Q =
7500
h 60 min
1250 kcal

Q=mcv (T2 T1 )

=1250 ( 427.34 )( 0.1714 )(T2 289 )

T2 = 306.1 K T2 = 33.10 C

Page | 89
4. A 1-hp stirring motor is applied to a tank containing 22.7 kg of water. The stirring
action is applied for 1 hour and the tank loses 850 kJ/h of heat. Calculate the rise
in temperature of the tank after 1 hour, assuming that the process occurs at
constant volume and that cv for water is 4.187 kJ/kg-C.

Solution

Q= (-850 kJ/hr) (1hr) = -850 kJ

W= (-1 hp)(1 hr)= (-1hp)(0.746 kW/hp)(h)(3600 sec/hr)

= -2685.6 kJ

Q= U+W

U=Q W=-850 (-2685.6)=1835.6 kJ

U= mcv( T)

U 1835.6 kJ
= T= = 19.3C
mcv ( 22.7 kg ) ( 4.187 kJ/kg.C0 )

5. A closed constant volume system receives 10.5 kJ of paddle work. The system
contains oxygen at 344 kPa, 278 K, and occupies 0.06 cu m. Find the heat (gain or
loss ) if the final temperature is 400 K. (EE board problem April 19, 1988)

Page | 90
Solution

cv=0.6595 kJ/kg(K) R= 259. 90 J/kg- K p1=344 kPa

V= 0.06 m3 T1=278 k T2= 400 k

=
m
p1V
=
( 344 )( 0.06=
) 0.2857 kg
RT1 ( 0.2599 )( 278)


=U mcv (T2 T1 ) =(0.2857)(0.6595)(400-278)

=22.99 kJ

Q=U + W

=22.99 + (-10.5)

=12.49 kJ

6) Air in the amount of 1 lbm is contained in a well insulated, rigid vessel equipped with
a stirring paddle wheel. The initial state of this air is 30psia and 40F. How much work,
in BTU, must be transferred to the air with the paddle wheel to raise the air pressure to 50
psia? Also, what is the final temperature of air?

Solution:

Page | 91
m= 1 lbm p1= 30 psia T1= 40 + 460= 500 R p2= 50 psia

p1 p2 p2T1 50 psia ( 500 R )


= ; =
T2 = = 833.33R
T1 T2 p1 30 psia

Since the vessel is well insulated Q=0

= Wn + U
Q

Wn = U = mcv (T2 T1 )

Btu
Wn =
1 lb m 0.1714 ( 833.33R 500 R ) =
57.33Btu work done on
lb R

7) A well sealed room contains 60 kg of air at 200 kpa and 250C. Now solar energy
enters the room at an average rate of 0.8 kJ/sec while a 120 Watt fan is turned on to
circulate the air in the room. If heat transfer through the walls is negligible, what will be
temperature of the air in 30 minutes?

Solution:

m= 60 kg p1= 200 kpa T1= 25+273 = 2980C


Q = 0.8kJ / sec W n = 120 watts t=30min
Since process is irreversible

Q= Wn + U

( 30 min )
kJ 60sec
= =
Q 0.8 1440kJ
sec min

J 1kJ 60sec
Wn =
120 ( 30 min ) =216kJ
sec 1000 J min

Page | 92
kJ
1440kJ =216kJ + 60kg 0.7186 (T2 298 K )
0

kg K
= =
T2 336.4 0 0
K 63.4 C

8) A closed rigid container has a volume of 1 m3 and holds air 344.8 kPa and 2730K.
Heat is added until the temperature is 6000K. Determine a) final pressure b) H c) U.

Solution:

= 1033.92

V1= 1 m3 p1=344.8 kPa T1= 2730K T2= 6000K

T2 600
=
(a) p2 p=
1 344.8 =
757.8kpa
1
T 273

(b) Q mcv (T2 T1 )


=

p1V1 344.8 (1)


=
m = = 4.4kg
RT1 0.28708 ( 273)

H mc p (T2 =
= T1 ) 4.4 (1.062 )( 600 273
= ) 1447.55kJ

(c)
=U mcv (T2 =
T1 ) 4.4 ( 0.7186 )( 600 273)

= 1033.92 kJ

Page | 93
Isobaric Process

An Isobaric process is an internally reversible process of a substance during


which the pressure remains constant.

(a) Relation between V and T

T2 V2
=
T1 V1

(b) Nonflow work

2
=
Wn 1
= p (V2 V1 )
pdV

Wn = mR (T2 T1 ) = ( k 1)( U )

(c) The change of internal energy

=
U mcv (T2 T1 )

(d) The heat transferred.

=Q mc p (T2 T1 )

(e) The change of enthalpy


=H mc p (T2 T1 )

Page | 94
(f) The change of entropy

T2
S =mc p ln
T1
(g) Steady flow isobaric

(a)
Q = PE + KE + H + Ws

Ws = (KE + PE )

Ws = KE , if PE =
0

(b)
2
Vdp
= Ws + E
1

=
0 Ws + KE

Ws = KE

Problems

1. A certain gas, with cp=0.529 Btu/lb.R0 and R= 96.2 ft.lb/lb.0R, expands from 5 cu
ft and 800F to 15 cu ft while the pressure remains constant at 15.5 psia. Compute
(a) T2, (b) H, (c) U (d) S. (e) For an internally reversible nonflow process,
what is work?

Solution

p= 15.5 psia V1= 5 cu ft V2= 15 cu ft T1= 80 + 460 =5400R

(a)=
T2
TV
=1 2 ( 540 )(15
=
) 16200 R
V1 5

Page | 95
lb in 2
( 5 ft )
3
15.5 144
p1V1 in 2 ft 2
(b)=
m = = 0.2148 lb
ft lb
( 540 R )
RT1 0
96.2
lb R


=H mc p (T2 T1 )

= (0.2148) (0.529) (1620-540)

= 122.7 Btu

96.2 Btu
(c) cv = c p R = 0.529 = 0.4053
778 lb R

=
U mcv (T2 T1 )

= (0.2148) (0.4053) (1620-540)

= 94 Btu

T2
(d) S =mc p ln
T1
1620
= (0.2148) (0.529) ln
540
Btu
=0.1249 0
R

(e)=
Wn p (V2 V1 )

lb in 2
15.5 2 144 2 (15 ft 5 ft )
3 3

in ft
=
ft lb
778
Btu

= 28.7 Btu

2. A perfect gas has a value of R =319.2 J/kg-K and k = 1.26. If 120 kJ are added to
2.27 kg of this gas at constant pressure when the initial temperature is 32.20C,
find (a) T2, (b) H , (c) U, and (d) work for a nonflow process.

Page | 96
Solution

k= 1.26 m= 2.27 kg R= 319.2 J/kg-K

Q=120 kJ T1= 32.2 +273 = 305.2 K

(a)=
cp
kR
=
(1.26 )( 0.3192
=
) 1.5469
kJ
k 1 1.26 1 kg K

=Q mc p (T2 T1 )

120= (2.27) (1.5469) (T2-305.2)

T2=339.40 K

(b)
=H mc p (T2 T1 ) =120 kJ

R 0.3192 kJ
(c)=
cv = = 1.2277
k 1 1.26 1 kg K

=
U mcv (T2 T1 )

= (2.27) (1.2277) (339.4 305.2)

= 95.3 kJ

(d)=
Wn mR(T2 T1 )

= (2.27) (0.3192) (339.4 -305.2)

= 24.78 kJ

Page | 97
3) A mass of 15 kg of air in a piston- cylinder device is heated from 250C to 770C
by passing current through a resistance heater inside the cylinder. The pressure
inside the cylinder is held constant at 300 kPa during the process, and a heat loss
of 60 kJ occurs. a) Determine the electric energy supplied in kWh. b) How high
will the piston rise if the bore is 120mm diameter?

Solution:

m= 15 kg T1= 25 +273= 2980K T2= 77 +273= 3500K

p= 300 kPa Qloss= 60 kJ

Energy Balance:

We = H + Qloss

We mc p (T2 T1 ) + Qloss
=

kJ
= ( 350 K 298 K ) + 60kJ
0 0
We 15kg 1.0062
kg K

1hr
= =
We 844.84 kJ 0.235kWh
3600sec

4.) A piston-cylinder device, whose piston is resting on a set of stops, initially


contains 3kg of air at 200 kpa and 270C. The mass of the piston is such that a
pressure of 400 kPa is required to move it. Heat is now transferred to the air until
its volume doubles. Determine the work done by the air and the total heat
transferred to the air during this process?

Page | 98
Solution:

m=3kg p1= 200kPa T1= 27+ 273 =3000K

p2= 400kPa V3= 2V2

Process 1-2 is constant volume:

T1 P2 300 K ( 400kPa )
0

=
T2 = = 6000 K
P1 200kPa

kJ
Q1= mcv (T2 T=
1) ( 600 K 300 K=) 646.74kJ
0 0
2 3kg 0.7186
kg K

W1 2 = 0

Process 2-3 is constant pressure;

T2 2 ( 6000 K )
V3T2 2V2T2
=
T3 = = 2= = 12000 K
V2 V2

Q2=
3 mc p (T3 T=
2) ( 3kg )(1.0062 ) (12000 K 6000 K=) 1811.16kJ

W=
n mR(T2 T=
1) ( 3)( 0.28708)(1200 600=) 516.44kJ

Qtotal = Q1 2 + Q23 = 646.74kJ + 1811.16kJ = 2457.9kJ

Wtotal = Wn1 2 + Wn 23 = 0 + 516.44 = 516.44kJ

5.) Consider that 1 lb of air has a decrease of internal energy of 20.58 Btu while
its Fahrenheit temperature is reduced to one third of the initial temperature during
a reversible nonflow constant pressure process. Determine (a) the initial and final
temperature (b) Q (c) W and (d) S

Solution:

Page | 99
U= -20.58 Btu t1= 3t2

a) U= mcv(T2 T1)

=
20.58 (1lb )( 0.1714 )( t2 t1 )

=
20.58 (1lb )( 0.1714 )( t2 3t2 )
t2= 600F t1= 1800F

b) Q= mcp(T2 T1)

=Q (1)( 0.24 )( 60=


180 ) 28.8 Btu

c) Wn= mR(T2 T1)

ft lb
(1) 53.34 ( 520 R 640 R )
0 0

lb R
Wn = = 8.22 Btu
ft lb
778
Btu

T2 520 Btu
d) S =mc p ln =(1)( 0.24 ) ln =0.05
T1 640 R

Page | 100
Isothermal Process

An isothermal process is an internally reversible constant temperature process of a


substance.

(a) Relation between p and V

p1V1 = p2V2

(b) Nonflow work

2 2 CdV V V
=
Wn =
1
pdV =
V 1
C=
ln 2 p1V1 ln 2
V1 V1

V2
Wn = mR (T ) ln
V1

(c) The change of internal energy

U =
0

(d) The heat transferred

V2 p
Q=U + Wn =p1V1 ln =mRT ln 2
V1 p1

(e) The change of enthalpy.

H=0

(f) The change of entropy

Q p
S = = mR ln 1
T p2

Page | 101
(g) Steady flow isothermal.

(a) Q = PE + KE + H + Ws

Ws= Q PE KE

Ws = Q , if PE =0, KE =0

(b)
2
Vdp= Ws + KE
1

pdV
From pV=C, pdV + Vdp=0, dp =
V

2 2 pdV 2
Vdp =
V
= pdV
1 1
V 1

V2
p1V1 ln = Ws + KE
V1

W=
n Ws , if KE= 0

Problems

1. During an isothermal process at 880F, the pressure on 8 lb of air drops from 80


psia to 5 psig. For an internally reversible process, determine (a) the pdV and
the work of a nonflow process, (b) the - Vdp and the work of a steady flow
process during which KE=0, (c) Q, (d) U, and H, and (e) S.

Solution

Page | 102
T= 88 + 460 = 548 0R m= 8 lb p1= 80 psia

p2=5 + 14.7 = 19.7 psia

V p
pdV p1=
(a)= V1 ln 2 mRT ln 2
V1 p1

=
(8)( 53.34 )( 548) ln 80
= 421.2 Btu
778 19.7

=
Wn =
pdV 421.2 Btu

V
- Vdp p1=
(b)= V1 ln 2 421.2 Btu
V1

(c) Q =U + Wn =421.2 Btu

(d) U= 0

H= 0

Q 421.2 Btu
(e) S=
= = 0.7686
T 548 R

2. During a reversible process, there are abstracted 317 kJ/s from 1.134kg/s of a
certain gas while the temperature remains constant at 26.7 0C. For this gas,
cp=2.232 and cv=1.713 kJ/ kg-K The initial pressure is 586 kPa. For both nonflow
and steady flow ( PE=0, KE=0) process, determine (a) V1, V2, and p2, (b) the
work and (c) S and H.

Solution

Page | 103

Q= 317 kJ / s T= 26.7 + 273 = 299.70 K

m = 1.134 kg / s p1= 586 kPa
(a) R = c p c p = 2.232 1.713 = 0.519 kJ/kg.K

kN m

(1.134kg ) 0.519
kg K
( 299.7o K )
=
V1
m RT1
= = 0.301 m3 / s
p1 kN
586 2
m

V2
Q = p1 V 1 ln
V1

kJ

317
V2 Q sec
ln
=
= = 1.80
kN
586 2 ( 0.301m )
3
V1 p1 V 1
m


V2 1.80
=

e= 0.1653
V1


= = 0.0498 m3 / sec
V 2 (0.1653)(0.301)

=
p1 V 1
p2 =
( 586 )( 0.301
=
) 3542 kPa

V2 0.0498


(b) since PE=0 and KE=0 , W n = W s = Q = 317 kJ/sec

kJ

317
Q sec =1.058 kJ
(c) S = =
T 299.7 0 K K sec

H= 0

3. Air flows steadily through an engine at constant temperature, 400 K. Find the
work per kilogram if the exit pressure is one third the inlet pressure and the inlet
pressure is 207 kPa. Assume that the kinetic and potential energy variation is
negligible. (EE board problem April 1988)

Page | 104
Solution

T= 400 K R= 287.08 kJ/ (kg) (K) p1=207 kPa


p1
=3
p2

=
v1
RT1
=
( 0.28708)( 400
=
) 0.5447 m3 / kg
p1 207

v2 p
=w p=
1v1 ln p1v1 ln 1
v1 p2

= (207) (0.5547) ln 3

= 126.1 kJ/kg

4. A closed system containing 2 kg of air undergoes an isothermal process from


600kPa and 2000C to 80kPa. Determine the initial volume of this system, the
work done and the heat transfer during this process.

Solution:

m= 2kg p1= 600 kPa T1= 200+273= 4730K

p2= 80 kPa

Page | 105
kJ
2kg 0.28708
kg K
( 4730 K )
a)=
V1 =
mRT
p1 kN
600 2
m
V1 = 0.453 m 3

kJ
ln 2 2kg 0.28708 ( 4730 K ) ln
p 600
= =
b) Wn mRT
p1 kg 80

Wn = 547.2 kJ

Q= 547.2 kJ

5) A piston-cylinder device contains 5kg of air at 400 kPa and 300C. During the quasi-
equilibrium isothermal expansion, 15 kJ of boundary work is done by the system, and 3kJ
of paddle wheel work is done on the system. Find the a) initial volume and the final
volume, b) heat transfer during this process.

Solution:

m= 5 kg p1 = 400 kPa T= 30 +273= 3030K

Wn= 15 kJ Wp= -3 kJ

a)
kJ
( 303 K )
0
5kg 0.28707
mRT kg K
=
V1 = = 1.087m3
p1 kn
400 2
m
V
Wn = mRT ln 2
V1
kJ
( 300 K ) ln
V2
15 = 5kg 0.28708 0

kg K 1.087

V2 = 1.12m3

Page | 106
b) Since the system is in isothermal expansion U= 0

=
Q Wn + U + Wp

Q 15kJ + 0 + ( 3kJ=
= ) 12kJ

Page | 107
Isentropic Process

An isentropic process is a reversible adiabatic process. Adiabatic simply means no


heat. A reversible adiabatic is one of constant entropy.

1. Relation among p, V, and T

(a) Relation between p and V

p=
1V1
k
p=
2V2
k
C

(b) Relation between T and V

p1V1 p2V2
From p1V1k = p2V2k and = , we have
T1 T2
k
p2 V1 p2 V1T2
= = and then
p1 V2 p1 V2T1

k
V1T2 V1
=
V2T1 V2

k 1
T2 V1
=
T1 V2

(c) Relation between T and p.

1
V2 p1 k V2 p1T2
= = and
V1 p2 V1 p2T1

1
p1T2 p1 k
=
p2T1 p2

Page | 108
1 1 k
1
T2 p1 k T p k
= so 2 = 1
T1 p2 T1 p2

k 1
T2 p2 k
=
T1 p1

2. Non-flow work

From pV k = C , p = CV k

2 2 2
=
pdV dV C V k dV
k
=
Wn CV=
1 1 1

Integrating and simplifying

p2V2 p1V mR (T2 T1 )


=Wn =
1 k 1 k

mR(T2 T1 ) R
Wn =
but cv =
k 1 k 1

Wn = mcv (T2 T1 ) = U

3. The change of internal energy

=
U mcv (T2 T1 )

4. The heat transferred

Q=0

5. The change of enthalpy


=H mc p (T2 T1 )

6. The change of entropy

S=0

Page | 109
7. Steady flow isentropic
(a)
Q = PE + KE + H + Ws

Ws = PE KE H

Ws = , if PE = 0 and KE = 0

2
(b) Vdp= Ws + KE
1
1 1
Let C = p k V or V = Cp k

1
2
Vdp =
Cp k dp
1

Integrating and simplifying,

2 k ( p2V2 p1V1 ) 2
Vdp
=
1
= k pdV
1 k 1

2
Vdp = k U = H
1

Problems

1. From a state defined by 300 psia, 100 cu ft. and 240 0F, helium undergoes and
isentropic process to 0.3 psig,. find (a) V2 and T2, (b) U and H, (c) pdV , (d)
Vdp , (e) Q and S. What is the work (f) if the process is non-flow, (g) if the
process is steady flow with KE= 10 Btu?

Solution

p1= 300 psia p2= 0.3 + 14.7 =15 psia V1= 100 cu ft

Page | 110
T1= 240 +460= 7000R

(a)
1 1
p1 k 300 1.666
= 2 V=
V 1 100 = 603.4 ft 3
2
p 15

1.6661
k 1
p2
1.666
15 k
=T2 T=
7001 = 211.3 0 R
1
p 300

T2= -248.70F

(b)
lb in 2
2 (
300 2
144 100 ft 2 )
p1v1 in ft
=
m = = 15.99 lb

RT1 700
ft
lb R
lb
( 386.04 0
R )
H =mc p (T2 T1 ) =(15.99)(1.241)(211.3 700) =9698 Btu

U =mcv (T2 T1 ) =(15.99)(0.745)(211.3 700) =5822 Btu

p2V2 p1V1 (144) (15 x603.4 300 x100 )


(c) pdV
= = = 5822 Btu
1 k ( 778)(1 1.666 )

(d) Vdp= k pdV= (1.666 )( 5822 )= 9698 Btu

(e) Q=0

S=0

(f)
Q=U + Wn

Wn = U = ( 5822 ) = 5822 Btu


(g)
Vdp= Ws + KE

= Ws + 10
9698
Ws = 9688 Btu

Page | 111
2. An adiabatic expansion of air occurs through a nozzle from 828 kPa and 710C to 138
kPa. The initial kinetic energy is negligible. For an isentropic expansion, compute the
specific volume, temperature and speed at the exit section.

Solution

p1= 828 kPa T1= 71 + 273 =344 K p2= 138 kPa

k 1 1.4 1
p2 k 138 1.4
=T2 T=
3441 = 2060 K
1
p 828

T2= -670C

kJ
0.28708 kg R ( 344 R )
0

RT1
=
v1 = = 0.1193 m3 / kg
p1 kN
828 3
m
1 1
p1 k 828 1.4
=v2 v=
1 0.1193 = 0.429 m3 / kg
2
p 138
h =c p (T2 T1 ) =(1.0062)(206 344) =138.9 kJ/kg

q = pe + ke + h + ws

ke = h = 138,900 J/kg

22
ke = ke2 ke1 =
2k

kg m N m
2=
2
( 2k )( K =) 2 1 2
138,900 = 2
277,800 m / sec
2

N s kg

2 = 527.1 m/ sec

Page | 112
3) During an isentropic process of 4 lb/sec of air, the temperature increases from 400F

to 3400F. Compute (a) U (b) H (c) Q (d) S (e) pdV (f) Vdp (g)the non flow
Btu
work (h)the steady flow work where KE =
10 , PE = 0 (i) For an irreversible
sec
adiabatic process from the same initial state to the same final pressure, the final
temperature is 4000F. Find the works, for nonflow and steady flow processes

( KE =
10 Btu / sec , PE =
0 ).

Solution:


m = 4lb / sec T1= 40+460=5000R T2= 340+ 460= 8000R


a)
=U mcv (T2 T1 )


lb Btu Btu
=U 4 =
0.1714 ( 800 500 ) 205.68
sec lb R sec


H mc p (T2 T1 )
b) =

lb Btu Btu
=H 4 =
0.24 ( 800 500 ) 288.68
sec lb R sec

c) Q = 0


d) S =0

m R (T2 T1 ) Btu
e) pdv = = U = 205.68
1 k sec

m Rk (T2 T1 ) Btu
f) Vdp = = H = 288.68
1 k sec

Btu
g) Wn = U = 205.68
sec

Page | 113

h) Ws = PE H KE

Btu
W s = ( 288.68 ) ( 10 ) =278.68
sec

i) At the same final pressure and same initial conditions,

T2 ' = 400 + 460 = 8600 R


k k'
p2 T2 k 1 T2 ' k '1
= =
p1 T1 T1

1.4 k'
800 1.41 860 k '1
=
500 500

k=1.492

ft lbs
53.34
=
cv =
R lb R = 0.1393 Btu
k ' 1 ft lbs
(1.492 1) 778
lb
Btu

Btu
W n = U = ( 4 )( 0.1393)( 860 500 ) = 200.66
sec

Ws = PE H KE

k 'R 1.492 ( 53.34 ) Btu


=
cp = = 0.208
k ' 1 (1.492 1)( 778) lb R


lb Btu Btu
= H 4 =
0.208 ( 860 500 ) 299.52
sec lb R sec

Btu
W s =299.52 ( 10 ) =289.52
sec

Page | 114
Polytropic Process

A polytropic process is an internally reversible process during which

pV n = C And p=
1V1
n
p=
2V2
n
piVi n

Where n is any constant

1. Relation among p, V and T

(a) Relation between p and V

p1V1n = p2V2n

(b) Relation between T and V

n 1
T2 V1
=
T1 V2

(c) Relation between T and p

n 1
T2 p2 n
=
T1 p1

2. Nonflow work

2 p2V2 p1V1 mR (T2 T1 )


=
Wn =
pdV
1
=
1 n 1 n

k 1
Wn =
U
n 1

Page | 115
3. The change of internal energy

=
U mcv (T2 T1 )

4. The heat transferred

Q=U + Wn
mR(T2 T1 )
= mcv (T2 T1 ) +
1 n

c ncv + R
=m v (T2 T1 )
1 n

c ncv
= m p (T2 T1 )
1 n

k n
= mcv (T2 T1 )
1 n

=Q mcn (T2 T1 )

k n
cn = cv , the polytropic specific heat
1 n

5. The change of enthalpy


=H mc p (T2 T1 )

6. The change of entropy

T2
S =mcn ln
T1
7. Steady flow polytropic

(a)
Q = PE + KE + H + Ws

Ws= Q PE KE H

Ws= Q H if (PE =0, KE =0)

Page | 116
2
(b) Vdp= Ws + KE
1

2 n ( p2V2 p1V1 ) 2
Vdp
=
1
= n pdV
1 n 1

Problems

1. During a polytropic process, 10 lb of an ideal gas, whose R = 40 ft.lb/lb R and


cp=025 Btu/lb. R, changes state from 20 psia and 40 0F to 120 psia and 340 0F.
Determine (a) n, (b) U and H, (c) S, (d) Q, (e) pdV , (f) Vdp . (g) If the
process is steady flow during which KE=0, what is Ws? What is KE if Ws=0?
(h) What is the work for a Nonflow process?

Solution

ft.lb
p1= 20 psia m= 10 lb p2= 120 psia R= 40
lb.0 R
T1= 40 + 460 = 5000R T2= 340 + 460 = 800 0R

Btu
cp= 0.25
lb.R 0

(a)
n 1
T2 p2 n
=
T1 p1

n 1
800 120 n
=
500 20

=
U mcv (T2 T1 )

n 1
ln 6 = ln1.6
n

Page | 117
n 1 0.4700
= ; n = 1.356
n 1.7918

(b)
40 Btu
cv = c p R = 0.25 = 0.1986
778 lb.R 0

= (10) (0.1986) (800-500)

=595.8 Btu


=H mc p (T2 T1 )

= (10) (0.25) (800-500)

= 750 Btu

cp 0.25
(c) =
k = = 1.259
cv 0.1986

k n 1.259 1.356 Btu


cn = cv = 0.1986 = 0.0541
1 n 1 1.356 lb.R 0

T2 800 Btu
S =mcn ln = (10 )( 0.0541) ln = 0.2543 0
T1 500 R

=
(d) Q mcn (T2 T1 )

= (10) (0.0541) (800- 500)

=162.3 Btu

(e) pdV
=
mR (T2 T1 )
=
(10 )( 40 )( 800 500 )
1 n ( 778)(1 1.356 )
= -433.3 Btu

(f) Vdp =
n pdV =
(1.3560)(433.3) =
587.6 Btu

(g)
If KE=0

Page | 118
Vdp =
Ws = 587.6 Btu

If Ws= 0

Vdp =
KE = 587.6 Btu

(h) Wn = pdV = 433.3 Btu

2. Compress 4 kg/ s of CO2 gas polytropically (pV1.2 =C) from p1=103.4 kPa, T1= 600C
to T2= 2270C. Assuming the ideal gas action, find p2, W, Q, S (a) as nonflow, (b) as
a steady flow where P=0, K=0

Solution

kg
p1=103.4 kPa m= 4
sec
T1= 60 + 273 = 333 K T2= 227 +273 = 500 K

(a) nonflow

n 1.2
T2 n 1
(103.4 ) =
500 1.21
=p2 p=
1 1184.9 kPa
T1 333

=
m R(T2 T1 )
Wn =
( 4 )( 0.18896 )( 500 333)
1 n 1 1.2

kJ
= -631.13
sec

k n 1.288 1.2 kJ
cn = cv = ( 0.6561) = -0.2887
1 n 1 1.2 kg .K

Page | 119

=Q m cn (T2 T1 )

kJ
= -193.8
sec

T2 500
S =m cn ln = ( 4 )( 0.2887 ) ln
T1 333

kJ
S =0.4694
sec K

(b) Steady flow

p2= 1184.9 kPa

kJ
Q= 193.8
sec

=
H m c p (T2 T1 )
kJ
= (4) (0.8452) (500 333) = 563.6
sec


Q = P E + K E + H + W s

kJ
W s = Q H = 193.8 563.6 = 757.4
sec

3. A piston cylinder device initially contains 0.07 m3 of nitrogen gas at 130 kpa and
1200C. The nitrogen is now expanded polytropically to a state of 100 kpa and 1000C.
Determine a) n b) final volume c) Wn d) Ws if PE=0, KE=0. e) heat.
Solution

Page | 120
For Nitrogen:

kJ J
cv= 0.7442 k=1.399 R=296.86
kg K kg K

V1= 0.07m3 p1= 130 kPa T1= 120+273=3930K

p2=100kPa T2= 100 + 273= 3730K

n 1
T2 p2 n
a) =
T1 p1

n 1
3730 K 100 n
=
3930 K 130

n= 1.401

n
p1 V2
b) =
p2 V1

1.401
130 V2
=
100 0.07

V2= 0.0797m3

130 2 ( 0.07m3 )
kN
p1V1 m
c) =
m = = 0.078kg
RT1 0.29686 kN m 3930 K
kg K
( )

kJ
mR (T2 T1 )
0.078kg 0.29686
kg K ( 3730 K 3930 K )
=Wn =
1 n 1 1.401

Wn= 1.155 kJ

kJ
( 0.078kg ) 3.7 x103
d) Q = ( 373 K 393 K ) =
5.79 J
0 0

kg K

Page | 121
nmR (T2 T1 )
=
WS = nW = 1.401(1.155kJ
= ) 1.618kJ
1 n
n

e) Q mcn (T2 T1 )
=

k n kJ 1.399 1.401 3 kJ
=cn c=
v 0.7442 = 3.71x10
1 n kg K 1 1.401 kg K

kJ
( 0.078kg ) 3.7 x103
Q= ( 373 K 393 K ) =
5.79 J
0 0

kg K

4. A frictionless piston-cylinder device contains 2 kg of nitrogen at 100 kPa and 3000K.


Nitrogen is now compressed slowly according to the relation pV1.4=constant until it
reaches a final temperature of 360 0K. Calculate the work input during this process.

Solution:

m= 2 kg p1=100 kPa T1=3000K

n=1.4 T2= 3600K

kJ
2kg 0.29686 ( 360 300 )
mR (T2 T1 ) kg K
Wn = = = 89.058kJ
1 n 1 1.4

Page | 122
Curves for different Values of n

Polytropic processes are all inclusive in that many of the prior equations can be
obtained by choosing proper values of n.

Let n=0; then pV0=C, or p=C, an isobaric process.


Let n= ; then, from pVn=C, we have
1 1
p nV= p V= V= C , an isometric process.
Let n=k; then pVk=C, an isentropic process
Let n=1; then pV=C, an isothermal process

The isentropic curve on the pV plane is steeper than the isothermal curve on the
TS plane and the constant volume curve is steeper than the constant pressure curve when
both are drawn between the same temperature limits.

Page | 123
Process Isometric Isobaric Isothermal Isentropic Polytropic
V=C p=C T=C S=C pV n = C
p,V,T relations T2 P2 T2 V2 p1V1 = p2V2 p1V1k = p2V2k p1V1n = p2V2n
= =
T1 P1 T1 V1 k 1 n 1
k 1 n 1
T2 V1 p2 k
T2 V1 p2 n
= = = =
T1 V2 p1 T1 V2 p1
2 0 p (V2 V1 ) V2 p2V2 p1V1 p2V2 p1V1
1
pdV p1V1 ln
V1 1 k 1 n
Vdp
2
V ( p1 p2 ) 0 V k ( p2V2 p1V )1 n ( p2V2 p1V )1
1
p1V1 ln 2
V1 1 k 1 n
U 2 U1 m cv dT m cv dT 0 m cv dT m cv dT
mcv (T2 T1 ) mcv (T2 T1 ) mcv (T2 T1 ) mcv (T2 T1 )

Q m cv dT m c p dT m Tds 0 m cn dT
mcv (T2 T1 ) mc p (T2 T1 ) p1V1 ln
V2 mcn (T2 T1 )
V1
N 0 1 K to +
Specific heat cv cp 0 k n
C cn = cv
1 n
[k = C ]
H 2 H1 m c p dT m c p dT 0 m c p dT m c p dT
mc p (T2 T1 ) mc p (T2 T1 ) mc p (T2 T1 ) mc p (T2 T1 )
S 2 S1 cv dT c p dT Q 0 cn dT
m m m
T T T T
T T2 V2 T
mcv ln 2 mc p ln mR ln mcn ln 2
T1 T1 V1 T1

Page | 124
Review Problems

1. A perfect gas has a value of R= 58.8 ft.lb /lb-R and k=1.26. If 20 Btu are added to
5 lb of this gas at constant volume when the initial temperature is 900F, find (a)
T2, (b) H, (c) s, (d) U, and (e) work for a nonflow process.
Ans. (a) 563.80R; (b) 25.27 Btu; (c) 0.036 Btu/0R; (d) 20.06 Btu

2. A reversible, nonflow, constant volume process decreases the internal energy by


316.5 kJ for 2.268 kg of a gas for which R= 430 J/kg-K and k=1.35. For the
process, determine (a) work, (b) Q, and (c) S. The initial temperature is 204.4
0
C.
Ans. (a) 0 ;( b) -316.5 kJ; (c) -0.7572kJ/K

3. A 10-ft3 vessel of hydrogen at a pressure of 305 psia is vigorously stirred by


paddles until the pressure becomes 400 psia. Determine (a) U and (b) W. No
heat is transferred, cv=2.434 Btu/lb-R
Ans. (a) 434 Btu; (b) -434 Btu

4. A gas whose composition is not known has 42.2 kJ of paddle work input at
constant volume of 566 liters. Initially, p1=138 kPaa, t1=26.70C; finally
t2=82.20C. What are U and Q if k=1.21?
Ans. 68.9kJ, 26.7 kJ

5. Three pounds of a perfect gas with R=38 ft-lb/lb-R and k=1.667 have 300 Btu of
heat added during a reversible nonflow constant pressure change of state. The
initial temperature is 1000F. Determine the (a) final temperature, (b) H, (c) W,
(d) U, (e) S.
Ans. (a) 9190F; (c) 120 Btu; (d) 180 Btu; (e) 0.3301 Btu/0R

6. While the pressure remains constant at 689.5 kPa the volume of a system of air
changes from 0.567 m3 to 0.283 m3. What are (a) U, (b) H, (c) Q, (d) S? (e)
If the process is nonflow and internally reversible, what is the work?
Ans. (a) -490.2 kJ; (b) -686.3 kJ; (c) -686.3 kJ; (d) -0.6974 kJ/kg-K; (e)
-195.8 kJ
7. Oxygen at the rate of 3 lb/min undergoes a reversible adiabatic process during
which its entropy changes -0.35 Btu/lb-0R; V1= 17.75ft3 and t1=4000F. For both
nonflow and steady flow(PE=0, KE=0) process, compute (a) U and H (b) W
(c) Q
Ans. (a) -324, -451.6 Btu/ min (b) -127.6 Btu/min, (c)-451.6 Btu/min

8. A piston-cylinder device, with a set of stops on the top initially contains 3 kg of


air and 270C. Heat is now transferred through the air, and the piston rises until it
hits the stops, at which the volume is twice the initial volume. More heat was
added until the pressure inside the cylinder also doubles. Determine the work
done and the amount of heat transfer for this process.
Ans (a) 258.37 kJ, 2199 kJ

Page | 125
9. Four pounds of air gain 0.491 Btu/0R of entropy during a nonflow isothermal
process. If p1= 120 psia and V2= 42.5 ft3, find (a) V1 and T1, (b) W, (c) Q, and (d)
U.
Ans. (a) 7.093 ft3, 574.50R; (b) 282.1 Btu ;(c) 282.1 Btu; (d) 0

10. If 10 kg/min of air are compressed isothermally from p1= 96 kPa and
V1=7.65m3/min to p2= 620 kPa, find the work, the change of entropy and the heat
for (a) nonflow process and (b) a steady flow process with 1= 15 m/s and 2=
60 m/s.
Ans. (a) -1370 kJ/min, -5.356 kJ/min; (b) -1386.9 kJ/min

11. One pound of an ideal gas undergoes an isentropic process from 95.3 psig and a
volume of 0.6 ft3 to a final volume of 3.6 ft3. If cp= 0.124 and cv=0.093 Btu/lb-R,
what are (a) T2, (b) p2, (c) H and (d) W.
Ans. (a) -243.1 0F; (b) 10.09 psia; (c) -21.96 Btu; (d) 16.48 Btu

12. Hydrogen is compressed isentropically from p1=108 psia, V1=3 ft3/sec, and
t1=400F to p2=256psia. For both nonflow and for a steady flow (KE=0, PE=0)
processes, find (a) t2 and V2 (b) pdv and vdp (c) H, U, and S, and (d) W
and Q.
Ans. (a) 1810F,162ft3/sec (b)-41.9, -58.8 (c) 58.8, 41.9 (d) 41.9, 0

13. A certain ideal gas whose R= 278.6 J/kg.K and cp=1.015 kJ/kg.K expands
isentropically from 1517 kPa, 2880C to 965 kPa. For 454 g/s of this gas
determine, (a) Wn, (b) V2, (c) U and (d) H.
Ans. (a) 21.9 kJ/s; (b) 0.06495 m3/s; (d) -30.18 kJ/s

14. A polytropic process of air from 150 psia, 3000F, and 1 ft3 occurs to p2=20 psia in
accordance with pV 1.3 = C . Determine (a) T2 and V2, (b) U, H, and S, (c)
pdV and Vdp . (d) Compute the heat from the polytropic specific heat and
check by the equation Q =U + pdV . (e) Find the nonflow work and (f) the
steady flow work for K=0.
Ans. (a) 17.40F, 4.711ft3; (b) -25.81 Btu, -36.14 Btu, 0.0141 Btu/0R; (c)
34.41 Btu, 44.73 Btu; (d) 8.60 Btu; (e) 34.41 Btu; (f) 44.73 Btu

15. The work required to compress a gas reversibly according to pV 1.3 = C is


67,790 J, if there is no flow. Determine U and Q if the gas is (a) air, (b)
methane. For methane k= 1.321, R= 518.45 J/kg.K, cv= 1.6187, cp= 2.1377
kJ/kg.K
Ans. (a) 50.91 kJ, -16.88 kJ; (b) 63.50 kJ, -4.29 kJ

Page | 126
16. Consider a gas mixture whose apparent molecular weight is 33, initially at 3 bar
and 3000K, and occupying a volume of 0.1 m3. The gas undergoes an expansion
during which pV1.3= constant and the energy transfer by heat to the gas is 3.84 kJ.
Neglecting kinetic and potential energy effects and cv= 0.675 kJ/kg-0K, determine
(a) the final temperature (b) final pressure (c) final volume (d) work for both non
flow and steady flow work.
Ans(a) 226.670K (b) 89.2 kPa (c) 0.254 m3 (d) 24.39kJ, -31.71kJ

17. During a polytropic process 10 lb/sec of oxygen expand reversibly from 15000R
to 10000R, the expansion ratio being re=V2/V1=4. For both nonflow and steady
flow (PE=0, KE=0) processes, compute (a) the exponent n, (b) U, H, and
S, (c) pdv and vdp , (d) W and Q.
Ans(a) n=1.292 (b) -786.9 Btu/sec, -1097 Btu/sec, 0.1158Btu/sec-R
(d)1063.93Btu/sec, -1379.6Btu/sec (d) Q=277.43Btu/sec

18. The power stroke in an internal combustion engine can be approximated with a
polytropic expansion. Consider air in a cylinder volume of 0.2 liters at 7 Mpa,
18000K. It now expands in a reversible polytropic process with, n=1.5 through a
volume ratio of 8:1. Calculate the work and heat transfer for the process.
Ans.

Page | 127
5 Gas Cycles
Heat or thermal engine is a closed system (no mass crosses its boundaries) that
exchanges only heat and work with its surrounding and that operates in cycles.
Elements of a thermodynamic heat engine with a fluid as the working substance:

1. A working substance, matter that receives heat, rejects heat, and does work;
2. A source of heat(also called a hot body, a heat reservoir or just source), from
which the working substance receives heat;
3. A heat sink (also called a receiver, a cold body or just sink), to which the
working substance can reject heat; and
4. An engine, wherein the working substance may do work or have work done
on it.

A thermodynamic cycle occurs when the working fluid of a system experiences a


number of processes that eventually return the fluid to its initial state.

Cycle Work and Thermal Efficiency

Qa= heat added


Qr= heat rejected
W= net work

Available energy is that part of the heat that was converted into mechanical work.
Unavailable energy is the remainder of the heat that had to be rejected into the receiver
(sink).

The Second Law of Thermodynamics

All energy received as heat by a heat-engine cycle cannot be converted into mechanical
work.

Work of a Cycle

(a) W = Q
W= Qa + (Qr ) ( algebraic sum)
W= Qa Qr ( Arithmetic difference)

Page | 128
(b) The net work of a cycle is the algebraic sum of the works done by the individual
processes.

W = W

W = W1 2 + W23 + W3 4 + .....

The Carnot Cycle

The Carnot cycle is the most efficient cycle conceivable. There are other ideal cycles as
efficient as the Carnot cycle, but none more so, such a perfect cycle forms a standard of
comparison for actual engines and actual cycles and also other less efficient ideal cycles,
permitting as to judge how much room there might be for improvement.

Operation of the Carnot engine

A cylinder C contains m mass of a


substance. The cylinder head, the only
place where heat may enter or leave the
substance (system) is placed in contact
with the source of heat or hot body which
has a constant temperature T1. Heat flows
from the hot body into the substance in the
cylinder isothermally, process 1-2, and the
piston moves from 1 to 2. Next, the
cylinder is removed from the hot body and
the insulator I is placed over the head of
the cylinder, so that no heat may be
transferred in or out. As a result, any
further process is adiabatic. The isentropic
change 2-3 now occurs and the piston
moves from 2 to 3. When the piston
reaches the end of the stroke 3, the
insulator I is removed and the cylinder
head is placed in contact with the receiver
or sink, which remains at a constant temperature TA. Heat then flows from the substance
to the sink, and the isothermal compression 3-4 occurs while the piston moves from 3-4.
Finally, the insulator I is again placed over the head and the isentropic compression 4-1
returns to its initial condition, as the piston moves from 4 to 1

Page | 129
Analysis of the Carnot cycle

=
QA T1 ( S 2 S1 ) , area 1-m-1

=
QR T3 ( S 4 S3 ) , area 3-4-m-n-3

= T3 ( S 4 S3 ) =
T3 ( S 2 S1 )

W = QA QR = T1 ( S 2 S1 ) T3 ( S 2 S1 )

= (T1 T3 )( S 2 S1 ) , area 1-2-3-4-1

W (T1 T3 )( S 2 S1 )
=
e =
QA (T1 )( S 2 S1 )

T1 T3
e=
T1

The thermal efficiency e is defined as the fraction of the heat supplied to a


thermodynamic cycle that is converted into work.

Work from the TS plane

V2
QA = mRT1 ln
V1
V V
QR = mRT3 ln 4 = mRT3 ln 3
V3 V4

Page | 130
From process 2-3

k 1
T3 V2
=
T2 V3

From process 4-1

k 1
T4 V1
=
T1 V4

But T4 = T3 and T1 = T2

k 1 k 1
V V
Therefore, 2 = 1
V3 V4

V3 V2
Then, =
V4 V1

V2
QR = mRT3 ln
V1

V2 V
W = QA QR = mRT1 ln mRT3 ln 2
V1 V1

V2
(T1 T3 )mR ln
W V1
=
e =
QA V2
mRT1 ln
V1
T1 T3
e=
T1

1
e= 1
rk k 1

Work from the pV plane.

W=
W =
W=W1 2 + W23 + W3 4 + W41

V2 p3V3 p2V2 V p V p4V4


W = p1V1 ln + + p3V3 ln 4 + 1 1
V1 1 k V3 1 k

Page | 131
Mean Effective Pressure (mep)

W W
=
mep =
Vd Vmax Vmin

Vd= displacement volume, the volume swept by the piston in one stroke.
Vmax= maximum volume
Vmin= minimum volume

Mean effective pressure is a fictitious pressure that, it acted upon on the piston would
produce the same amount of net work during the actual cycle.

Ratio of Expansion, Ratio of Compression

volume at the end of expansion


Expansion ratio =
volume at the beginning of expansion

V2
Isothermal expansion ratio=
V1
V
Isentropic expansion ratio= 3
V2
V
Overall expansion ratio= 3
V1
V
Isothermal compression ratio= 3
V4
V4
Isentropic compression ratio, rk =
V1
V3
Overall compression ratio=
V1

The isentropic compression ratio rk is the compression ratio most commonly used.

Problems

1. A Carnot power cycle operates on 2 lb of air between the limits of 700 F and 5000 F.
The pressure at the beginning of isothermal expansion is 400 psia and at the end of
isothermal expansion is 185 psig. Determine (a) the volume at the end of isothermal
compression. (b) S during an isothermal process, (c) QA, (d) QR, (e) W, (f) e, (g) the
ratio of expansion during isothermal heating and the overall ratio of expansion, and
(h) the mean effective pressure.

Page | 132
Solution

m = 2 lb p1 = 400 psia T1 = 960 oR

P2 = 199.7 psia T3 = 530 oR

Point 1:

=
V1
mRT1
=
( 2 )( 53.34 )( =
960 )
1.778 ft 3
p1 ( 400 )(144 )
Point 2:

=
V2
mRT2
=
( 2 )( 53.34 )(=
960 )
3.561 ft 3
p2 (199.7 )(144 )
Point 3:

k 1.4
T3 k 1 530 1.41
=p3 p=
2 (199.7) = 24.97 psia
T2 960

=
V3
mRT3
=
( 2 )( 53.34 )(=
530 )
15.72 ft 3
p3 ( 24.97 )(144 )
Point 4:

V1 1.778
=V4 V=
3 (15.72) = 7.849 ft 3
V2 3.561

(a) V4= 7.849 ft3

Page | 133
V2 2 ( 53.34 ) 3.561 Btu
(b) S=
1 2 mR ln= ln = 0.0952 0
V1 778 1.778 R

(c) QA = T1S = ( 960 )( 0.0952 ) = 91.39 Btu

(d) QR =T3 S = ( 530 )( 0.0952 ) =50.46 Btu

(e) W=Q A QR= 91.39 50.46= 40.93 Btu

W 40.93
= = 0.4477 or 44.77 %
(f) e=
QA 91.39
V2 3.561
(g) Isothermal expansion ratio= = =2
V1 1.778
V3 15.72
Overall expansion ratio = = = 8.84
V1 1.778

(h) mep=
W
=
W
=
( 40.93)( 778) = 15.86 psi
VD V3 V1 (15.72 1.778 )(144 )

2. A Carnot engine operating between 775 K and 305 K produces 54 kJ of work.


Determine (a) QA, (b) S during heat rejection, and (c) e.

Solution

T1= 775 K T3= 305 K W = 54 kJ

W 54
(a) Q=
A = = 89.04kJ
e 0.6065

(b) QR =
QA W =
89.04 54 =
35.04 kJ

Page | 134
QR 35.04 kJ
S3 4 = =
0.115
T3 305 K

T1 T3 775 305
=
(c) e = = 0.6065
T1 775

e = 60.65%

3. An ideal gas Carnot cycle uses helium as the working fluid and rejects heat to
lake at 150C. Determine the isentropic pressure ratio, compression ratio, and
minimum temperature of the heat source for this cycle to have a thermal
efficiency of 50 percent.

Solution:

For helium:
k= 1.666 T3= 15+273 =2880K e=50%

1
a) e = 1
rk k 1
1
0.5 = 1 1.666 1
rk
rk = 2.83

b) T1 = 9100 R

T1 288
0.5 = T1=5760K
T1
p1 p2
c) isentropic pressure ratio= = = ( rk ) = ( 2.83)
k 1.666
= 5.65
p4 p3

Page | 135
4 Consider two carnot heat engines operating in series. The first engine receives
heat from the reservoir at 18000K and rejects the waste heat to another reservoir at
temperature T. The second engine receives this energy rejected by the first one,
converts some of it to work, and rejects the rest to a reservoir at 3000K. If the
thermal efficiencies of both engines are the same, Determine the temperature T.

Solution:
18000K T
3000K

Engine 1 Engine 2

T1 =18000K T2 = 3000K

e1 = e2

T1 T T T2
=
T1 T

1800 T T 300
=
1800 T

T= 734.840K

5 A heat engine operating on a carnot cycle and has a thermal efficiency of 75


percent. The waste heat from this engine is rejected to a nearby lake at 60oF at a
rate of 800Btu/min. Determine (a) the power output of the engine (b) The
temperature of the source.

Solution:

Btu
e= 75% Q r = 800 T3=60 +460 =5200R
min

Page | 136

W Qa + Qr
=
e =
Qa Qa


Btu
Q a 800
min = 0.75

Qa

Btu
Q a = 3200
min

Btu
a) W = Q a + Q r = 3200 800 = 2400
min

T1 T3
b) e =
T1

T1 520
0.75 =
T1

T1 = 20800 R

6) Consider a Carnot cycle executed in a closed system with air as the working fluid. The
maximum pressure in the cycle is 800 kPa while the maximum temperature is 7500 K. If
the entropy increase during the isothermal heat rejection process is 0.25 kJ/kg-K and the
net work output is 100kJ/kg, Determine a) the minimum pressure in the cycle b) heat
rejection from the cycle c) the thermal efficiency of the cycle d) If an actual heat engine
cycle operates between the same temperature limits and produces 5200 kw of power for
an air flow of 90 kg/sec, Determine the efficiency of the engine.

Solution:

Page | 137
p1=800kpa T1=7500K S3-4= -0.25kJ/kg-K

w=100kJ/kg

p1
a) s =R ln
p2
800
0.25 = 0.28708ln
p2

p2 = 334.88kPa

s (T1 T3 )
w=

kJ
=
100 0.25 ( 750 T3 )
kg

T3 = 3500 K

k 1.4
T3 k 1 350 1.41
= p3 p=
2 334.88kPa = 23.25kPa
T2 750
b)
kJ kJ
qR =
0.25 ( 350 ) =
87.5
kg K kg

w 100
c) =
e = = 53.33%
q A 187.5

Wactual 5200kW
e
d)= = = 57.78%
Wideal ( 90kg / sec )(100kJ / kg )

Page | 138
Stirling Cycle
Stirling cycle differs from the Carnot cycle in that the two isentropic processes are
replaced by two constant volume regeneration processes. Regeneration is a process
during which heat is transferred to a thermal energy storage device, a regenerator, during
one part of the cycle and is transferred back to the working fluid during another part of
the cycle.
The system consists of a cylinder with two pistons on each side and a regenerator
in the middle. The regenerator can be a wire or a ceramic mesh or any kind of porous
plug with a high thermal mass. The mass of the working fluid contained within the
generator at any instant is considered negligible and the net heat transfer to the
regenerator during a cycle is zero.

Operation of the Stirling Cycle


Initially, the left chamber houses the entire working fluid, which is at high
temperature and pressure. During process 1-2, heat is transferred to the gas at TH from a
source at TH. As the gas expands isothermally, the left piston moves outward, doing
work, and the gas pressure drops. During process 2-3, both pistons are moved to the right
at the same rate(to keep the volume constant) until the entire gas is forced into the right
chamber. As the gas passes through the regenerator, heat is transferred to the regenerator
and the gas temperature drops from TH to TL. During process, 3-4, the right piston is
moved inward, compressing the gas. Heat is transferred from the gas to a sink at
temperature TL so that the gas temperature remains constant at TL while the pressure
rises. Finally, during process 4-1, both pistons are moved to the left at the same rate,
forcing the entire gas into the left chamber. The gas temperature rises from TL to TH as it
passes through the regenerator and picks up the thermal energy stored there during
process 2-3. This complete the cycle.

Figure 5-5 operation of stirling cycle

Page | 139
Analysis of Stirling cycle

1-2 isothermal expansion


2-3 constant volume regeneration (internal heat transfer from the working fluid to the
regenerator)
3-4 isothermal compression
4-1 constant volume regeneration(internal heat transfer from regenerator back to the
working fluid)

=
QA T1 ( S 2 S1 )

=
QR T3 ( S 4 S3 )

But S1-2= S3-4

W = QA QR = T1 ( S 2 S1 ) T3 ( S 2 S1 )

= (T1 T3 )( S 2 S1 )

W (T1 T3 )( S 2 S1 )
=
e =
QA (T1 )( S 2 S1 )

T1 T3
e=
T1

Problems:

1. Consider an ideal Stirling cycle using air as the working fluid. Air is at 3500K
and 200 kPa at the beginning of the isothermal compression process, and heat is
supplied to air from a source at 18000K in the amount of 900 kJ/kg. Determine
(a) the maximum pressure in the cycle and (b) the net work output per unit mass
of air.

Page | 140
Solution:

T3= 3500K p3= 200 kPa T1= 18000K q1-2= 900 kJ/kg

Basis unit mass:

p3T2 200kPa (1800 K )


0

a) =
p2 = = 1028.57kPa
T3 3500 K

p1
q1 2 = RT1 ln
p2

kJ p1
900 = 0.28708 (1800 ) ln
kg 1028.57

p1 = 5870 kPa

p1T4 5870kPa ( 350 K )


0

b) =p4 = = 1141.39kPa
T1 18000 K
p
=
qr q= 3 4 RT3 ln 3
p4
200 kJ
qr = 0.28708 ( 350 ) ln = 175
1141.39 kg
kJ
w = qa qr = 900 175 = 725
kg

Page | 141
2. An ideal Stirling cycle uses energy reservoirs at 400F and 6400F, and uses air as
the working gas. It is designed such that its minimum volume is 0.1ft3, maximum
volume is 1 ft3, and maximum pressure is 400 psia. Calculate a) the amount of
external heat addition b) the external heat rejection c) heat transfer between the
working fluid and regenerator d) thermal efficiency of the cycle.
Solution:

T1= 640 + 460= 11000R T3= 40 + 460= 5000R V1= 0.1 ft3

V2= 1 ft3 p1= 400 psia

3 in 2
400
lb
in 2
( 0.1 ft ) ft 2 1
144
=
a) Qa p=
V1 = ln
1V1 ln 17.047 Btu
V2 ft lb 0.1
778
Btu

3 in 2
400
lb
in 2
( 0.1 ft ) ft 2
144
b) =
m
p1V1
= = 0.0982lbs
ft lb
RT1 53.34 (1100 )
lb R

ft lb
( 500 R ) 0.1
0
0.0982lbs 53.34
V4 lb R
Qr = mRT3 ln = ln = 7.75 Btu
V3 ft lb 1
778
Btu

c) Heat absorbed by regenerator:

=Q mcv (T3 T2 )

=Q 0.0982 ( 0.1714
= )(1100 500 ) 10.09 kJ
T1 T3 T1 T3 1100 500
d) e = = e = = 54.54%
T1 T1 1100

Page | 142
Ericsson Cycle

The Ericsson cycle is very similar to the stirling cycle except the regeneration
occurs in a constant pressure process.

A steady-flow system operating on an Ericsson cycle is shown in Figure 5-6. Here the
isothermal expansion and compression processes are executed in a compressor and a
turbine and a counterflow heat excanger serves as a regenerator. In ideal case, the
temperature difference between the two streams does not exceed a differential amount at
any point, and the cold fluid leaves the heat exchanger at the inlet temperature of the hot
stream.

Analysis of the Ericsson Cycle

1-2 isothermal expansion


2-3 constant pressure compression
3-4 isothermal compression
4-1 constant pressure expansion

=
QA T1 ( S 2 S1 )

=
QR T3 ( S 4 S3 )

Page | 143
But S1-2= S3-4

W = QA QR = T1 ( S 2 S1 ) T3 ( S 2 S1 )

= (T1 T3 )( S 2 S1 )

W (T1 T3 )( S 2 S1 )
=
e =
QA (T1 )( S 2 S1 )

T1 T3
e=
T1
Note: Stirling and Ericsson cycles are difficult to achieve in practice because they
involve heat transfer through a differential temperature difference in all components
including the regenerator.
Both these cycles are external combustion engines, that is the fuel in these engines
is burned outside the cylinder.

Problems:

1) Consider an ideal Ericsson cycle with air as the working fluid executed in a steady
flow system. Air is at 270C and 120 kPa at the beginning of the isothermal
compression, during which 150 kJ/kg of heat is rejected. Heat transfer to air
occurs at 12000K. Determine (a) the maximum pressure in the cycle (b) the net
work output per unit mass of air, and (c) the thermal efficiency of the cyce (d)
mean effective pressure.

Solution :

T3= 27 + 273= 3000K p3= 120 kpa qR= -150kJ/kg

T1= 12000K

Page | 144
p3
a) qr = RT3 ln
p4
120
( 0.28708)( 300 ) ln
150 =
p4

p4 = 684.83kPa

p1 684.83 kJ
= =
b) qa RT1 ln ( 0.28708)(1200 ) ln= 600
p2 120 kg

kJ
w =qa qr =600 150 =450
kg
w 450
c) =
e = = 75%
qa 600

d) at point 3

kN m
0.28708 kg K ( 300 K )
0

RT3 m3
= v3 = = 0.7177
p3 kN kg
120 2
m
at process 3-4:

p3v3 120 ( 0.7177 ) m3


=
v4 = = 0.1258
p4 684.83 kg

at process 2-3

v3T2 0.7177 (1200 ) m3


=
v2 = = 2.8708
T3 300 kg

kn m
450
w w kg
=
mep = = = 163.93kPa
vmax vmin v2 v4 m3
( 2.8708 0.1258)
kg

2. The Ericsson cycle is composed of two isothermal processes and two isobaric
processes, with regenerative heat exchange during isobaric processes. Properties
at the beginning of isothermal expansion are 689.48 kPaa, 142 liters, and 282.20C.
For a ratio of isothermal expansion of 2 and a minimum temperature of 4.40C,

Page | 145
Find (a) S during the isothermal process; (b) Qa, QR, W, e (c) the volume at the
end of isothermal expansion and the over-all ratio of expansion; and (d) mep.

Solution

p1=689.48 kPaa

V1= 142 liters T1=282.2+273=555.2 0K

isothermal expansion ratio= 2 T3=4.4+273=277.40K

kn
689.48 2 ( 0.142m3 )
=
p1V1
m = m
= 0.6142kg
RT1 kn m
0.28708 kg K ( 555.2 K )
0

V kJ
a)=
S mR=
ln 2 ( 0.6142 )( 0.28708=
) ln 2 0.1222
V1 K
b) QA = ( 0.1222 )( 555.2 ) =
S T1 = 67.86kJ

QR = ( 0.1222 )( 277.4 ) =
S T3 = 33.9kJ

W = QA QR = 67.86 33.9 = 33.96kJ

= ( 0.142 ) 0.284m3
c) V2 2=

T4 277.4
=V4 V=
1 0.142 = 0.071m
3

T1 577.2

W 33.96
=
d) mep = = 159.44kPa
Vmax Vmin 0.284 0.071

Page | 146
Three- Process Cycle

Problems

1. Ten cu ft of helium at 20 psia and 800 F are compressed isentropically 1-2 to 80 psia.
The helium is then expanded polytropically 2-3 with n=1.35 to the initial temperature. An
isothermal 3-1 returns the helium to the initial state. Find T2, V2,p3, QA,QR, W, S3-1,
and mep.

Solution:

p1= 20 psia T1= 540 0R V1= 10 cu ft

p2= 80 psia T3= 5400R

=
m
p1V1
=
( 20 )(144 )(=
10 )
0.1382 lb
RT1 ( 386.04 )( 540 )
Point 2:

k 1 1.666 1
p2 k
( 540 ) =
80 1.666
=T2 T=
1 939.90 R
p1 20

1 1
p1 k
(10 ) =
20 1.666
=V2 V=
1 4.351 ft 3
p2 80

Point 3:

n 1.35
T3 n 1
(80 ) =
540 1.351
=p3 p=
2 9.435 psia
T2 939.9

Page | 147
1 1
p2 n
( 4.351) =
80 1.35
=V3 V=
2 21.2 ft 3
p3 9.435

k n 1.666 1.35 Btu


cn = cv = ( 0.754 ) = 0.6808 0
1 n 1 1.35 lb. R

=QA ( m )( cn )(T3 T2 )

Q=
A ( 0.1382 )( 0.6808)( 540 939.9=) 37.62 Btu

V1
QR = mRT1 ln
V3
0.1382 ( 386.04 )( 540 ) 10
QR = ln = 27.82 Btu
778 21.2

W = QA QR = 37.62 27.82 = 9.79 Btu

Q 27.82 Btu
S31 == =
0.0515 0
T 540 R

W 9.79 ( 778 )
=mep = = 3.13 psi
Vmax Vmin ( 21.2 4.351)144
2. Two and a half kg of an ideal gas with R=296.9 J/kg-K and cv= 0.7442 kJ/kg-K at
a pressure of 827.4 kPa and a temperature of 6670C reject 132.2 kJ of heat at constant
pressure. The gas is then expanded according to pV1.25 = C to a point where a
constant volume process will bring the gas back to its original state. Determine p3,
Qa, and the power in kW for 100Hz.

Solution:

Page | 148
p1=827.4 kPa T1= 677 +273= 9500K Qr = -132.2 kJ

kJ
c p = cv + R = 0.7442 + 0.2969 = 1.0411
kg K

cp 1.0411
=
k = = 1.399
cv 0.7442

Point 1:

kN m
( 950 K )
0
2.5 kg 0.2969
mRT1 kg K
=
V1 = = 0.8522 m3
p1 kN
827.4 2
m
Point 2:

=Qr mc p (T2 T1 )

=
132.2 ( 2.5)(1.0411)(T2 950 )

T2 = 899.20 K

T2
( 0.8522 ) =
899.2
=V2 V=
1 0.8066 m3
T1 950

Point 3:

n 1 1.25 1
V2
(899.2 )
0.8066
=T3 T=
2 = 886.90 K
V1 0.8522

n
V2
1.25

(827.4 ) =
0.8066
=p3 p=
2 772.4 kPa
V3 0.8522

k n 1.399 1.25
( 0.7442 )
kJ
=
cn = cv = - 0.4435
1 n 1 1.25 kg K

=
Qa mc n (T3 T2 ) + mcv (T1 T3 )

Qa = ( 2.5)( 0.4435)(886.9 899.2 ) + ( 2.5)( 0.7442 )( 950 886.9 ) = 131kJ

Page | 149
W = Q a Qr = 131 - 132.2= -1.2 kJ

kJ cycles
=W =
1.2 100 -120kW
cycle sec

3. A thermodynamic cycle is composed of the following processes: polytropic


compression with n k 1-2; isothermal 2-3; isentropic 3-1. It operates on 1.36 kg
of air for which these data apply: p1= 103.4 kPaa, t1= 37.80C, t2= 426.70C, S2-3=
-0.456 kJ/K. Determine (a) p, V, T at each point (b) the polytropic exponent n,
(c) QA and QR, (d) W from the pV plane (check by net Q), (e) the thermal
efficiency, (f) mep, (g) If 50 Hz are completed find the power in hp and kW.

Solution:

p1= 103.4 kPaa T1= 37.8 + 273= 310.80K T2= 426.7 + 273=699.70K

S2-3= -0.456 kJ/K m=1.36 kg

kN m
1.36kg 0.287
kg K
( 310.80 K )
a)=
mRT1
V1 = = 1.17 m3
p1 kN
103.4 2
m
1 1
T1 k 1 310.8 1.41
=V3 V=
1 1.17 = 0.154m3
T3 699.7

k 1.4
T3 k 1 699.7 1.41
=p3 p=
1 103.4 = 1770.22kPaa
1
T 310.8

Page | 150
p2
S =mR ln
p3
p2
(1.36 )( 0.28708) ln
0.456 =
1770.22

p2=550.55 kPaa

p3 1770.22
=V2 V=
3 0.154 = 0.495m
3

2
p 550.55
n
p1 V2
b) =
p2 V1

n
103.4 0.495
=
550.55 1.17

n= 1.937

c)=
QA mcn (T2 T1 )

k n 1.4 1.937
( 0.186 )
kJ
=cn c=
v = 0.411
1 n 1 1.937 kg K

=QA (1.36 )( 0.411)( 699.7


= 310.8 ) 217.41kJ
V3 0.154
= =
QR mRT2 ln (1.36 )( 0.28708)( 699.7 )=
ln -319kJ
V2 0.4951
d) W = pdV

mR (T2 T1 ) V mR (T1 T3 )
W= + mRT3 ln 3 +
1 n V2 1 k

(1.36 )( 0.28708)( 699.7 310.8) + (1.36 )( 0.28708)( 310.8 699.7 )


W ( 319.02 ) +
1 1.937 1 1.4

W = 101.62kJ

Check:
W= QA + QR =
217.41 319.02 =
101.62kJ

Note: the negative sign on the net work means that the cycle acts as a refrigeration
cycle.

Page | 151
W 101.62
e) =
e = = 46.67%
QA 217.41

W 101.47
f) =
mep = = 99.48kPa
VD 1.174 0.154

kJ cycles
g) W =101.47 50 5073.5kW
cycle sec \

Review Problems

1. The working substance for a Carnot cycle is 8 lb of air. The volume at the
beginning of isothermal expansion is 9 cu ft and the pressure is 300 psia. The ratio
of expansion during the addition of heat is 2 and the temperature of the cold body
is 900F. Find (a) Qa, (b) Qr, (c) V3, (d) p3, (e) V4, (f) p4, (g) pm, (h) the ratio of
expansion during the isentropic process, and (i) the overall ratio of compression.
Ans. (a) 346.4 Btu; (b) -209.1 Btu; (c) 63.57 cu ft; (d) 25.64 psia; (e)
31.79 cu ft; (f) 51.28 psia; (g) 13.59 psia; (h) 3.53; (i) 7.06

2. Gaseous nitrogen actuates a Carnot power cycle in which the respective volumes
at the four corners of the cycle, starting at the beginning of the isothermal
expansion, are V1= 10.10 L, V2= 14.53 L, V3=226.54 L, V4=157.73 L. The cycle
receives 21.1 kJ of heat. Determine (a) the work and (b) the mean effective
pressure.
Ans. (a) 14.05 kJ; (b) 64.91 kPa

3. show that the thermal efficiency of the Carnot cycle in terms of the isentropic
1
compression ratio rk is given by e = 1 k 1
rk

4. An Ericsson cycle operates on 0.75 lb of oxygen from 60 psia and 12000 F at the
beginning of isothermal process to the lower temperature limit of 2000F. If the
isothermal expansion ratio is 3. Determine a) QA and QR b) e c) mep d) What is
the efficiency of the same cycle without regeneration?

5. An ideal Stirling cycle operates with 1 kg of air between thermal energy


reservoirs at 270C and 5270C. The maximum cycle pressure is 2000kPa and the
minimum cycle pressure is 100 kPa. Determine the net work produced and the
cycle thermal efficiency. (b) the external heat input and power produced when it
is repeated 500 times per minute.

Page | 152
6. Two and one-half pounds of air actuate a cycle composed of the following
processes; polytropic compression 1-2, with n= 1.5; constant pressure 2-3;
constant volume 3-1. The known data are : p1= 20 psia, T1=1000F, QR=-1682 Btu.
Determine (a) T2 and T3, (b) the work of the cycle using the pV plane, in Btu; (c)
QA, (d) the thermal efficiency, and (e) pm
Ans. (a) 11200R, 44850R; (b) 384.4 Btu; (c) 2067 Btu; (d) 18.60%; (e)
106.8 psi

7. A three- process cycle of an ideal gas, for which cp= 1.064 and cv= 0.804
kJ/kg.K0, is initiated by an isentropic compression 1-2 from 103.4 kPa, 270C to
608.1 kPa. A constant volume process 2-3 and a polytropic 3-1 with n=1.2
complete the cycle. Circulation is a steady rate of 0.905 kg/s, compute (a) QA , (b)
W, (c) e, (d) pm.
Ans. (41.4 kJ/s; (b) -10 kJ/s; (c) 24.15%; (d) 19.81 kPa

8. A three process operates on 113g of nitrogen is initiated by an isentropic


expansion 1-2 with an expansion ratio of re= 5, p1= 689.5 kPaa, t1=1490C; an
isothermal process 2-3 and constant pressure 3-1. Determine (a) p, V, T at each
point, (b) QA and QR, (c) W from the pV plane (d) e and mep (e) if 100 Hz are
completed find the power in kW. Ans. (b) 23.53, -16.8 kJ (c) 6.73 kJ (d)73.3 kPa
(e) 673kW

9. A thermodynamic cycle is composed of the following reversible processes:


isothermal expansion, 1-2; polytropic (n=4), 2-3; isentropic 3-1. The cycle
operates on 0.25 lbs of nitrogen for which the following data apply for the process
1-2; the expansion ratio re = 5; p1= 100 psia and t1= 8000F. (a) Determine the p,V,
T at each point (b) QA and QR (c) mep (d) e (e) the speed in rpm to produce 250 hp
assuming for every revolution 1 cycle is completed.

10. A thermodynamic cycle is composed of the following processes:


Constant volume 1-2
Polytropic 2-3 with n= - 1.1
Isentropic 3-1
The cycle operates on a 0.75lb/sec of a gas (R=80 ft-lb/lb-R), k=1.3 and the
following data applies: p1= 15 psia, t1 = 1500F, and the isentropic expansion ratio,

re= 3. Compute a) p, V and T at each point b) Q a and Q r c) W in hp and kw ; and
d) e and pm.

Page | 153
6 Internal Combustion Engines
Internal combustion engine is a heat engine deriving its power from the energy
liberated by the explosion of a mixture of some hydrocarbon, in gaseous or vaporized
form, with atmospheric air.

In internal combustion engine energy is provided by burning a fuel within the


system boundaries and because of this combustion process, the composition of the
working fluid changes from air and fuel to combustion products during the course of the
cycle. However, air which is predominantly nitrogen which hardly undergoes a chemical
reaction in the combustion chamber, the working fluid closely resembles air at all times.

The actual cycles are rather complex, to reduce the analysis to a manageable
level, It is a common practice to use the air-standard assumptions:

the air - standard assumptions:


1. The working fluid is air, which continuously circulates in a closed loop and always
behaves as an ideal gas.
2. All the processes that make up the cycle are internally reversible.
3. The combustion process is replaced by a heat addition process from an external
source.
4. The exhaust process is replaced by a heat rejection process that restores the working
fluid to its initial state.

AIR
COMBUSTION
Combustion
chamber
PRODUCTS
FUEL
(a) Actual
HEAT

AIR AIR
HEATING
section

(b) ideal
Figure 6-1

Page | 154
Another assumption that is often utilized to simplify the analysis even more is that
air has constant specific heats whose values are determined at room temperature. When
this assumptions are called cold-air-standard assumptions. A cycle for which the air-
standard assumptions are applicable is frequently referred to as an air-standard cycle.

Spark Ignition (SI) or Gasoline Engine

The four-stroke cycle is one wherein four strokes of the piston, two revolutions,
are required to complete a cycle.
Schematic diagram of each stroke for an actual four-stroke spark ignition engine with the
pV diagram showing the various stages of the cycle.

Figure 6-2

Description of the four stroke spark ignition engine:

Initially, both intake and the exhaust valves are closed, and the piston is at its
lowest position (BDC). During the compression stroke, the piston moves upward,
compressing the air fuel mixture. Shortly before the piston reaches its highest
position(TDC), the spark plug fires and the mixture ignites, increasing the pressure and
temperature of the system. The high pressure gases force the piston down, which in turn
forces the crankshaft to rotate, producing a useful work output during the expansion or
power stroke. At the end of this stroke, the piston is at its lowest position and the
cylinder is filled with combustion products. Now the piston moves upward one more
time, purging the exhaust gases through the exhaust valve, the exhaust stroke. The piston
moves a second time drawing in fresh air-fuel mixture through the intake valve, the
intake stroke. Notice on the pV diagram that the pressure in the cylinder is slightly above
the atmospheric value during the exhaust stroke and slightly below during the intake
stroke.
If we will use air- standard assumptions, the resulting cycle will closely resemble
the actual condition , which is the ideal Otto Cycle.

Page | 155
Figure 6-3

Analysis of 0tto Cycle

It consists of four internally reversible processes:

Figure 6-4

1-2: isentropic compression


2-3: constant volume addition of heat
3-4: isentropic expansion
4-1: constant volume rejection of heat

=
QA mcv (T3 T2 )

QR =
mcv (T1 T4 ) =
mcv (T4 T1 )

W = QA QR = mcv (T3 T2 ) mcv (T4 T1 )

Page | 156
W mcv (T3 T2 ) mcv (T4 T1 )
=
e =
QA mcv (T3 T2 )

(T4 T1 )
e= 1
(T3 T2 )

1
e= 1 (1)
rkk 1

V1
Where rk= , the isentropic compression ratio
V2

Derivation for the formula for e

Process 1-2:

k 1
T2 V1
=
T1 V2 (2)
T2 = T1rkk 1

Process 3-4

k 1 k 1
T3 V4 V1
= =
T4 V3 V2 (3)
k 1
T3 = T4 rk

Substituting the equations (2) and (3) in equation (1)

(T4 T1 )
e= 1
T4 rkk 1 T1rkk 1
1
e = 1 k 1
rk

Work for the pV plane

p V p1V1 p4V4 p3V3


W=
W =2 2 +
1 k 1 k

Page | 157
Clearance volume, per cent clearance

V1 VD + V3 VD + cVD
rk = = =
V2 V3 cVD

1+ c
rk =
c

Where c= per cent clearance


V3= clearance volume
VD= displacement volume

MEAN EFFECTIVE PRESSURE

Wnet
mep =
Vmax Vmin

Ideal standard of Comparison

Cold- air standard, k=1.4


Hot air standard, k<1.4

The thermal efficiency of the theoretical Otto cycle is

1. Increased by an increase in rk
2. Increased by an increase in k
3. Independent of the heat added

The average family car has a compression ratio of about 9:1

The economical life of the average car is 8 years or 80,000 miles of motoring.

Problems

1. An Otto cycle operates on 0.1 lb/s of air from 13 psia and 1300F at the beginning
of compression. The temperature at the end of combustion is 50000R;
compression ratio is 5.5; hot air standard, k= 1.3. (a) Find V1, p2, T2, p3, V3, T4,
and p4. (b) Compute QA, QR, W, e, and the corresponding hp.

Page | 158
Solution


m = 0.1 lb / sec rk = 5.5 k= 1.3

p1 = 13 psia T1 = 130 +460 = 5900R T3 = 50000R

(a) Point 1:

=
V1
mRT1
=
( 0.1)( 53.34 )( =
590 )
1.681
ft 3
p1 (13)(144 ) sec

Point 2:

k

= V 1
p2 p1 = 1 ( rk )
p=
k
(13)(=
5.5 )
1.3
119.2 psia
V
2

k 1

V1
T2 = T1 = T1rkk 1 = (590) ( 5.5 )
1.31
= 983.90 R
V
2


V 1 1.681 ft 3
V=
2 = = 0.3056
rk 5.5 sec

Point 3:

ft 3
V=3 V=2 0.3056
sec
p2 119.2
= p3 T=
3 ( 5000 ) = 605.8 psia
T2 983.9

Page | 159
Point 4:

k 1
1.31
V 3 1
=T4 T= ( 5000 )=
5.5 2998 0 R
V
3

p1
13
=p4 T=
4 ( 2998)=
66.1 psia
T1 590

R 53.34 Btu
(b)=
cv = = 0.2285
k 1 ( 778)(1.3 1) lb.R 0


Q=
A m cv (T3 T=
2) ( 0.1)( 0.2285)( 5000 983.9 )
Btu
Q A = 91.77
sec

Q=
R m cv (T1 T=
4) ( 0.1)( 0.2285)( 590 2998)
Btu
Q R = 55.03
sec
Btu
= Q A Q R = (91.77 55.03)= 36.75
W
sec
W 36.75
e= = = 0.4005 or 40.05%
QA 91.77

Btu s
36.75 60

W =
s min
= 52 hp
Btu
42.4
( min )( hp )
2. The conditions at the beginning of the compression in an Otto engine operating on hot
air standard with k = 1.34, are 101.3 kPa, 0.038 m3 and 320C. The clearance is 10 %
and 12.6 kJ are added per cycle. Determine (a) V2, T2, p2, T3, p3, and p4, (b) W, (c) e
and (d) mep,

Page | 160
Solution

p1=101. 3 kPa V1= 0.038 m3 T1= 32 + 273 = 305 K

R 0.28708 kJ
=
cv = = 0.8444
k 1 (1.34 1) kg .K

=
m
p1V1
=
(101.3)( 0.038
=
) 0.04396 kg
RT1 ( 0.28708)( 305)
1 + c 1 + 0.10
=
rk = = 11
c 0.10

(a) Point 2

V2 0.038
V=
1 = = 0.003455 m3
rk 11

( 305)(11)
k 1 1.34 1
=
T2 T=
1rk =689 K

=
p2 p= (101.3)(11=
)
k 1.34
1rk 2518 kPa

Point 3:

=
QA mcv (T3 T2 )

12.6= (0.04396) (0.8444) ( T3 -689)

T3 =1028 K

Page | 161
T3
( 2518) =
1028
=p3 p=
2 3757 kPa
T2 689

Point 4:

k 1 k 1 1.34 1
V3 1
(1028) =
1
=T4 T3=
T=
3 455 K
V4 rk 11

k k
V3 1
1.34
1
=p4 p=
3 p=
3 ( 3757 ) =
11 151 kPa
V4 rk

=
(b) QR mcv (T1 T4 ) = (0.04396) (0.8444) (305 455)

QR = - 5.57 kJ

= QA QR = 12.6 -5.57= 7.03 kJ


W

W 7.03
(c)=
e = = 0.559 or 55.8%
QA 12.6

W W 7.03
=
(d) mep = = = 203.47 kPa
Vd V1 V2 0.038 0.003455

3. An air standard otto cycle has a compression ratio of 8.5. At the beginning of
compression the pressure is 100 kPa and temperature is 3000K. The heat addition
per unit mass of air is 1400 KJ/ kg. Determine (a) W (b) e (c) mep (d) maximum
temp in the cycle (e) percent clearance

Solution

Page | 162
rk= 8.5 p1= 100 kPa T1= 3000K qA= 1400 kJ/kg

1 1
(a) e =
1 k 1
=
1 =
57.51%
rk (8.5)1.41

w kJ kJ
e = ; w= ( 0.5751) 1400 805.21
qa kg kg

(b) e = 57.51%

w
(c) mep =
vD

kJ
0.28708 kg K ( 300 K )
0

RT1 m3
=
v1 = = 0.861
p1 kN kg
100 2
m

0.861m3 / kg m3
=v2 = 0.1013
8.5 kg

805.21
=mep = 1060kpa
( 0.861 0.1013)
k 1
V1
=
(d) T2 T= = (8.5) 706.140 K
0.4
1 3000 K
V2
1400
qa =cv (T3 T2 ) ; T3 = + 706.14 =2654.37 K
0

0.7186

1+ c
(e) rk =
c

1
=
c = 13.33%
7.5

Page | 163
Compression Ignition or Diesel Engine

Figure 6-5

A cycle begins with the intake stroke when the piston moves down and draws air
into the cylinder. The piston rises and compresses air into the cylinder. The piston rises
and compresses the air. During the compression stroke, the temperature of the air rises to
about 9000F (4800C). When oil is injected into the cylinder, it mixes with the hot air and
burns explosively. Gases produced by this combustion action push the piston down for
the power stroke. During the exhaust stroke, the piston moves up again and forces the
burned gases out of the cylinder.

1-2: Isentropic compression


2-3: constant pressure addition of heat
3-4: Isentropic expansion
4-1: constant volume rejection of heat

Page | 164
Analysis of the Diesel Cycle

=
QA mc p (T3 T2 )

QR =
mcv (T1 T4 ) =
mcv (T4 T1 )

= QA QR = mc p (T3 T2 ) mcv (T4 T1 )


W

W mc p (T3 T2 ) mcv (T4 T1 )


e= =
QA mc p (T3 T2 )

T4 T1
e= 1 (4)
k (T3 T2 )
1( rck 1)
e= 1
rkk 1 ( k )( rc 1)
V1
Where rk = , the compression ratio
V2

V3
rc = , The cutoff ratio
V2

Point 3 is called the cutoff point

Derivation of the formula for e

Process 1-2:

k 1
T2 V1
=
T1 V2

T2 = T1rkk 1 (5)

Process 2-3:

T3 V3
= = rc
T2 V2

T3 = T1rkk 1rc (6)

Page | 165
Process 3-4:

k 1 k 1
T4 V3 V2 rc rc k 1
= =
= k 1
T3 V4 V1 rk

r k 1
T4 = T1rkk 1rc c k 1
rk
T4 = T1rc k
(7)

Substituting equations (5), (6), and (7) in equation (4).

T1rc k T1
e= 1
k (T1rkk 1rc T1rkk 1 )

rc k 1
e= 1
krkk 1 ( rc 1)

The efficiency of the diesel cycle differs from the Otto cycle by the racketed factor
rc k 1
. This factor is always greater than 1, because rc is always greater than 1. Thus,
k ( rc 1)
for a particular compression ratio rk , the Otto cycle is more efficient. However, since the
Diesel engine with a compression ratio of 15 is more efficient than an actual Otto engine
with a compression ratio of 9.

Relation among rk, rc, and re (expansion ratio)

V4 V1
=
re =
V3 V3
V1 V3 V1
=
rk =
V2 V2 V3
rk = rc re

Problems

1. A diesel cycle operates with a compression ratio of 13.5 and with a cutoff
occurring at 6% of the stroke. State 1 is defined by 14 psia and 1400F. For the
hot air standard with k=1.34 and for an initial 1 cu ft, compute (a) T2, p2, T3, V3,
p4, T4 (b) QR (c) W, (d) e and mep .(e) For a rate of circulation of 1000 cfm,
compute the horsepower.

Page | 166
Solution

rk= 13.5 k= 1.34 p1= 14 psia T1= 140 + 460 = 6000R

V1= 1 cu ft

R 53.34 Btu
=
cv = = 0.2016
k 1 ( 778)(1.34 1) lb.R 0

Btu
= =
c p kcv (1.34 )( 0.2016=) 0.2702
lb.R 0

=
m
p1V1
=
(14 )(144=)1 0.630 lb
RT1 ( 53.34 )( 600 )
(a) Point 2

V1 1
V=
2 = = 0.0741 ft 3
rk 13.5

( 600 )(13.5)
k 1 1.34 1
=
T2 T=
1rk =1454 0R

T2= 994 0F

=
p2 p= (14 )(13.5=
)
k 1.34
1rk 457.9 psia

Point 3:

V3 =
V2 + 0.06VD =
V2 + 0.06(V1 V2 )

V3 =0.0741 + ( 0.06 )(1 0.0741) =0.1297 ft 3

Page | 167
V3 0.1297
=T3 T=
2 (1454 ) = 2545 0 R
V2 0.0741

T3= 2085 0 F

Point 4:

k 1 1.34 1
V3
( 2545)
0.1297
=T4 T=
3 = 1271 0 R
V4 1

T4= 811 0 F

k
V3
1.34

( 457.9 ) =
0.1297
=p4 p=
3 29.7 psia
V4 1

(b)
Q=
A mc p (T3 T=
2) ( 0.063)( 0.2702 )( 2545 1454 )
QA = 18.57 Btu
Q=
R mcv (T1 T4=
) ( 0.063)( 0.2016 )( 600 1271)
QR = 8.52 Btu

= QA QR =18.57 8.52 = 10.05 Btu


(c) W

W 10.05
(d)=
e = = 0.5412 or 54.12%
QA 18.57

W 10.05 ( 778 )
=
mep = = 58.64 psi
VD (1 0.0741)144

Btu ft 3

ft 3
10.05 1000

=
(e) W = 237 hp
min
Btu
42.4
( min )( hp )
2. There are supplied 317 kJ/cycle to an ideal Diesel engine operating on 227 g
air:p1= 97.91 kPa, T1=48.90C. At the end of compression, p2= 3930 kPa.
Determine (a) rk, (b) c, (c) rc, (d) W, (e) e and (f) mep.

Page | 168
Solution

m= 0.221 kg
p1= 97.91 kPa
T1= 48.9 +273 = 321.9 K
p2= 3930 kPa
QA= 317 kJ/cycle

Point 1:

=
V1 =
mRT1 ( 0.227 )( 0.28708)( 321.9
=
) 0.2143 ft 3
P1 ( 97.91)
Point 2:

1 1
p1 k
( 0.2143) =
97.91 1.4
=V2 V=
1 0.0153m3
p2 3930
k 1 1.4 1
p2 k
( 321.9 ) =
3930 1.4
=T2 T=
1 924.40 K
p1 97.91

Point 3:

Q=
A mc p (T3 T=
2) ( 0.227 )(1.0062 )(T3 924.4 )

T3 = 2312 K

T3
( 0.0153) =
2312
=V3 V=
2 0.0383 m3
T2 924.4

Page | 169
Point 4:

k 1 1.4 1
V3
( 2312 ) =
0.0283
=T4 T= 3 1161 K
4
V 0.2143
V1 0.2143
(a) =
rk = = 14
V2 0.0153
1+ c
(b) rk =
c
1+ c
14 =
c
c= 0.0769 or 7.69%

V3 0.0383
(c) =
rc =
V2 0.0153

(d) Q=
R mcv (T1 T4=
) ( 0.227 )( 0.7186 )( 321.9 1161)
QR = 136.9 kJ
= QA QR =317 136.9 = 180.1 kJ
W

W 180.1
(e)=
e = = 0.5681 or 56.81%
QA 317

W W 180.1
=
(f) mep = = = 905 kPa
Vd V1 V2 0.2143 0.0153

3. An ideal diesel cycle has a maximum cycle temperature of 20000C and a cut off
ratio of 2.2; isentropic compression ratio of 9. The state of the air at the
beginning of compression is p1=95 kPaa and t1=150C. Determine the specific
work of the cycle.
Solution

Page | 170
p1= 95 kpaa T1= 15+273=2880K T3=2000+273=22730K rc=2.2

At point 2:

v2 1
=T2 T=
3 2273=

0
1033.18 K
3
v 2.2

At point 4:
rc k 1 ( 2.2 )1.41
=T4 T=
3 k 1 2273 = 1.4 1
1293.810 K
rk ( 9 )

kJ
qa= c p (T3 T2 )= 1.0062 ( 2273 1033.18 )= 1247.51
kg

kJ
cv (T1 T4 ) =
qR = 0.7186 ( 288 1293.81) =
722.77
kg
kJ
w = qa + qr = 1247.51 722.77 = 524.74
kg

Page | 171
Dual Cycle
Approximating the combustion process in internal combustion engines as s constant-
volume or a constant-pressure heat addition process is overly simplistic and not quite
realistic. A better approach would be to model the combustion process in both gasoline
and diesel engines as a combination of two heat transfer processes, one at constant
volume and the other at constant pressure. The ideal cycle based on this concept is called
the dual cycle

1-2: isentropic compression


2-3: constant volume addition of heat
3-4: constant pressure addition of heat
4-5: isentropic expansion
5-1: constant volume rejection of heat

Analysis of Dual Combustion Cycle

Q=
A mcv (T3 T2 ) + mc p (T4 T3 )

QR =
mcv (T1 T5 ) =
mcv (T5 T1 )

W = QA QR = mcv (T3 T2 ) + mc p (T4 T3 ) mcv (T5 T1 )

W mc (T T ) + mc p (T4 T3 ) mcv (T5 T1 )


e= =e = v 3 2
QA mcv (T3 T2 ) + mc p (T4 T3 )

T5 T1
e= 1 (8)
(T3 T2 ) + k (T4 T3 )

rp rc k 1
e= 1
rkk 1 ( rp 1 + rp k ( rc 1) )

Page | 172
p3
Where rp = , the pressure ratio during the constant volume portion of combustion
p2

V1
rk = , the compression ratio
V2
V
rc = 4 , the cutoff ratio
V3

The thermal efficiency of this cycle lies between that of the ideal Otto and the Ideal
Diesel.

Derivation of formula for e

Process 1-2

k 1
T2 V1
=
T1 V2

T2 = T1rkk 1 (9)

Process 2-3

T3 p3
= = rp
T2 p2

T3 = T1rk k 1rp (10)

Process 3-4

T4 V4
= = rc
T3 V3

T4 = T1rk k 1rp rc (11)

Process 4-5

k 1 k 1 k 1 k 1
T5 V4 V4 V3 rc V2 rc rck 1
= =
=
= =
T4 V5 V1 V1 V1 rkk 1

Page | 173
r k 1
T5= T1rk k 1rp rc ck 1
rk

T5 = T1rp rc k (12)

Substituting equations (9), (10), (11), and (12) in equation (8).

T1rp rc k T1
e= 1
(T r
1 k
k 1
rp T1rkk 1 ) + k (T1rk k 1rp rc T1rk k 1rp )

rp rc k 1
e= 1
rk k 1 ( rp 1) + krp ( rc 1)

Problems

1. At the beginning of compression in an ideal combustion cycle, the working fluid


is 1 lb of air at 14.1 psia and 800F. The compression ratio is 9, the pressure at the
end of the constant volume addition of heat is 470 psia, and there are added 100
Btu during the constant pressure expansion. Find (a) rp, (b) rc, (c) the percentage
clearance, (d) e, and (e) mep

Solution

m= 1 lb air
p1= 14.1 psia
T1= 80 + 460 = 540 0R
p3= 470 psia
rk = 9
Q3-4= 100 Btu

Point 1:

=
V1
mRT1
=
(1)( 53.34 )( =
540 )
14.186 ft 3
P1 (14.1)(144 )
Page | 174
Point 2:

V1 14.186
V=
2 = = 1.576 ft 3
rk 9
k 1
V1
( 540=
)( 9 )
1.4 1
=T2 T=
1 1300 0 R
V2

k
V1
= p2 p= (14.1
= )( 9 )
1.4
1 305.6 psia
V2
Point 3:

p3
(1300 ) =
470
=T3 T= 2 1999 o R
p2 305.6
Point 4:

=
Q3 4 mcv (T4 T3 )

100= (1) (0.24) ( T4 - 1999)


T4 =2416 0R

T4
(1.576 ) =
2416
=V4 V=
3 1.905 ft 3
T3 1999

Point 5:

k 1 1.4 1
V4
( 2416 )
1.905
=T5 T=
4 = 1082 0 R
V5 14.186

p3 470
(a) =
rp = = 1.54
p2 305.6

V4 1.905
(b) =
rc = = 1.21
V3 1.5716

1+ c
(c) rk =
c

1+ c
9=
c

Page | 175
c= 0.125 or 12.5%

(d) QA = Q23 + Q3 4 = mcv (T3 T2 ) + 100

= (1) (0.1714) (1900 1300) + 100= 219.8 Btu

QR = (1)( 0.1714 )( 540 1082 ) =


mcv (T1 T5 ) = 92.9 Btu

W 219.8 92.9
=
e = = 0.5773 or 57.73%
QA 219.8

W W 126.9 ( 778 )
=
mep = = = 54.37 psi
Vd V1 V2 (14.186 1.576 )(144 )
2. An ideal dual combustion cycle operates on 454 g of air. At the beginning of
compression the air is at 96.53 kPa, 43.3 0C. Let rp=1.5, rc=1.60, and rk=11.
Determine (a) the percentage clearance, (b) p,V, and T at each corner of the cycle,
(c) QA, (d) e, and (e) mep.

Solution

m= 0.454 kg of air
p1=96.53 kPa
T1= 43.3 +273 = 316.3 K
rp=1.5
rc=1.6
rk=11

1+ c
(a) rk =
c
1+ c
11 =
c
c= 0.10 or 10%

Page | 176
=
(b) V1
mRT1
=
( 0.454 )( 0.28708)( 316.3
=
) 0.4271 m3
P1 ( 96.53)
V1 0.4271
V=
2 = = 0.03883 m3
rk 11

k 1
V
( 316.3)(11)
k 1 1.4 1
=
T2 T1 1=
T=
1rk
2
V

k
V1
=p2 p1 = 1 [ rk ]
p= [11]
( 96.53)=
k 1.4
2770.8 kPa
V2

=
p3 p=
2 rp ( 2770.8)(1.5
= ) 4156.2 kPa

p3
(825.4 )
4156.2
=T3 T=
2 = 1238.1 K
p2 2770.8

=
V4 V=
3 rc ( 0.03883)(1.6
= ) 0.062213 m3

V4
=T4 T=
3 (1238.1=
)(1.6 ) 1981 K
V3

k 1 1.4 1
V4
(1981) =
0.06213
=T5 T=
4 916.2 K
V5 0.4271

T5
( 96.53) =
916.2
=p5 p=
1 279.k kPa
T1 316.3

(c) Q=
A mcv (T3 T2 ) + mc p (T4 T3 )

= (0.454) (0.7186) (1238.1 825.4) + (0.454) (1.0062) (1981 1238.1)

= 474 kJ

(d) QR= mcv (T1 T5 ) = (0.454) (0.7186) (316.3 916.2) =195.7 kJ

= QA QR = 474 -195.7 =278.3 kJ


W

Page | 177
W 278.3
=
e = = 0.5871 or 58.71%
QA 474

W W 278.3
=
(e) mep = = = 716.8 kPa
Vd V1 V2 ( 0.4271 0.03883)
3. An air standard dual cycle has a compression ratio of 20 and a cut off ratio of 1.3.
The pressure ratio during the constant -volume heat addition is 1.2. Determine
the thermal efficiency, amount of heat added and the maximum gas pressure and
temperature when this cycle is operated at 14.7 psia and 700F at the beginning of
the compression.

Solution

rk=20 rc=1.3 rp=1.2 p1=14.7psia T1=70+460=5300R

at point 1

ft lb
RT1
53.34
lb 0 R
( 5300 R ) ft 3
=
v1 = = 13.36 =v5
p1 lb in 2 lb
14.7 2 144 2
in ft
at point2

v1 13.36 ft 3
v=
2 = = 0.668 =v3
rk 20 lb

1 ( rk ) 530 (=
20 )
k 1 1.4 1
=T2 T= 1756.660 R

= 1 ( rk )
p2 p= ( 20 ) 974.45 psia
14.7 =
k 1.4

At point 3
=p3 p= 2 ( rp ) (1.2 ) 1169.34 psia
974.45=

Page | 178
= 2 ( rp )
T3 T= (1.2 ) 21080 R
1756.66=

At point 4

= 3 ( rc )
T4 T= (1.3) 2740.40 R
2108=

ft 3
= 3 ( rc )
v4 v= (1.2 ) 0.8016
0.668=
lb

At point 5

k 1
v4
0.4
0.8016
=T5 T=
4 2740.4 = 889.350 R
v5 13.36

q A= cv (T3 T2 ) + c p (T4 T3 )

q A 0.1714 ( 2108 1756.66 ) + 0.24 ( 2740.4 2108 )


=

=212Btu/lb

qR cv (T1 T5 )
=

0.1714 ( 530 889.35 ) =


qR = 61.6 Btu / lb

w = q A + qR = 212 61.6 = 150.41Btu / lb

w 150.41
=
e = = 70.95%
qA 212

Page | 179
Review Problems

1. An Ideal Otto engine, operating on the hot air standard with k= 1.34, has a
compression ratio of 5. At the beginning of compression the volume is cu ft, the
pressure is 13.75 psia and the temperature is 100 0F. During the constant volume
heating, 340 Btu are added per cycle. Find (a) c, (b) T3, (c) p3, (d) e, and (e) pm.
ans. (a) 25%; (b) 5209 0R; (c) 639.4 psia; (d) 42.14%; (e) 161.2 psi

2. An ideal Otto cycle engine with 15% clearance operates on 0.227 kg/s of air;
intake state is 100.58 kPa, 37.30C. The energy released during combustion is
110 kJ/s. For hot air standard with k=1.32, compute (a) p,V and T at each
corner, (b) W, (c) e, and (d) pm.
Ans. (a) 0.2013 m3/s, 0.02626 m3/s, 596.2 K, 1479.85 kPa, 1136.4 K,
2820.7 kPa, 592.2 K, 191.71 kPa; (b) 52.7 kJ/s; (c) 47.91%; (d) 301.1 kPa

3. An Otto cycle operates with conditions stated as follows; point 1 being at the
beginning of the isentropic compression process. These data apply for the air
standard cycle: p1=101.4 kPa; T1=333.30K, V1=283 liters, rk=5, T3=20000K.
Solve for (a) m (b) p2, V2, T2 (c) p3, (d) p4, T4 (e) Qa (f) Qr (g) e (h) % clearance
Ans (a) 0.3 kg (b) 965 kPa, 0.057m3,6340K (c)3045kPa(d) 320kPaa,
10520K,(e) 294kJ (f) 154kJ (g) 47.42%(h) 25%
4. The compression ratio of an air-standard Otto cycle is 9.5. Prior to the isentropic
compression process, the air is at 100kPa, 350C, and 600cm3. The temperature at
the end of isentropic expansion process is 8000K. Determine (a) the highest
temperature and pressure in the cycle; (b)amount of heat transferred in,in kJ(c)the
thermal efficiency (d) mean effective pressure.
Ans.(a)1969 K ,6072kPa, (b) 0.59 kJ (c) 59.4 % (d) 652 kPa
5. In an ideal Diesel engine compression is from 14.7 psia, 80 0F, 1.43 cu ft to 500
psia. Then 16 Btu/cycle are added as heat. Make computation for cold air
standard and find (a) T2, V2, T3, V3, T4, and p4, (b) W, (c) e and pm, and (d) the hp
for 300 cycle /min.
Ans. (a) 14790R, 0.1152 ft3, 21130R, 0.1646 ft3, 8900R, 24.2 psia; (b) 9.7
Btu; (e) 60.63%, 39.9 psi, (d) 68.6 hp

6. For an ideal Diesel cycle with the overall value of k=1.33, rk=15, rc=2.1, p1=97.9
kPa, find p2 and pm.
Ans. 3589 kPa, 602 kPa

7. An ideal diesel engine has a compression ratio of 20 and uses air as the working
fluid. The state of air at the beginning of the compression process is 95 kPa and
200C. If the maximum temperature is not to exceed 22000K. Determine (a)
thermal efficiency and (b) mean effective pressure.
Ans(a) 63.5% (b) 933 kPa

Page | 180
8. State 1 for a dual combustion engine is p1=1 atm and T1=60.300C; rk=18; at the
end of the constant volume combustion process the pressure is 7695 kPa, rc= 1.5.
Base on 1 kg/cycle of a hot air standard with k= 1.31, determine (a) percentage
clearance, (b) p, V and T at each corner point on the cycle, (c) W, (d) e, and (e)
pm.
Ans. (a) 5.88%; (b) 0.9443 m, 0.05246 m3, 4468 kPa, 816.5 K, 1406.2 K,
0.07869 m3, 2109.3 K, 296.8 kPa, 976.3 K; (c) 803.5 kJ; (d) 57.43%; (e)
900 kPa

9. An ideal dual cycle has a compression ratio of 15 and cut off ratio 1.4. The
pressure ratio during constant volume heat addition process is 1.1. The state of
the air at the beginning of the compression is p1=14.2 psia and T1=750F.
Calculate the cycles net specific work, specific heat addition and thermal
efficiency.

Page | 181
7 Gas Compressors
Operation of Compressors

Figure 18shows a conventional indicator card for a compressor without clearance.


As the piston starts the stroke 4-1, the inlet valve opens and gas is drawn into the cylinder
along the line 4-1. At point 1, the piston starts the return stroke, all valves being closed,
and the gas is compressed along the curve 1-2. At 2, the discharge valve opens and the
compressed gas is delivered to the receiver.
The events of the diagram with clearance are the same as those with no clearance,
except that since the piston does not force all the gas from the cylinder at the pressure p2,
the remaining gas must re-expand to the intake pressure, process 3-4, before intake starts
again. Without clearance, the volume of gas taken into the cylinder is equal to the
displacement volume. As seen from fig. 19 for the diagram with clearance, the volume of
gas drawn into the cylinder V1 V4= Vi and is less than the displacement volume VD.

p1Vi ' = m ' RT1


p1V1 = m1 RT1

Where:
m1=m ' + mc, total mass after admission
m ' = mass of gas drawn in and delivered
mc= clearance or residual gas
Vi ' = volume of gas drawn in, corresponding to mass m '
p1= pressure at start of compression
T1= temperature at start of compression
V1= total volume at end of admission, corresponding to mass m1.

Page | 182
Compressor work

Q = PE + KE + H + W
if PE=0
W= Q KE H
if KE = 0
W= Q H

For reciprocating compressors, KE is negligible,

(a) Adiabatic compression ( KE=0)

W = H = m ' c p (T2 T1 )
m ' kRT1 T2
W= 1
k 1 T1
k 1

m ' kRT1 p2 k
=W 1
1 k p1

k 1

kp1V1 ' p2 k
=W 1

1 k p1

(b) Polytropic compression ( KE=0)

W=Q- H
W m ' cn (T2 T1 ) m ' c p (T2 T1 )
=
W =m ' ( cn c p ) (T2 T1 )
c (k n)
=
W m' v kcv (T2 T1 )
1 n
c k ncv kcv + nkcv
=W m' v
1 n (T2 T1 )

m ' ncv (k 1)
=W
1 n (T2 T1 )

m ' nRT1 T2
=W 1
1 n T1

Page | 183
n 1

m ' nRT1 p2 n
= W 1

1
1 n p

n 1

np1V1 ' p2 n
= W 1

1 n p1

(c) Isothermal compression ( KE=0)

W=Q - H
W=Q
p1
W = p1V1 'ln
p2
p
W = m ' RT1 ln 1
p2

Preferred Compression Curves

The work necessary to drive the compressor decreases as the value of n decreases.
Polytropic compression and values of n less than k are brought about by circulating
cooling water.

Heat Rejected

The heat rejected during compression 1-2 is,


= Q1 2 m1cn (T2 T1 )

Problems

1. A rotary compressor receives 6m3/min. of a gas (R=410 J/kg.K, cp=1.03 kJ/kg.K,


k=1.67) at 105 kPa, 270C and delivers it at 630 kPa. Find the work if compression
is (a) isentropic, (b) polytropic with pV1.4=C and isothermal.

Page | 184
Solution


V1 ' = 6 m3 / min . T1=27 + 273= 300 K p1= 105 kPa
p2= 630 kPa

=

p1 V1 '
m =
(105)(= 6)
5.122kg / min
RT1 ( 0.410 )( 300 )
(a) Isentropic compression

k
k 1


= np V ' p
1
1 1 2
W
1 k p1

1.67 (105 )( 6 ) 630 1.67


1.67 1
kJ
W= 1=1652
1 1.67 105 min

Another solution:

k 1 1.67 1
p2 k
( 300 ) =
630 1.67
=T2 T=
1 615.6 K
p1 105


W = H = m' c p (T2 T1 )

= -(5.122) (1.03) (615.6 300) = -1665 kJ/min

Page | 185
(b) Polytropic compression

n
n 1


= np V ' p
1
1 1 2
W
1 n p1

(1.4 )(105)( 6 ) 630 1.4


1.4 1

1 =1474 kJ/min.
105
=
1 1.4

Another solution

n 1 1.4 1
p2 n
( 300 ) =
630 1.4
=T2 T=
1 500.5 K
p1 105

c p 1.03 kJ
c=
v = = 0.6168
k 1.67 kg .k 0

k n 1.67 1.4 kJ
cn = cv = 0.6168 = 0.4163
1 n 1 1.4 kg .k 0


W = H + Q = m' c p (T2 T1 ) + m' cn (T2 T1 )

= - (5.122) (1.03) (500.5 300) + (5.122) (-0.4163) (500.5 300)

= -1485.29 kJ/min

(c) Isothermal compression

p1
W = p1 V 1'ln
p2
105
= (105 )( 6 ) ln
630

= -1129 kJ/min

2. A centrifugal compressor handles 300 cu ft per minute of air at 14.7 psia and
800F. The air is compressed to 30 psia. The initial speed is 35 fps and the final
speed is 170 fps. If the compression is polytropic with n=1.32, what is the work?

Page | 186
Solution


V1 ' = 300 cfm p1=14.7 psia p2=30 psia T1=80 + 460= 540 R

1=35 fps 2=170 fps


=
m'
p1 V 1'
=
(14.7 )(144 )( 300
=
) 22.05 lbs / min
RT1 ( 53.34 )( 540 )
n 1 1.32 1
p2 n
( 540 ) =
30 1.32
=T2 T=
1 641.9 0 R
p1 14.7

k n 1.4 1.32 Btu


cn = cv = 0.1714 = 0.0429
1 n 1 1.32 lb.R 0



= H m' c p (T2 T1 )

= (22.05) (0.24) (641.9 540) = 539.3 Btu/min


=Q m' cn (T2 T1 )

= (22.05) (-0.0429) (641.9 540) = -96.4 Btu/min


( 22.05) (1702 ) ( 35)
2
m'(22 12 )

=
KE = = 12.2 Btu/min
2k 2 ( 32.174 )( 778 )

Q = PE + KE + H + W


W = Q KE H

Page | 187

=

p1 V1 '
m =
(105)(= 6)
5.122kg / min
RT1 ( 0.410 )( 300 )

W = -96.4 12.2 539.3

= -647.9 Btu/min or -15.28 hp

Volumetric Efficiency

Volume of gas drawn in


Conventional volumetric efficiency=
displacement volume
V1 ' V1 V4
n=
v =
VD VD

Displacement volume VD is the volume swept by the face of the piston in one stroke.

V3
The clearance ratio per cent clearance, c =
VD
1
p n
Then, n v = 1 + c c 2
p1

If the compression process is isentropic, let n=k


VD = D 2 LN
4

Where:
D= diameter of piston
L= Length of stroke
N= number of cycle completed per minute
N= (n) (1) (number of cylinders), for single acting compressors
N= (n) (2) (number of cylinders), for double acting compressors
n= compressor speed, revolution per min., rpm

A single acting compressor makes one complete cycle in one revolution

Page | 188
A double acting compressor makes two complete cycles in one revolution.

Free Air

Free air is air at normal atmospheric conditions in a particular geographical location.

Problems

1. A twin cylinder, double acting compressor with a clearance of 5% handles 20


m3/min. of nitrogen from 100 kPa, 370C to 725 kPa. Compression and expansion
are polytropic with n= 1.30. Find (a) the work, (b) the heat rejected and (c) the
bore and stroke for 150 rpm and L/D = 1.30.

Solution


V1 ' = 20 m3 / min p1= 100 kPa p2=725 kPa T1= 37 + 273 = 310 K
c= 5% n= 150 rpm L/D= 1.30

n
n 1


= np V ' p
1
1 1 2
(a) W
1 n p1

Page | 189
(1.3)(100 )( 20 ) 725 1.3
1.31
kJ
W= 1 =5023
1 1.30 100 min

1
p n
(b) n v =1 + c c 2
p1
1
725 1.3
= 1 + 0.05 ( 0.05 ) =
0.8205
100


V 1' 20 m3
V=
D = = 24.38
n v 0.8205 min

V1 = VD + V3 = VD + cVD = VD (1 + c)
m3
V 1 = (24.38) (1+ 0.05) =25.60
min

=m =

p1 V 1' (100 )( 25.6
=
) 27.83
kg
RT1 ( 0.2967 )( 310 ) min

n 1 1.31
p2 n
( 310 ) =
725 1.3
=T2 T=
1 489.7 K
p1 100

k n 1.399 1.30 kJ
cn = cv = 0.7442 = 0.2456
1 n 1 1.30 kg .K 0


Q1 2 m'1 cn (T2 T1 )
=

= (27.83) (-0.2456) (489.7 310)

kJ
= -1228
min


=
(c) V D = D 2 LN D 2 (1.3 D )(150 =
)( 2 )( 2 ) 612.6 D3
4 4

24.38 = 612.6 D3

D= 0.3414 m or 34.14 cm
L= (1.30) (34.14) = 44.38 cm

Page | 190
2. A single acting air compressor operates at 150 rpm with initial condition of air
at 97.9 kPa, 270C and discharges the air at 379 kPa to a cylindrical tank. The bore
and stroke are 355 mm and 381 mm, respectively with a percentage clearance of
5%. If surrounding air are at 100 kPa and 200C while the compression and
expansion processes are pV1.3= C. Determine (a) Free air capacity in m3/s. (b)
Power of the compressor in kW. (ME Board problem Oct. 1986)

Solution

c=5% D=355 mm L=381 mm n=150 rpm


p1=97.9 kPa T1=300 K

1 1
p n 379 1.3
(a) n v =1 + c c 2 =1 + 0.05 ( 0.05 ) = 0.9084
p1 97.9
2 m3
= VD = ( 0.355) ( 0.381=
)(150 ) 5.657
2
D LN
4 4 s


m3
= V D ( nv )
V 1' = ( 0.9084=
)( 5.657 ) 5.139
min

p T

97.9 293 m2 m3
=V o V=1'
1

o
5.139 =
100 300 4.918 or 0.082
p2 T1 min s

n
n 1


np V 1
' p
=(b) W 1 1
2
1 n p1

(1.3)( 97.9 )( 5.144 ) 379 1.3


1.31
kJ
= 1 =800.3 or 13.34 kW
1 1.30 97.9 min

Page | 191
3. A single acting air compressor with a clearance of 6% takes in air at
atmospheric pressure and a temperature of 850F, and discharges it at a pressure of
85 psia. The air handled is 0.25 cu ft per cycle measured at discharge pressure. If
the compression is isentropic, find (a) piston displacement per cycle, and (b) air
hp of compressor if rpm is 750. (ME Board Problem march 1978)

Solution

p1=14.7 psia p2= 85 psia V2= 0.25 ft3/cycle T1=85 + 460 = 5450R

k 1 1.4 1
p2 k
( 545) =
85 1.4
=
(a) T2 T=
1 900 0 R
p1 14.7

=
m'
p2V2
=
( 85)(144 )( 0.25
=
) 0.06374 lb/cycle
RT2 ( 53.34 )( 900 )

=V1
m ' RT1
=
( 0.06374 )( 53.34 )(=
545 )
0.8754 ft 3 / cycle
p1 (14.7 )(144 )
1 1
p k 85 1.4
n v =1 + c c 2 =1 + 0.06 ( 0.06 ) = 0.8499
p1 14.7

V1 ' 0.8754 ft 3
V=
D = = 1.030
n v 0.8499 cycle


ft 3
= V D ( nv )
(b) V 1' = ( 0.8754
= )( 750 ) 656.6
min

k 1

(1.4 )(14.7 )( 656.6 ) 85 1.4
1.4 1

kp1 V 1' p2 k
= = 1 1 = 96 hp
( 33000 )(1 1.4 ) 14.7
W
1 k p1

Page | 192
4. A single acting compressor has a volumetric efficiency of 87% and operates at
500 rpm. It takes in air at 100 kPa and 300C and discharges it at 600 kPa. The air
handled is 6 cu m per min measured at discharge condition. If the compression is
isentropic, find (a) piston displacement per stroke in cu m, and (b) mean effective
pressure in kPa. (ME Board Problem April 1983)

Solution


p1=100 kPa p2=600 kPa V 2 = 6m3 / min

T1= 30 + 273 = 303 K

1 1
p2
( 6 ) =
k
600 1.4
=
(a) V 1' V=
2 21.58 m3 / min
p1 100


V 1' 21.58 m3
V=
D = = 24.8
nv 0.87 min

m3
24.8 3
= min = 0.0496 m
strokes strokes
500
min

k 1

(1.4 )(100 )( 21.58) 600 1.4
1.4 1
kp1 V 1' p2 k


= = 1 1
100
(b) W
1 k p1 (1 1.4 )

kJ
= -5049.26
min

W 5049.26
= =
mep = 203.6kPa
VD 24.8

Page | 193
5. A compressor is to be designed with 6% clearance to handle 500 cfm of air at
14.7 psia and 700F, the state at the beginning of compression stroke. The
compression is isentropic to 90.3 psig.
(a) What displacement in cfm is necessary?
(b) If the compressor is used at an altitude of 6000 ft and if the initial temperature
and discharge pressure remain the same as given in (a), by what percentage is
the capacity of the compressor reduced?
(c) What should be the displacement of a compressor at the altitude of 6000 ft to
handle the same mass of air as in (a)?

Solution


p1= 14.7 psia p2= 90.3 + 14.7 =105 psia V1 = 500 ft 3 / min
0
T1= 70 + 460 =530 R

1 1
p k 105 1.4
(a) n v =1 + c c 2 =1 + 0.06 ( 0.06 ) = 0.8156
p1 14.7

V1 500 ft 3
V=
D = = 613
n v 0.8156 min

(b) Barometric pressure at 6000 ft =11.78 psia or 23.99 in Hg


New intake pressure, p1N= 11.78 psia
New discharge pressure, p2N= 90.3 + 11.78 = 102.08 psia
New volumetric efficiency,

1 1
p k 102.08 1.4
n vn = 1 + c c 2 =1 + 0.06 ( 0.06 ) = 0.7795
p1 11.78

ft 3
=
New capacity, V 1N ' = V D ( nv ) ( 0.7795
= )( 613) 477.8
min

500 477.8
Percentage decreased in capacity = = 4.44%
500

Page | 194
(c) p1=14.7 psia p1 at 6000 ft = 11.78 psia

V 1' =500 cfm T1 at 6000 ft = 530 0R T1= 530 0R

V1 at 6000 ft= capacity to handle the same mass of air as in (a)


VD at 6000 ft= displacement volume to handle the same mass of air as in
(a)

=
p1 V 1'
m' =
( p1 at 6000 ft )( V1 at 6000ft )
RT1 ( R )(T1 at 6000 ft )

V1' at 6000 ft=
(14.7 )( 500 ) = 623.9 ft 3 / min
(11.78)

V D at 6000 ft=
( 623.9 ) = 800.4 ft 3 / min
( 0.7795)
Compressor Efficiency

ideal work
In general, efficiency =
actual work

A. Mechanical Efficiency

The mechanical efficiency of a compressor is

indicated work of compressor, W1


nm =
Brake work of compressor, WB

If the compressor is driven by a steam or internal combustion engine, the


mechanical efficiency of the compressor system is

indicated work of compressor


nms =
indicated work of driving engine

B. Compression Efficiency

Adiabatic compression efficiency is

adiabatic ideal work


nc =
indicated work of compressor

Page | 195
Isothermal compression efficiency is

isothermal ideal work


nt =
indicated work of compressor

Polytropic compression efficiency is

polytropic ideal work


np =
indicated work of compressor

C. Overall Efficiency

Overall efficiency is

no = (mechanical efficiency) (compression efficiency)

Adiabatic overall efficiency is

WI adiabatic ideal work


=noc (=
nm )( nc )
WB WI

adiabatic ideal work


noc =
WB

Isothermal overall efficiency is

WI isothermal ideal work


=not (=
nm )( nt )
WB WI

isothermal ideal work


not =
WB

Polytropic overall efficiency is


WI polytropic ideal work
= nop (= nm ) ( n p )
WB WI

polytropic ideal work


nop =
WB

Indicated work is the work done in the cylinder.


Brake work or shaft work is the work delivered at the shaft.

Page | 196
Adiabatic compression efficiency is the compression efficiency commonly
use. Compression efficiency, therefore, would mean adiabatic compression
efficiency.

Problems

1. A two cylinder, single acting air compressor is directly coupled to an electric


motor running at 1000 rpm.
Other data are as follows:
Size of each cylinder, 150 mm x 200 mm
Clearance volume, 10% of displacement
Exponent (n) for both compression and re expansion process, 1.6
Air constant, k=1.4
Air molecular mass, 29

Calculate:
(a) The volume rate of air delivery in terms of standard air for a delivery pressure
of 8 times ambient pressure under ambient conditions of 300 K and 1 bar.
(b) Shaft power required if the mechanical efficiency is 81%
(ME Board Problem 1984)

Solution

p2
p1=1 bar = 100 kPa =8
p1

k 1

(1.395)(8.48)( 0.2599 )( 299.7 ) 210.27 1.395
1.395 1
k m RT1 p2 k
= = 1 1
101.35
(a) W isen

1 k p1 (1 1.395)

m 3
= = ( 0.150 ) ( 0.200 )( 2 =
)(1000 ) 7.069
2
VD D 2 LN
4 4 min

Page | 197
1
p n 1
n v = 1 + c c 2 =1 + 0.10 ( 0.10 )( 8 )1.6 = 0.7332
p1

m3 m3
=V 1' = V D ( nv ) ( 0.7332=
)( 7.069 ) 5.183 or 0.0864
min s

n 1


np1 V 1' p2 n (1.6 )(100 )( 0.0864 ) 8 1.61.61 1
=
(b) W = 1 ( )

1 n p1 (1 1.6 )

= -27.21 kW

27.21
Shaft power = = 33.59 kW
0.81

2. A 12 x 14 in. double acting air compressor with 5.5 % clearance operates at 150
rpm, drawing air at 14.5 psia and 850F and discharging it at 62 psia, the
compression and expansion processes are polytropic with n = 1.34, Determine (a)
the volume of free air handled per minute, if atmospheric conditions are 820F and
14.7 psia, (b) the heat ejected, (c) the indicated work of the compressor if the
compression efficiency is 87%, and (d) the ideal work.

Solution

po =14.7 psia To= 820F +460 = 542 0R p1= 14.5 psia


T1= 85 + 460 = 545 0R

1 1
p n 62 1.34
(a) n v =1 + c c 2 =1 + 0.055 ( 0.055 ) = 0.8923
p1 14.5
12 14
2

= = ( 2= )(150 ) 274.9 cfm
2
VD D LN
4 4 12 12

Page | 198


=V 1' =V D ( nv ) ( 0.8923= )( 274.9 ) 245.3 cfm


V 1' ( p1 )(To ) ( 245.3)(14.5 )( 542 )
=V o = = 240.6 cfm
( po )(T1 ) 14.7 ( 545 )


(b) V 1 = V D + V 3 = V D + cV D = V D (1 + c)

= (274.9) (1+0.055) = 290.02 cfm

=

p1 V 1
m' =
(14.5)(144 )( 290.02
=
) 20.83
lb
RT1 ( 53.34 )( 545) min
n 1 1.34 1
p2 n
( 545) =
62 1.34
=T2 T=
1 788 0
R
p1 14.5

k n 1.4 1.34 Btu


cn = cv = 0.1714 = 0.3025
1 n 1 1.34 lb.R 0


Q1 2 m'1 cn (T2 T1 )
=

= (20.83) (-0.03025) (788 545)= - 153.1 Btu/min

k 1

(1.4 )(14.5)(144 )( 245.3) 62 1.4
1.4 1
kp1 V 1' p2 k
= = 1 1
14.5
(c) W isen
1 k p1 (1 1.4 )( 778)

= - 1186 Btu/min or -27.97 hp

adiabatic ideal work


nc =
indicated work of compressor

27.97
Indicated work = = 32.15 hp
0.87

n 1
1.34 1

=
np1 V 1' p2 n

(1.34 )(14.5 )(144 )( 245.3 ) 62 1.34
1
= 1
14.5
(d) W
1 n p1 (1 1.34 )( 778 )

= - 1157 Btu/min or -27.29 hp

Page | 199
3. There are compressed 8.48 kg/min of oxygen by a 35.56 x 35.56 cm, double
acting, motor driven compressor operating at 100 rpm. These data apply: p1=
101.35 kPa, t1= 26.7 0C and p2= 310.27 kPa. Compression and expansion are
polytropic with n=1.31. determine (a) the conventional volumetric efficiency, (b)
heat rejected, (c) the work , and (d) the kW input by the driving motor for an
overall adiabatic efficiency of 71%.

Solution

D=L=0.3556 m p1= 101.35 kPa


T1= 26.7 0C +273 =299.7 K p2= 310.27 kPa

m3
= = ( 0.3556 ) ( 0.3556 )( 2=
)(100 ) 7.063
2
(a) V D D 2 LN
4 4 min

=
m' RT1
V 1' =
(8.48)( 0.2599 )( 299.7
=
) 6.517 m3 / min
p1 (101.35)

V 1' 6.517
nv =
= = 0.9227 or 92.27%
VD 7.063

n 1 1.311
p2 n
( 299.7 ) =
310.27 1.31
=
(b) T2 T=
1 390.5 K
p1 101.35

k n 1.395 1.31 kJ
cn = cv = 0.6595 = 0.1808
1 n 1 1.31 kg .K
1
p n
nv =1 + c c 2
p1
1
310.27 1.31
0.9227= 1+ c -c
101.35

c= 0.0573 or 5.73%

Page | 200

V 1 = V D + V 3 = V D + cV D = V D (1 + c)
m3
= (7.063) (1+0.0573) = 7.468
min

=

p1 V 1
m1 =
(101.35)( 7.468
=
) 9.717
kg
RT1 ( 299.7 )( 0.2599 ) min


Q1 2 m1 cn (T2 T1 ) = (9.717) (-0.1808) (390.5 299.7)
=

kJ
= -159.5
min

n 1

(1.31)(8.48)( 0.2599 )( 299.7 ) 310.27 1.31
1.311

n m RT1 p2 n
= = 1 1
101.35
(c) W
1 n p1 (1 1.31)

kJ
= -846.1 or -14.1 kW
min

k 1

(1.395)(8.48)( 0.2599 )( 299.7 ) 210.27 1.395
1.395 1

k m RT1 p2 k
= = 1
101.35
(d) W isen 1

1 k p1



(1 1.395)

kJ
= -869.5 or -14.29 kW
min

adiabatic ideal work


noc =
WB

14.49
Brake work = = 20.41 kW
0.71

Work input by the driving motor = 20.41 kW

Multistage Compression

Multistaging is simply the compression of the gas in two or more cylinders in


place of a single cylinder compressor. It is used in reciprocating compressors in order to
(1) save power, (2) limit the as discharge temperature, and (3) limit the pressure
differential per cylinder.

Page | 201
The figures above show the events of the conventional cards of a two stage machine,
with the high pressure (HP) superposed on the low pressure (LP). Suction in the LP
cylinder begins at A and the volume V1 ' is drawn in. Compression 1-2 occurs and the gas
is discharged along 2-B. The discharged gas passes through the intercooler and is cooled
by circulating water through the intercooler tubes. Conventionally, it is assumed that the
gas leaving the intercooler and entering the HP cylinder has the same temperature as it
had upon entering the LP cylinder (T3= T1). The gas is then drawn into the HP cylinder
along E-3, is compressed 3-4, and finally discharged from the compressor unit 4-F. The
residua; gas always remains in each cylinder because of clearance and must re-expand F-
E (HP cylinder) and B-A (LP cylinder).

W= W of the low pressure cylinder + W of the high pressure cylinder

n 1
n 1


p4
1
nm ' RT1 p2 n nm ' RT3
n
=
1 + p3
1 n p1 1 n

Page | 202
It is common practice to adjust the operation of multistage compressors so that
approximately equal works are done in the cylinders, a practice that results in minimum
work for compressing a given quantity of a gas. Thus, for the particular case of T1= T3
and of p2= p3= px, we have the work of the LP stage equal to that of the HP stage, or

n 1
n 1

nm ' RT1 px n nm ' RT3
p n

= 1 1
4

1 n p1
1 n px

px = p1 p4

Where: px= intermediate pressure for minimum work

Since the work of each cylinder is the same, the total work for the two stage
machine is twice the work in each cylinder, or

n 1
n 1

2nm ' RT1 p2 n 2 nm ' RT1
p n

=W = 1 1
4

1 n p1 1 n p1

A pressure drop in the intercooler could be spread on each side of this ideal value.

pressure drop
p=
2 px +
2
pressure drop
p=
3 px
2

Heat Transferred in Intercooler

The heat rejected in the intercooler is,


= QIC m ' c p (T3 T2 )

Where m ' is the mass of gas passing through the intercooler ( also the mass
drawn in by the LP cylinder and delivered by the HP cylinder).

Page | 203
Problems

1. There are compressed 11.33 m3/min of air from 26.7 0C, 103. 42 kPa to 827.36
kPa. All clearance are 8%.
(a) Find the isentropic power and piston displacement required for a single stage
compression.
(b) Using the same data, find the minimum ideal work for two stage
compression when the intercooler cools the air to the initial temperature.
(c) Find the displacement of each cylinder for the conditions of part (b)
(d) How much heat is exchanged in the intercooler?
(e) For an overall compression efficiency of 78%, what driving motor output is
required?
Solution


V1 ' = 11.33m3 / min p1= 103.42 kPa p2= 827.36 kPa
T1= 26.7 +273 =299.7 K

k 1

kp1 V 1 p2 k


=(a) W 1
1 k p1

(1.4 )(103.42 )(11.33) 827.36 1.4


1.4 1

1
(1 1.4 ) 103.4

kJ
= -3327 or -55.45 kW
min

1 1
p n 827.36 1.4
n v = 1 + c c 2 =1 + 0.08 ( 0.08 ) =0.7267
p1 103.42


V 1 11.33 m3
V=
D = = 15.59
n v 0.7267 min

Page | 204
b)

p1= 103,42 kPa p4= 827.36 kPa

=px =
p1 p4 (103.42 )(827.36
= ) 292.52 kPa

k 1
1.4 1

=
kp1 V 1 p2 k

(1.4 )(103.42 )( 11.33 ) 292.52 1.4
1
W LP = 1
1 k p1 (1 1.4 ) 103.42

kJ
= -1416 or -23.6 kW
min

Total work = (2) (23.6) = -47.2 kW

1 1
p n 292.52 1.4
(c) n v = 1 + c c 2 =1 + 0.08 ( 0.08 ) = 0.9119
p1 103.42

V 1' 11.33 m3
= =
V DLP = 12.42
n v 0.9119 min

=

p1 V 1'
m1 =
(103.42 )(11.33
=
) 13.62
kg
RT1 ( 0.28708)( 299.7 ) min

=
V3 =

m' RT3 (13.62 )( 0.28708)( 299.7
=
) 4.006 m3 / min
p3 ( 292.52 )

V 3 4.006 m3
= =
V DHP = 4.393
nv 0.9119 min

Page | 205

=
(d) Q IC m' c p (T3 T2 )
kJ
= (13.62) (1.0062) (299.7 403.4) = -1421
min

47.2
(e) Output of driving motor= = 60.5 kW
0.78

2. A two stage, double acting compressor is to deliver 90 lb/min of air from 14.3
psia and 900F to a final pressure of 185 psia. The normal barometer is 29.8 in. Hg
and the temperature is 800F. The pressure drop in the intercooler is 3 psi and the
temperature of the air at the exit of the intercooler is 900F, the speed is 210 rpm
and pV1.34=C during the compression and expansion. The clearance is 5% for both
cylinders. The temperature of the cooling water increase by 180F. Find (a) the
volume of free air, (b) the discharge pressure of the low pressure cylinder for
minimum work, (c) the temperature at discharge from both low pressure and high
pressure cylinders, (d) the mass of cooling water to be circulated about each
cylinder and through the intercooler, (e) the work, and (f) if, for the low pressure
cylinder, L/D= 0.68 and if both cylinders have the same stroke, what should be
the cylinder dimensions?

Solution

m ' =90 lb/min po= (29.8) (0.491) = 14.63 psia To= 80 +460= 540
0
R
p1=14.3 psia T1= 90 + 460= 5500R p4=185 psia

=
(a)
m' RT1
V 1' =
( 90 )( 53.34 )(=
550 )
1282 cfm
p1 (14.3)(144 )

Page | 206

V 1' ( p1 )(To ) (1282=
)( 540 )(14.3)
=V o = 1230 cfm
( po )(T1 ) 14.63 ( 550 )

=
(b) px =
p1 p4 (14.3)(=
185 ) 51.4 psia
3
p2= 51.4 + = 52.9 psia
2

3
(c) p3=51.4 - = 49.9 psia
2
n 1 1.34 1
p2
( 550 ) =
n
52.9 1.34
=T2 T=
1 767 0 R
p1 14.3
n 1 1.34 1
p4 n
( 550 ) =
185 1.34
=T1 T=
2 767 0 R
p3 49.9

k n 1.4 1.34 Btu


(d) cn = cv = 0.1714 = 0.0302
1 n 1 1.34 lb.R 0

Low pressure cylinder

1
52.9 1.34
1 + 0.05 ( 0.05 )
nv = =
0.9173
14.3


V 1' 1282
V=
D = = 1398 cfm
nv 0.9173


V 1 = V D + V 3 = V D + cV D = V D (1 + c) = (1398) (1+ 0.05) = 1467 cfm

=
p1 V 1
m1 =
(14.3)(144 )(1467
=
) 103 lb/min
RT1 ( 53.34 )( 550 )

Q1=
2 m1 cn (T2 T1=
) (103)( 0.0302 ) (767 550) = -675 Btu/min

Page | 207
Heat to water = heat from air


m w ( cw )( tw ) =
Q1 2

Btu
675

min lb
=mw = 37.5
Btu
(18 F )
min
1
lb F

High pressure cylinder

Intercooler

Btu
Q=
IC m' c p (T3 T=
2) ( 90 )( 0.24 ) (550 767)
= 4687
min

4687 lb
Mass of cooling water = = 260.4
18 min

(e) Low pressure cylinder

n 1


n m' RT1 p2 n
=W LP 1

1 n p1

(1.34 )( 90 )( 53.34 )( 550 ) 52.9 1.34


1.34 1

1
( 778)(1 1.34 ) 14.3

= -5265 Btu/min = -124.2 hp

Total work, WT= (2) (-124.2) = -248.4 hp

(f) Low pressure cylinder


= = ( D ) ( 0.68D )( 2=
)( 210 ) 224.3 D3 cfm
2
VD D 2 LN
4 4

224.3 D3=1398

Page | 208
D= 1.84 ft or 22.08 in
L= (1.84) (0.68) =1.25 ft or 15.01 in

High pressure cylinder

=
m' RT3
V3 =
( 90 )( 53.34 )(=
550 )
371.17 cfm
p3 ( 49.4 )(144 )

V 3' 371.17
V=
D = = 404.63 cfm
nv 0.9173


= = ( D ) (1.25)( 2=
)( 210 ) 412.3 D 2 cfm
2
VD D 2 LN
4 4

412.3 D2= 404.63

D= 0.99ft or 11.89 in.


L= 15.01 in.

Three Stage Compression

Page | 209
Conditions for minimum work

1) W LP = WIP = WHP

2) T5 = T3 = T1

n 1
n 1
n 1

nm ' RT1 px n
nm ' RT3 p y n
nm ' RT5 p6 n
= 1 = 1 1
1 n p1 1 n p x 1 n p y

px py p6
= =
p1 px py
1
px = ( p y p1 ) 2 (1)
1
p y = ( px p6 ) 2 (2)

Solving equations (1) and (2) simultaneously,

px = 3
p12 p6 and p y = 3
p1 p6 2

n 1
n 1

3nm ' RT1 p2 n 3nm ' RT1
p 3n

=W = 1 1
6

1 n p1 1 n p1

Problem

1. Air is compressed from 103.4 kPa and 320C to 4136 kPa by a three stage
compressor with value of n= 1.32. Determine (a) the work per kg of air (b) the
heat rejected in the intercoolers.

Solution

Page | 210
m ' = 1 kg p1= 103.4 kPa p6= 4136 kPa
0
T1= 32 C +273 = 305 K

= = (103.4 ) (=
4136 )
2
(a) px 3
p12 p6 3
353.6 kPa
n 1

3nm ' RT1 p2 n
=W 1
1 n p1

3 (1.32 )(1)( 0.28708 )( 305 ) 353.6 1.32


1.32 1

1
(1 1.32 ) 103.4

= -376.2 kJ

(b) T3= T1= 305 K

n 1 1.32 1
p2 n
( 305) =
353.6 1.32
=T2 T=
1 411 K
p1 103.4

Heat rejected in the first intercooler,

m ' c p (T3 T2 ) =
QIC = (1)(1.0062 ) (305 411) =
106.7 kJ

Total heat rejected = (2) (-106.7) = -216.4 kJ

Page | 211
Review Problems

1. A reciprocating compressor handles 1000 cfm of air measured at intake where p1=
14 psia and t1= 800F. The discharge pressure is 84 psia. Calculate the work if the
process of compression is (a) isothermal, (b) polytropic with n=1.25, and (c)
isentropic.
Ans. (a) -109.5 hp; (b) -131.7 hp; (c) -143 hp

2. A twin cylinder, double acting, compressor with a clearance of 5% draws in


oxygen at 450 kPa, 170C and discharges it at 1800 kPa. The mass flow rate is 20
kg/min, compression and expansion are polytropic with n=1.25. Find (a) work, (b)
the heat transferred, and (c) the bore and stroke for 100 rpm and L/D= 1.20
Ans. (a) -40.23 kW; (b) -829 kJ/min; (c) 21.71 x 25.76 cm

3. A double acting compressor with c=7% draws 40 lb per minute of air at 14.7
psia and 800F and discharges it at 90 psia. Compression and expansion are
polytropic with n= 1.28, find (a) the work, (b) the heat rejected, and (c) the bore
and stroke for 90 rpm and L/D =1.25.
Ans. (a) 77.68 hp; (b) -1057 Btu/min; (c) 18.96 x 23.70 in.

4. A 14 x 12 in., single cylinder, double acting air compressor with 5.5%


clearance operates at 125 rpm. The suction pressure and temperature are 14 psia
and 1000F, respectively. The discharge pressure is 42 psia. Compression and
expansion process are polytropic, with n=1.30. Determine (a) the volumetric
efficiency, (b) the mass and volume at suction conditions handled each minute, (c)
the work, (d) the heat rejected, (e) the indicated air hp developed if the polytropic
compression efficiency is 75%, and (f) the compression efficiency.
Ans. (a) 92.7%; (b) 247.8 cfm, 16.72 lb/min; (c) -18.93 hp; (d) -175.7 Btu/
min; (e) -25.24 hp; (f) 77.42%

5. From a test of an air compressor driven directly by a steam engine , the following
data and results were obtained: capacity, 800 cfm; suction at 14.7 psia; discharge
at 110 psia; indicated work of the compressor, 155 hp; indicated work of steam
engine, 172 hp. Calculate (a) the compression efficiency and (b) the overall
efficiency.
Ans. (a) 90.06%; (b) 81.16%

6. An air compressor with a clearance of 4% compresses 14.73 m3/min of air from


97 kPa, 270C to 462 kPa. If the overall adiabatic efficiency is 61 %, determine the
indicated horsepower of the directly connected driving steam engine.
Ans. 91.39 hp

7. Methane is compressed in a two stage, double acting compressor which is


electrically driven at 165 rpm. The low pressure cylinder (30 x 35.5 cm) receives
6.86 cu m per minute of air at 96.53 kPa,43.30C, and the high pressure cylinder

Page | 212
(20.3 x 35.5 cm) discharges the methane at 717.06 kPa. The isothermal overall
efficiency is 74%. Find nv, and the kW output of the motor.
Ans. 80.02%, 90.86%
8. A two stage compressor with a clearance of 6% receives 80 lb/min of air at 14
psia and 850F and delivers it at 120 psia. The compressions are polytropic with
n=1.30, and the intercooler cools the air back to 850F. Find (a) the work, (b) the
heat transferred in the various processes, (c) the work for a single stage
machine, (d) the corresponding percentage saving for the two stage machine,
and (f) the mass of water to be circulated through the intercooler if its temperature
rise is 15% 0F
Ans. (a) -171.6 hp; (b) -802.8 Btu/min; (c) -2938 Btu/min; (d) -196 hp; (e)
12.45%; (f) 196 lb/min

Page | 213
8 Brayton Cycle
Operation of a simple gas turbine power plant

OPEN CYCLE
CLOSED CYCLE

Air continuously enters the compressor 1. After compression, it enters the


combustors, some of it going around the outside of the combustion chamber proper and
the remainder furnishing oxygen for burning the fuel, which is continuously injected into
the combustion chamber. Because of their temperature rise, the gases expand and enter
the turbine in state 3. After expansion through the turbine, the exhaust to the atmosphere
is in some condition 4. In an ordinary power plant arrangement, the work of the turbine
Wt is great enough to drive the compressor Wc and deliver brake work WB to drive, say a
generator or propeller; Wt=WB+Wc. An external source of power is needed to start a gas
turbine unit.

1-2: isentropic compression


2-3: constant pressure addition of heat
3-4: isentropic expansion
4-1: constant pressure rejection of heat

Page | 214
Analysis of the Brayton cycle

=
QA mc p (T3 T2 )
QR =
mc p (T1 T4 ) =
mc p (T4 T1 )
W = QA QR = mc p (T3 T2 ) mcv (T4 T1 )

W mc p (T3 T2 ) mc p (T4 T1 )
=
e =
QA mc p (T3 T2 )
(T4 T1 )
e= 1 (1)
(T3 T2 )
1 1
e=
1 k 1
=
1 k 1
r
k k
rp

V1
Where rk= , the compression ratio
V2
p
rp= 1 , the pressure ratio
p2
Derivation of the formula for e

Process 1-2:

k 1
k 1
T2 V1 p1 k
= =

T1 V2 p2
T2 = T1rkk 1 (2)
k 1
rkk 1 = rp k (3)

Process 3-4:

k 1 k 1
T3 p3 k p k
= = 2
T4 p4 p1

T3 = T4 rkk 1 (4)

Page | 215
Substituting equations (2) and (4) in (1).

Total compressor work, WC= Q H


WC =
mc p (T2 T1 )
Total turbine work, W=
t Q H
Wt =
mc p (T4 T3 )
= Wt mc p (T3 T4 )
Net work, W or WB= Wt Wc

Back work ratio- the fraction of the turbine work used to drive the compressor.

Wc
back work ratio=
Wt

Problems

1. The intake of the compressor of an air standard Brayton cycle is 40,000 cfm at
15 psia and 900F. The compression ratio, rk=5 and the temperature at the turbine
inlet is 14400F. The exit pressure of the turbine is 15 psia. Determine the net
work, thermal efficiency and the mean effective pressure.

V1= 40000 cfm p1=15 psia T1=5500R


T3=19000R p4=15 psia

V1
rk = =5
V2

=m =

p1 V 1 (15)(144 )( 40000
=
) 2945 lb/min
RT1 ( 53.34 )( 550 )

Page | 216
Point 1:

V1 40000
v=
1 = = 13.58 ft 3 / lb
m 2945

Point 2:

v1 13.58
v=
2 = = 2.72 ft 3 / lb
rk 5

p2 = p1rkk = (15 )( 5 ) =142.8 psia


1.4

T2 = T1rkk 1 = ( 550 )( 5 )
1.4 1
=1047 0 R
Point 3:

T3
( 2.72 )
1900
= v3 v=
2 = 4.94 ft /lb
3

T2 1047
Point 4:

1 1
p k 142.8 1.4
=
v4 = v3 3 (=
4.94 ) 24.7 ft 3 /lb
p4 15
k 1 1.4 1
v3
(1900 )
4.94
=T4 T=
3 = 998 0 R
v4 24.7

Compressor work, wc =
c p (T2 T1 )

= - (0.24) (1047 -550) = -119.3 Btu/lb

Turbine work,=
wt c p (T3 T4 )
= (0.24) (1900 998) = 216.5 Btu/lb

Net work, wB= wt wc = 216.5 119.3 = 97.2 Btu/lb

=
( 97.2 )( 2945) = 6749.68 hp
42.41

Heat added, q A= c p (T3 T2 )= (0.24)(1900 1047)= 204.7 Btu/lb

w 97.2
=
e =
q A 204.7

Page | 217
w w 97.2 ( 778 )
p= = = = 23.89 psi
m
vD v4 v2 ( 24.7 2.72 )(144 )
2. There are required 2238 kW net from a gas turbine unit for pumping of crude oil
from the North Alaskan slope. Air enters the compressor section at 99.975 kPa,
278 K; the pressure ratio rp=10. The turbine section receives the hot gases at 1111
K. Assume the closed Brayton cycle, and find (a) the required air flow and (b) the
thermal efficiency.

Solution

WB= 2238 kW p1= 99.975 kPa T1= 278 K


p
T3= 1111 K rp= 2 =10
p1

Point 2:

= 1 ( rp )
p2 p= ( 99.975=
)(10 ) 999.75 kPa
k 1
p k 1.4 1
T2== T1 2 (=
278 )(10 ) 1.4 536.7 K
p1
Point 4:

k 1 1.4 1
p4 k
)
1 1.4
T=
4 = T3 (1111
= 575.4 K
p3 10

q A= c p (T3 T2 )= (1.0062)(1111 536.7)= 577.9 kJ/kg

qR =
c p (T1 T4 ) =
(1.0062)(278 575.4) =
299.2 kJ/kg

w= 577.9 299.2 =278.7 kJ/kg

Page | 218

(a) m (278.7)= 2238


m = 8.03 kg/s

w 278.7
(b) =
e= = 0.4823 or 48.23%
qA ( 577.9 )
3. A gas turbine power plant operating on a ideal Brayton cycle has a pressure ratio
of 8. The gas temperature is 3000K at the compressor inlet and 13000K at the
turbine inlet. The inlet pressure at the compressor and exit pressure at the turbine
is at standard atmospheric pressure. For an air standard assumptions, determine
(a) the gas temperature at the exits of the compressor and the turbine, (b)the back
work ratio, and thermal efficiency.

Solution:

p2
T1= 3000K p1=101.325kPaa =
rp = 8 T3=13000K
p1
(a) At point 2:

k 1
p2 k 1.4 1
=T2 T=
1 =
300 [8] 1.4 543.430 K
p1

At point 4:
k 1 1.4 1
p4 k 1 1.4
=T4 T=
3 =
1300 717.660 K
p3 8

Page | 219
(b) Compressor work
kJ
c p (T2 T1 ) =
wc = 1.0062 ( 543.43 300 ) =
244.94
kg
Turbine work
kJ
wt= c p (T3 T4 )= 1.0062 (1300 717.66 )= 585.95
kg
Back work ratio

wc 244.94
r=
bw = = 0.418
wt 585.95
Heat added
kJ
qa= c p (T3 T2 )= 1.0062 (1300 543.43)= 761.26
kg
Net work
kJ
wnet = wt wc = 585.95 244.94 = 341.01
kg
Thermal efficiency

wnet 341.01
=e = = 44.8%
qa 761.26

Page | 220
Review Problems

1. The turbine section of a Brayton cycle gas turbine receives the hot compressed air
at 150 psia, 21000R, expands it to 15 psia and develops a gross output of 15,000
hp. Air enters the compressor section at 15 psia, 5000R. Determine (a) mass of air
required, lbs, (b) compressor power required, (c) net power output, and (d) cycle
efficiency.

Ans. (a) 43.62 lb/s, (b) 6895 hp, (c) 8195 hp, (d) 48.22%

2. In a Brayton cycle, air enters the compressor at 101.32 kPa, 320C and leaves at a
pressure of 808 kPa. The air is heated to 7170C in the combustor. For a net output
of 2125 kW, compute (a) the rate of flow of air per second, (b) the thermal
efficiency, and (c) the mean effective pressure

Ans. (a) 10.78 kg/s, (b) 44.75%, (c) 145.65 kPa

3. Air is used as the working fluid in a simple ideal Brayton cycle that has a pressure
ratio of 12, a compressor inlet temperature of 3000K, and a turbine inlet
temperature of 10000K. Determine the required mass flow rate of air for a net
power output of 70MW.
Ans: 352 kg/sec

4. A stationary gas-turbine power plant operates on a simple ideal Brayton cycle


with air as the working fluid. The air enters the compressor at 95 kpa and 2900K
and the turbine at 760 kPa and 11000K. Heat is transferred to air at a rate
35,000kJ/sec. Determine the power delivered by this plant.

Page | 221
Cp Cv Cp Cv k R R
Gas Btu/lb- Btu/lb- kJ/kg-Ko kJ/kg-Ko Ft.lb/lb- J/kg-oK
Ro Ro o
R
Argon (A) 0.1244 0.0747 0.5215 0.3152 1.666 38.68 208.17

Helium 1.241 0.745 5.2028 3.1233 1.666 386.04 2077.67


(He)
Mercury 0.0248 0.0148 0.1039 0.0624 1.666 7.703 41.45
(Hg)
Neon 0.246 0.1476 1.0313 0.6188 1.666 76.57 412.10
(Ne)
Air 0.24 0.1714 1.0062 0.7186 1.4 53.342 287.08

Carbon 0.2487 0.1778 1.0426 0.7454 1.399 55.170 296.92


Monoxide
(CO)

Hydrogen 3.419 2.434 14.3338 10.2043 1.40 766.54 4125.52


(H2)
Nitrogen 0.2484 0.1775 1.0414 0.7442 1.399 55.158 296.86
(N2)
Oxygen 0.2194 0.1573 0.9198 0.6595 1.395 48.291 259.90
(O2)
Carbon 0.2016 0.1565 0.8452 0.6561 1.288 35.11 188.96
dioxide
(CO2)
Ammonia 0.499 0.382 2.0920 1.6015 1.304 90.73 488.31
(NH3)
Methane 0.5099 0.3861 2.1377 1.6187 1.321 96.33 518.45
(CH4)

Page | 222

Das könnte Ihnen auch gefallen